Назначение сердечных гликозидов



бет4/5
Дата23.03.2024
өлшемі321.83 Kb.
#496328
1   2   3   4   5
ИГА пробный БАРИ косылган

variant>Фармаконадзором+
Комплаенсом
Фармакоэтика
Фармакоэпидемиология
судебно-медицинская экспертиза





Вопросы на русском языке

























1

У пациента диагностирован острый коронарный синдром, сатурация поддерживается на уровне 90% путем инсуфляции кислорода через лицевую маску. Необходимо ингибировать миокардиальные эффекты циркулирующих катехоламинов и снизить потребление кислорода миокардом за счет снижения ЧСС, АД и сократимости сердца. Препараты какой группы необходимо назначить в данной клинической ситуации, согласно национальному клиническому протоколу?


























нитраты


























наркотические анальгетики


























антикоагулянты


























антиагреганты


























β-адреноблокаторы

























2

Пациент 75 лет доставлен в приемный покой в экстренном порядке с клиническими признаками кардиогенного шока, на фоне инфаркта миокарда передней локализации. Со слов родственников: перенес первый инфаркт миокарда 5 лет назад, страдает хронической сердечной недостаточностью, принимает фозиноприл, карведилол. Физикальное обследование: акроцианоз, мраморная влажная холодная кожа, АДср=56 мм.рт.ст. Какой препарат необходимо назначить в данной ситуации, с целью повышения сердечного выброса, с учетом собранного лекарственного анамнеза, согласно национальному клиническому протоколу?


























декстран


























сукцинилированный желатин


























метилпреднизолон


























адреналина гидрохлорид


























левосимендан

























3

Пациенту Р., 56 лет, в связи с недавним развитием фибрилляции предсердий, показана фармакологическая кардиоверсия. Назначение какого антиаритмического препарата наиболее целесообразно в данной клинической ситуации?


























амиодарон


























верапамил


























дилтиазем


























бисопролол


























лидокаин

























4

Пациентка 49 лет находится на стационарном лечении с диагнозом: Артериальная гипертензия степень 3, группа риска 4 (сахарный диабет, ОНМК). Гипертензивный криз, тип 2. Явления гипертензивного криза купированы, коррекция лечения сахарного диабета проведена эндокринологом. Какие гипотензивные препараты должна получать пациентка для рациональной фармакотерапии артериальной гипертензии в данной ситуации, согласно национального клиническому протоколу?


























спиронолактон, валсартан


























телмисартан, фозиноприл


























урапидил, фуросемид


























лозартан, индапамид+


























бисопролол, верапамил

























5

У пациентки диагностирована гестационная гипертензия.АД=160 и 105 мм.рт.ст. PS=ЧСС=100 в мин. Какой препарат необходимо назначить в данной клинической ситуации в качестве первоочередного лечения, согласно клиническому протоколу?


























магния сульфат


























телмисартан


























метилдопа


























фозиноприл


























индапамид

























6

Пациент 57 лет находится на стационарном лечении с диагнозом: ИБС. Стабильная стенокардия напряжения. ФК 2. Артериальная гипертензия степень 3 группа риска 4 (гипертрофия левого желудочка, стенокардия). ХОБЛ, преимущественно бронхитический тип, категория B, фаза ремиссии. Самостоятельно принимает нитраты с минимальным эффектом.Какой препарат должен получать пациент для фармакотерапии симптомов стенокардии в данной ситуации?


























нитроглицерин


























бисопролол


























триметазидин


























тофизопам


























амлодипин

























7

Пациент 49 лет с диагнозом: ИБС. Стабильная стенокардия напряжения. ФК 2. Артериальная гипертензия степень 3 группа риска 4 (гипертрофия левого желудочка), гиперлипидемия получает бисопролол с целью фармакотерапии симптомов стенокардии с хорошим эффектом, ацетилсалициловую кислоту. Какой препарат необходимо дополнительно назначить пациенту с целью профилактики кардиоваскулярных событий?


























эноксапарин натрия


























аторвастатин


























триметазидин


























тофизопам


























аргинин

























8

Пациент 64 лет, обратился с жалобами на кашель без отхождения мокроты, повышение температуры тела до 38,9ºС, одышку, болевые ощущения умеренной интенсивности в правой половине грудной клетки, возникающие при кашле и глубоком вдохе, выраженную общую слабость, сильную потливость и головную боль. Микробиологически – Chlamydophila pneumoniae.Какой препарат составит основу рациональной фармакотерапии в данной клинической ситуации?


























цефазолин 1 г 2 раза в сутки


























ципрофлоксацин по 0,25 г 3 раза в сутки


























кларитромицин 0,5 г 2 раза в сутки


























метронидазол 0,5 г 3 раза в сутки


























левофлоксацин 0,5 г 1 раз в сутки

























9

Пациент 26 лет, предъявляет жалобы на удушье, продолжающееся более 5 часов, прекращение выделения мокроты, отсутствие эффекта от ингаляций сальбутамола. В анамнезе бронхиальная астма, настоящее ухудшение связывает с самостоятельным прекращением приема глюкокортикостероидов. Объективно: пациент испуган, возбужден, положение ортопноэ, кожный покров бледно-серый, влажный, набухание шейных вен, ЧДД=55 в минуту, АД=80 и 60 мм.рт.ст, Ps=ЧСС=145 в минуту. Какие препараты составят основу рациональной фармакотерапии в данной ситуации?


























эпинефрин, преднизолон, сальбутамол, аминофиллин?+


























натрия хлорид, хлорпирамин


























адреналина гидрохлорид, натрия хлорид, бетаметазон


























триамцинолон, амоксициллина клавуланат


























глюкокортикостероиды, левосимендан

























10

Пациентка 60 лет, находится на стационарном лечении с диагнозом ХОБЛ, преимущественно эмфизематозный тип, категория С, фаза обострения. ДН III. Хроническое легочное сердце в стадии декомпенсации. ХСН ФК III. Антибактериальная терапия назначена адекватно.Какие препараты должна получать пациентка для рациональной фармакотерапии основного заболевания в данной ситуации, согласно национальному клиническому протоколу?


























бекламетазон, сальметерол


























сальбутамол, эуфиллин


























ацетилцистеин, беклометазон


























теофиллин, кетотифен


























монтелукаст, ипратропия бромид

























11

Вы назначаете теофиллин 28-летнему человеку с массой тела 50 кг, страдающему бронхиальной астмой. Нужно, чтобы концентрация препарата в плазме составила 15 мг/л. Объем распределения (Vd) теофиллина составляет 0,5 л/кг, а Cl = 0,04 л/ч/кг. Укажите нагрузочную дозу для этого пациента:


























300 мг


























375 мг


























575мг


























650мг


























720мг

























12

У больного с пневмонией при бактериологическом исследовании в мокроте был высеян стафилококк, продуцирующий бета-лактамазу. Пациенту назначен комбинированный антибиотик широкого спектра из группы пенициллинов, устойчивый к действию бета-лактамаз. Назовите препарат


























Амоксиклав


























Карбенициллин


























Ампициллин


























Тиенам


























Оксациллин

























13

Пациентка 25 лет. Беременность 12 недель. Внебольничная пневмония с локализацией в верхней доле справа, средней степени тяжести, смешанной (Mycoplasma pneumoniae+ Chlamydophila pneumoniae) этиологии. ДН I.Какой препарат составит основу рациональной фармакотерапии в данной клинической ситуации, согласно клиническому протоколу?


























амоксициллин/клавулановая кислота


























спирамицин


























тетрациклин


























доксицилин


























метронидазол

























14

Девочка, 13 лет. Жалобы на ежедневные приступы затрудненного дыхания в дневное время, снижение переносимости физических нагрузок.Дыхание свистящее, слышно на расстоянии, выдох удлинен. Частота дыханий 34 в минуту. Над легкими коробочный звук, аускультативно масса сухих свистящих и влажных хрипов по всей поверхности легких. На спирограмме объем форсированного вдоха за 1 секунду - 70 %.Назначение какого препарата из перечисленных целесообразно?


























Беклометазон 100мкг 2 раза сутки


























Аминофиллин 150мг в сутки


























Преднизолон 0,2мг\кг\сутки перорально


























Сальбутамол 100мкг 1 доза 4 раза в день


























Теопэк 300мг 1 раз в день

























15

Пациентка 34 лет находится на стационарном лечении с диагнозом: Хронический первичный двусторонний пиелонефрит, латентное течение, стадия обострения, активность 2. Хроническая болезнь почек стадия III (СКФ 60 мл/мин/1,73 м2). ХПН1.Нефрогенная анемия III степени. С целью фармакотерапии анемии назначен дарбэпоэтин альфа. Каковы возможные осложнения фармакотерапии в данном случае?


























тошнота, головокружение, отек нижних конечностей, артериальная гипотония


























разрыв крупных сухожилий, миалгии, артралгии, судороги


























развитие желудочковых аритмий, угнетение гемопоэза, гемосидероз,


























гипогликемия, синусит, язвенно-некротическое поражение миндалин, лейкоцитоз


























артериальная гипертензия, тромбоэмболии, боль в месте инъекции, сыпь

























16

Какая группа антигипертензивынх препаратов назначается для профилактики кардио-васкулярных событий у больных с хронической боленью почек?


























Тиазидные диуретики


























Селективные бета-блокаторы


























Блокаторы кальциевых каналов


























Неселективные бета-блокаторы


























Ингибиторы АПФ

























17

Пациент 30 лет находится на стационарном лечении с диагнозом: Хронический диффузный мембранозный гломерулонефрит, нефротический синдром, фаза обострения. Хроническая болезнь почек стадия IV (СКФ 30 мл/мин/1,73 м2). Артериальная гипертензия степень 3 группа риска 4(ХБП).Железодефицитная анемия на фоне хронической болезни почек, стадия Б, средней степени тяжести. Какие препараты должен получать пациент в данном случае в качестве иммуносупрессивной терапии?


























метотрексат,преднизолон


























метилпреднизолон, циклофосфамид ????


























сульфасалазин, циклоспорин


























азатиоприн, ритуксимаб


























инфликсимаб, преднизолон

























18

Пациент 25 лет находится на стационарном лечении с диагнозом: Хронический диффузный мезангиальный пролиферативный гломерулонефрит, нефротический синдром, фаза обострения. Хроническая болезнь почек 3 стадия (СКФ 60 мл/мин/1,73 м2). Артериальная гипертензия степень 3 группа риска 4(ХБП), гиперлипидемия.Железодефицитная анемия на фоне хронической болезни почек, стадия Б, средней степени тяжести. Какие препараты должен получать пациент в данном случае в качестве первой линии терапии исходя из морфологической формы гломерулонефрита?


























метотрексат,преднизолон


























метилпреднизолон, циклофосфамид?????


























периндоприл, аторвастатин +


























азатиоприн, ритуксимаб


























инфликсимаб, преднизолон

























19

Пациентка 34 лет находится на стационарном лечении с диагнозом: Хронический интерстициальный нефрит инфекционной этиологии, латентное течение, стадия обострения, активность 2. ХПН0. Бактериологический посев мочи: Escherichiacoli. Какой препарат необходимо назначить в данной ситуации для рациональной фармакотерапии?


























цефазолин


























цефуроксим


























ко-тримаксозол


























ампициллин


























метронидазол

























20

У пациента диагностирована первичная хроническая надпочечниковая недостаточность, средней степени тяжести в стадии декомпенсации. Какой препарат необходимо назначить в данном случае с целью заместительной глюкокортикоидной фармакотерапии?


























гидрокортизон


























терлипрессин


























цетрореликс


























триамцинолон


























карбетоцин

























21

Какой диуретик из нижеперечисленных при использовании в обычных дозах и при обычной длительности приема вызывает существенный сдвиг кислотно-щелочного равновесия в сторону метаболического ацидоза?


























Новурит


























Диакарб


























Фуросемид


























Маннитол


























Спиронолактон

























22

Пациенту 55 лет с Острым отравлением ядом прижигающего действия (уксусная кислота) тяжелой степени,химическим ожогом ротоглотки, пищевода, желудка, по жизненным показаниям проводится интубация трахеи. С учетом специфики основного заболевания,назначение какого мышечного релаксанта необходимо исключить в данной клинической ситуации?


























атракурия


























мивакурия


























векурония


























пипекурониума


























сукцинилхолина

























23

Пациентке 34 лет выставлен диагноз: Функциональная диспепсия. Постпрандиальный дистресс-синдром. Какие препараты составят основу фармакотерапии в данной клинической ситуации?


























ингибиторы протеолиза


























холеретики


























прокинетики


























ферменты


























гастропротекторы

























24

Эта группа препаратов приводит к повышению тонуса нижнего пищеводного сфинктера, стимуляция опорожнения желудка, улучшение координации работы пищеварительной системы. Наиболее эффективны они в составе комплексной терапии с антисекреторными препаратами. Предпочтительно использование нового класса препаратов, так какони лишены традиционных для своей группы побочных эффектов (его минимальная способность проникать через гематоэнцефалический барьер значительно снижает риск экстрапирамидных нарушений, гиперпролактинемии, кроме того, не взаимодействует с ферментами цитохрома Р-450, что позволяет избежать лекарственного взаимодействия в составе комплексной терапии. О какой группе препаратов при лечении гастроэзофагеальнорефлюксной болезни и язвенной болезни идет речь?


























Ингибиторы протонной помпы


























Прокинетики


























Антациды


























Препараты висмута


























Алгинаты

























25

Беременная женщина обратилась по поводу тошноты после приема пищи, тяжесть в правом боку, повышенную температуру. При обследовании обнаружено увеличение АлАТ и АсАТ в 5 раз, снижение альбуминов, увеличение общего билирубина в 3 раза. В серологии обнаружены анти НВеIg. При повторном обследовании выявлен HBS антиген. Врач-гастроэнтеролог поставил диагноз: Хронический гепатит В. Какая тактика врача будет при лечении данной болезни?


























Ацикловир


























Рибавирин


























Ламивудин


























Занамивир


























Интерферон-альфа +

























26

Больной К., 40 лет, страдающий язвенной болезнью, поступил в клинику с желудочным кровотечением. Из анамнеза стало известно, что 7 дней назад он заболел гриппом и принимал для снижения температуры препарат Х. Какой это препарат:


























Ибупрофен


























Ацетилсалициловая кислота


























Парацетамол


























Метамизол натрия


























Хлоропирамин

























27

Подросток 13 лет, с в положении стоя; ощущение давления в эпигастрии после еды, неприятный вкус во рту, чаще утром. Врачом назначено лечение, один из препаратов - алмагель.
Когда наиболее целесообразно принимать алмагель?


























За 20 минут до приема пищи


























Во время приема пищи


























Сразу после приема пищи


























Через 1 час после приема пищи


























За 30 минут до приема пищи

























28

Мальчик 12 лет. Жалобы на ноющие боли в животе и тошноту по утрам после приема пищи, снижение аппетита, отрыжку воздухом, головную боль, утомляемость. Объективно: живот умеренно вздут, болезненность в пилородуоденальной зоне. В общем анализе крови: эритроциты 3,6х1012/л, гемоглобин 110г/л, лейкоциты 10,5х109/л, СОЭ 15мм/час. Склонность к запорам. Биопсия: гиперплазия желудочных желез с преимущественным поражением обкладочных клеток.
Назначение какого препарата из перечисленных является первоочередным?


























Омепразол


























Метронидазол


























Дротаверин


























Амоксициллин


























Домперидон

























29

Пациент 25 лет, находится на стационарном лечении с диагнозом: Несахарный диабет, центральный (травматическая болезнь головного мозга), средней степени тяжести, в стадии декомпенсации. Какой препарат составит основу фармакотерапии в данной клинической ситуации?


























десмопрессин


























толваптан


























гидрокортизон


























спиронолактон


























гидрохлортиазид

























30

Пациент Б, 63-х лет, в течение длительного времени состоит на диспансерном учете с сахарным диабетом, осложненным диабетической нефропатией. Несмотря на контролируемый уровень сахара в крови, при обследовании выявлено повышение артериального давления до 180/100 мм рт ст. Какая группа антигипертензивынх препаратов является препаратами выбора у данного пациента?


























Тиазидные диуретики


























Ингибиторы АПФ


























Селективные бета-блокаторы


























Блокаторы кальциевых каналов


























Неселективные бета-блокаторы

























31

Пациентке 57 лет, страдающей сахарным диабетом 2 типа среднетяжелого течения в стадии компенсации и принимающей глибенкламид 3,5 мг 2 раза в сутки (перед завтраком и ужином), в связи с обострением хронического гастрита назначен ранитидин 150 мг 2 раза в сутки в это же время. Через 4 дня от начала терапии ранитидином пациентка стала отмечать периодически возникающие эпизоды резкой слабости, мышечной дрожи, при этом уровень глюкозы составлял 2,8 ммоль-1 . Какова причина возникновения гипогликемии у данной пациентки:


























Фармакодинамическое взаимодействие Ранитидина и Глибенкламида на уровне рецепторов


























Фармакокинетическое взаимодействие Ранитидина и Глибенкламида на уровне всасывания


























Фармакокинетическое взаимодействие Ранитидина и Глибенкламида на уровне связи с белками


























Фармакокинетическое взаимодействие Ранитидина и Глибенкламида на уровне метаболизма


























Фармакокинетическое взаимодействие Ранитидина и Глибенкламида на уровне выведения

























32

У пациентки 26 лет, Беременность 35 недель+6 дней, в анамнезе: сахарный диабет, тип 1, средней степени тяжести. Каковы особенности инсулинотерапии в данном случае, согласно клиническому протоколу?


























увеличение дозы на 50% от стандартной


























увеличение кратности приема инсулина










снижение дозы на 30% от стандартной


























применение только пероральных сахароснижающих препаратов


























интенсивная инсулинотерапия в режиме многократных инъекций

























33

Бригадой скорой помощи доставлен больной сахарным диабетом 1 типа, заторможен, апатичен, кожный покров сухой, дыхание шумное со слабым запахом ацетона, тоны глухие, тахикардия до 110 ударов с единичными экстрасистолами, АД 80/60 мм.рт.ст., язык сухой малиновый, живот болезненный при пальпации в эпигастрии, олигурия.
С каких лекарственных препаратов следует начать экстренную помощь?


























Коллоидных и кристаллоидных растворов (10% от массы тела за сутки)


























Инсулина короткого действия 10-12 ЕД в час с кристаллоидами в другую вену










Раствора хлорида калия, доза в зависимости от уровня калия в крови


























Раствора добутамина с кристаллоидами (раствор Рингера, 5% глюкозы)


























Инсулина короткого действия 20 ЕД болюсом и поляризующая смесь

























34

Женщина 40 лет, после перенесенной тиреоидэктомии, жалобы на тошноту, учащенное сердцебиение, тремор рук, одышку, тошноту. Пациентка возбуждена. Лицо гиперемировано, кожные покровы иктеричны, температура тела 39ºС. Сердечные тоны приглушены, тахикардия до 140 в минуту, АД 100/70 мм. рт.ст.
Врачом приемного покоя выставлен диагноз тиреотоксический криз. Укажите первоочередной препарат для купирования криза?


























Амиодарон


























Пропроналол


























Супрастин


























Йодит натрия


























Атропин

























35

У пациента 11 лет диагностирован Сахарный диабет 1 типа. В виду лабильного течения заболевания и большой вариабельности гликемии в течение суток вне зависимости от уровня гликозилированного гемоглобина, пациент переведен на непрерывную подкожную инфузию инсулина путем помповой терапии. Какие препараты инсулина должны быть применены в качестве препаратов выбора в данной клинической ситуации?


























ультракороткого действия


























средней продолжительности действия


























длительного действия


























быстрого действия


























инсулиноподобные беспиковые препараты

























36

Пациентке 64 лет с целью предотвращения рецидивов подагрического артрита и профилактики осложнений, связанных с неконтролируемой гиперурикемией назначен аллопуринол в стартовой дозе 100 мг в сутки. Каковы индикаторы эффективности антигиперурикемической терапии в данном случае?


























уровень мочевой кислоты ниже 0,36 ммоль/л, рассасывание тофусов


























количество лейкоцитов в крови ниже 10 х 109/л, регресс болевого синдрома


























титр антинуклеарных антител ниже 1:160, подавление прогрессирования фиброза


























С-реактивный белок ниже 5 мг/л, снижение активности воспалительного процесса


























креатинфосфокиназа ниже 200 Ед/л, улучшение функциональной мышечной активности

























37

Пациентка 56 лет находится на стационарном лечении с диагнозом: Ревматоидный артрит серопозитивный, развернутая стадия, степень активности 2 (DAS28=3,5), эрозивный (рентгенологическая стадия 2 Б), с системными проявлениями (полинейропатия).НФС 2. Какой препарат с наилучшим соотношением эффективность/токсичность составит основу рациональной фармакотерапии в данной клинической ситуации?


























метотрексат


























метилпреднизолон


























лорноксикам


























ритуксимаб


























преднизолон

























38

Пациент 18 лет, находится на стационарном лечении с диагнозом: Острая ревматическая лихорадка, кардит (митральный вальвулит), мигрирующий полиартрит, ХСН I. ФК I. Активность процесса высокая-уровень СОЭ=50 мм/ч. Какой препарат составит основу патогенетической фармакотерапии в данном случае, согласно национальному клиническому протоколу?


























метотрексат


























варфарин


























диклофенак


























азитромицин


























преднизолон

























39

Больная Н., доставлена в травматологический пункт с закрытым переломом левой лучевой кости. Из анамнеза стало известно, что пациентка страдает ревматоидным артритом и регулярно получает базисную терапию. При рентгенологическом обследовании выявлен диффузный остеопороз. Какой препарат мог его спровоцировать?


























Метотрексат


























Сульфасалазин


























Лефлуномид


























Преднизолон


























Инфликсимаб

























40

У пациентки с системной красной волчанкой на фоне длительной фармакотерапии метилпреднизолоном в средних терапевтических дозах, были зафиксированы признаки угнетения функции надпочечников. К какому типу побочных эффектов лекарственных средств относится данная симптоматика?


























эффекты длительного применения


























непрогнозируемые эффекты


























идиосинкразия


























отсроченные эффекты


























непредсказуемая неэффективность

























41

Девочка 3 лет. Жалобы на боли и припухлость в правом коленном суставе при движениях, чаще в утренние часы, повышении температуры до 39С0. Объективно: правый коленный сустав отечный, конфигурирован, увеличен в обьеме, кожа на ощупь горячая, максимальное разгибание ограничено. В общем анализе крови: эритроциты 3,5х1012/л, гемоглобин 112г/л, СОЭ 36 мм/час.
Препараты какой группы из перечисленных целесообразны
при присоединении интеркуррентных заболеваний?


























Карбапенемы


























Аминогликозиды


























Цефалоспорины


























Макролиды


























Полусинтетические пенициллины+

























42

Больного ревматоидным артритом, длительное время принимающего НПВС, стали беспокоить боли в животе, тошнота, изжога. Какой препарат гастропротекторного действия наиболее целесообразно назначить для предупреждения подобных осложнений?


























Мизопростол+


























Сукральфат


























Ликвиритон


























Карбеноксолон


























Коллоидная соль висмута

























43

Пациенту 20 лет, с диагнозом: Эпилепсия генерализованная судорожная форма, с частыми эпилептическими приступами до 7-8 раз в месяц, осложненная психоорганическим синдромом, в качестве монотерапии назначен препарат карбамазепин в стартовой дозе 200 мг. Какова скорость наращивания дозы, согласно данным национального клинического протокола ?


























200 мг в неделю


























0,5 мг каждые 4 дня


























1000 мг каждые 2 недели


























500 мг в неделю


























5 мг/кг в неделю

























44

Пациенту 39 лет, с диагнозом: Эпилепсия, генерализованная судорожная форма, осложненная психоорганическим синдромом, назначена вальпроевая кислота в качестве монотерапии. Какова стартовая доза препарата, согласно данным национального клинического протокола ?


























500-1000мг


























25-50 мг


























600-1200 мг


























2-6 мг


























10-30 мг

























45

У пациента с диагностированной Болезнью Паркинсона, получающего леводопу в качестве симптоматической фармакотерапии, зафиксированы моторные флуктуации по типу «истощения конца дозы». Какие методы коррекции фармакотерапии целесообразно предпринять в данном случае, согласно национальному клиническому протоколу?


























полностью отменить препарат


























увеличить кратность приема при сохранении суточной дозы+


























уменьшить кратность приема при увеличении суточной дозы


























сохранить кратность приема приуменьшении суточной дозы


























дополнительно назначить толтеродин

























46

У пациентки 17 лет диагностирована Мигрень с типичной офтальмической аурой, тяжелого течения с преимущественной локализацией боли в лобно-височно-глазничной области справа. Какой препарат, со специфическим механизмом действия, составит основу фармакотерапии в данной ситуации?


























метамизол натрия


























пирацетам


























трамадола гидрохлорид


























парацетамол


























суматриптан

























47

Пациентка 45 лет, принимает карбамазепин по поводу невралгии тройничного нерва, в связи с гипертонической болезнью назначен верапамил-SR в дозе 240 мг в сутки. Несмотря на прием верапамила, АД у пациентки практически без тенденции к снижению. Какова причина неэффективности верапамила у данной пациентки?


























Фармакодинамическое взаимодействие Карбамазепина и Верапамила на уровне рецепторов


























Фармакокинетическое взаимодействие Карбамазепина и Верапамила на уровне всасывания


























Фармакокинетическое взаимодействие Карбамазепина и Верапамила на уровне связи с белками


























Фармакокинетическое взаимодействие Карбамазепина и Верапамила на уровне метаболизма


























Фармакокинетическое взаимодействие Карбамазепина и Верапамила на уровне выведения

























48

У ребенка 5-месячного ребенка с ОРВИ на фоне температуры 39,6 С впервые возникли судороги. Какой из перечисленных лекарственных средств будет препаратом выбора:


























пипольфен


























фенобарбитал


























кофеин


























дроперидол


























дифенин

























49

У пациента 29 лет, диагностировано обсессивно-компульсивное расстройство с сопутствующими депрессивными проявлениями. Какой препарат составит основу фармакотерапии в данной ситуации, согласно национальному клиническому протоколу?


























дулоксетин


























цитиколин


























рисперидон


























тригексифенидил


























бетагистин

























50

Мужчина 49 лет, состоит на учете у психиатра. Принимает препараты лития. Когда целесообразно взять образцы крови для получения максимальной информации при измерении концентрации лития?


























Через 24 часа после приема последней дозы


























Через 18 часов после приема последней дозы


























Через 12 часов после приема последней дозы+


























Через 9 часов после приема последней дозы


























Через 6 часов после приема последней дозы

























51

Мальчик 9 лет, выявлено нарушение обучения, снижение интеллектуального развития. Назначение какой группы психотропных средств оправдано в данном случае?


























Антидепрессантов


























Ноотропов


























Транквилизаторов


























Нейролептиков


























Адаптогенов

























52??

Женщина 58 лет с жалобами на нервозность, тревогу, эмоциональную напряженность, нарушение сна, частое повышение АД, в связи с чем назначен диазепам в суточной дозе 10 мг и раунатин по 1 тб 2 раза в день. В анамнезе хронический гепатит. Через 2 недели больная отметила появление галлюцинаций, ярких сновидений, вялость и дрожание пальцев рук. Что привело к появлению побочного действия диазепама?


























Назначение высокой дозы препарата


























Повышение всасывания препарата из ЖКТ


























Замедление процесса биотрансформации препарата в организме


























Повышение выведения препарата из организма


























Взаимодействие диазепама и раунатина

























53

Пациентка 35 лет. На третьи сутки после родов появились жалобы на общую слабость, озноб, жажду, сердцебиение, тошноту. Объективно: в сознании, заторможена, кожный покров холодный, мраморный, t тела 35,4 0С, на передней поверхности грудной клетки точечные экхимозы. ЧДД=25в минуту. Аускультативно: дыхание жесткое, рассеянные хрипы на выдохе. АД=85 и 40 мм.рт.ст ЧСС=Ps=125 в минуту. ОАК: лейкопения, токсическая зернистость нейтрофилов, увеличение СОЭ. Сахар крови 8 ммол/л. Какие препараты определят эффективность терапии в условиях реанимационного отделения в данной клинической ситуации?


























кристаллоиды


























глюкокортикостероиды


























пероральные сахароснижающие средства


























внутривенные антибиотики


























инсулиноподобные беспиковые препараты

























54

У больного в стационаре резко изменилось поведение: ведет себя хаотично, спасается от воображаемых преследователей, пытается выпрыгнуть в окно, нападает на окружающих. Для купирования психомоторного возбуждения больному показан:


























галоперидол


























амитриптилин


























новопассит


























персен


























адаптол

























55

Пациент находится на стационарном лечении с диагнозом: Цирроз печени, в исходе вирусного гепатита D, смешанный. Класс тяжести по Чайлд-Пью - C. Декомпенсированная портальная гипертензия: пищеводная флебэктазия II степени, асцит. Декомпенсированная печеночно-клеточная недостаточность. Печеночная энцефалопатия II стадии. Какие препараты должен получать пациент для фармакотерапии асцита в данном случае?


























ацетазоламид, торасемид


























гипотиазид, маннитол


























индапамид, мочевина


























спиронолактон, фуросемид


























торасемид, толваптан

























56

В рамках комплексного подхода к фармакотерапии железодефицитной анемии, пациентке 36 лет, в сроке беременности 23-24 недели была назначена фолиевая кислота. Какова доза препарата, необходимая пациентке, согласно рекомендациям Всемирной Организации Здравоохранения?


























60 мкг


























120 мг


























400 мкг?


























1100 мкг


























1300 мг

























57

Пациент Р, 45 лет, предъявляет жалобы на внезапно возникшую и постоянно нарастающую отечность в области губ, щек, лба. В анамнезе: аллергия на пищевой краситель тартразин, настоящее ухудшение связывает с рассасыванием леденца от кашля. С какого препарата необходимо начать неотложную фармакотерапию в данном случае, согласно национальному клиническому протоколу?


























хлоропирамина


























атропина


























триамцинолона


























сальбутамола


























аминофиллина

























58

Пациентке назначен перорально препарат джозамицин с целью фармакотерапии угревой сыпи, для избавления от симптомов сезонного аллергического ринита пациентка самостоятельно принимает терфенадин. Каковы возможные последствия лекарственных взаимодействий в данном клиническом случае?


























разрыв крупных сухожилий, артралгии


























алопеция, гиперчувствительный пневмонит


























развитие желудочковых аритмий


























миелосупрессия, олигоменорея


























ретинопатия, нейромиопатия

























59

Пациент 48 лет находится в отделении реанимации и интенсивной терапии с диагнозом: Острый калькулезный холецистит. Механическая желтуха. Гнойный холангит. Сепсис. Септический шок. Какой препарат необходимо назначить в качестве старта инфузионно-трансфузионной терапии, в условиях исходной гипотонии, в данном случае согласно клиническому протоколу лечения?


























натрия хлорид 0,9%


























альбумин 10%


























глюкоза 5%


























гидроксиэтилкрахмал 6%


























бикарбонат натрия 4%

























60

Мужчина З., 35 лет, жалоб нет. При прохождении медицинской комиссии терапевт обратила внимание на иктеричность склер. В анамнезе частые острые респираторные инфекции. Курит, работает в химической лаборатории. При физикальном обследовании повышенного питания, печень выступает из-под края реберной дуги на 1,0 см из-под края реберной дуги, безболезненная, обычной консистенции. В ОАК обращает на себя внимание высокое содержание гемоглобина 148 г/л, эритроцитов 5,1х 1012 л, ретикулоцитоз 3%. В биохимическом анализе крови – умеренная гипербилирубинемия- 48,41 мкмоль/л за счет непрямой фракции. Ваш препарат выбора.


























Иммуноглобулин


























Метионин


























Фенобарбитал


























Урзодезоксихолевая кислота


























Статины

























61

Известно, что парацетамол – слабая кислота. Какие рекомендации по применению парацетамола вы дадите с учетом его фармакокинетики? Парацетамол следует принимать внутрь:


























До еды за 2 часа


























Во время приема пищи


























Сразу после еды


























Через 2 часа после еды


























Вне связи с приемом пищи

























62

У пациента 10 лет диагностирована Доброкачественная вторичная внутричерепная гипертензия, в результате перенесенной черепно-мозговой травмы. Какие препараты составят основу фармакотерапии отека головного мозга, в данной клинической ситуации, согласно национальному клиническому протоколу?


























магния сульфат, пиридоксин, тиамин


























маннитол, фуросемид, ацетазоламид


























гидрохлортиазид, спиронолактон, триамтерен


























индапамид, цитиколин, пирацетам


























натрия цитрат, нимодипин, холина альфосцерат

























63

У ребенка 5-ти лет с хронической болезнью почек 4 стадии развилась анемия средней степени тяжести. Выберите оптимальное лечение анемического синдрома у пациента и критерий его эффективности.


























Парентеральные препараты железа под контролем уровня элементарного железа


























Пероральные формы препаратов железа под контролем уровня сывороточного железа


























Рекомбинантный человеческий эритропоэтин парентерально под контролем гемоглобина


























Рекомбинантный гормон роста под контролем уровня сывороточного железа


























Активные формы витамина Дпод контролем уровня фосфора крови и кальция в крови

























64

У пациента 40 лет перед операцией возникло психомоторное возбуждение, какой из перечисленных препаратов предпочтителен в данной клинической ситуации с целью седации?


























дулоксетин


























натрия вальпроат


























диазепам


























рисперидон


























галоперидол

























65

В ходе клинико-фармакологической экспертизы медицинской карты новорожденного был зафиксирован факт развития ядерной желтухи после применения препарата цефтриаксон в целях фармакотерапии пневмонии. Какие особенности фармакокинетики препарата в период новорожденности объясняют развитие данного побочного эффекта?


























большой объем распределения


























низкая скорость клубочковой фильтрации


























высокая скорость кровотока


























низкая связывающая способность альбуминов плазмы


























высокая скорость клубочковой фильтрации

























66

У пациента 6 лет диагностирован Эпидемический паротит, среднетяжелая форма. Температура тела 39,20С. Какой препарат необходимо назначить с целью купирования гипертермического синдрома в данной клинической ситуации?


























ацетилсалициловую кислоту


























парацетамол


























диклофенак натрия


























дифенгидрамин


























метамизол натрия

























67

У пациентки 10 лет диагностирован Термический ожог (кипятком) 28% ПТ (IIIБ-IV=12%)/ IIIБ-IV степени спины, ягодиц. Ожоговый шок тяжелой степени. С целью фармакотерапии болевого синдрома назначен морфина гидрохлорид. Каковы доза и кратность введения препарата в данной ситуации, согласно национальному клиническому протоколу?


























2,5-10 мг каждые 2 часа


























100 мкг/кг каждые 6 часов


























200 мкг/кг каждые 4 часа


























2мкг/кг каждые 30 минут


























4-8 мг/кг каждые 8 часов

























68

У пациента 12 лет диагностирована Гипертрофическая кардиомиопатия, симметричная, необструктивная форма, II степени в стадии субкомпенсации.
В анамнезе: Бронхиальная астма. Какие группы препаратов составит основу фармакотерапии в данном случае, согласно национальному клиническому протоколу ?


























β-адреноблокаторы, тиазидные диуретики


























блокаторы рецепторов ангиотензина II, петлевые диуретики


























антагонисты альдостерона, альфа1-адреноблокаторы


























антагонисты кальция, ингибиторы АПФ


























ингибиторы карбоангидразы, нитраты

























69

У пациента З., 13 лет с протезированными клапанами сердца развился инфекционный эндокардит, вызванный штаммами метициллин-резистентного стафилококка. Получает ванкомицин в сочетании с гентамицином. Назначение какого препарата дополнительно, наиболее целесообразно с целью эрадикации бактерий при имплантированных инородных материалах, согласно рекомендации клинического протокола?


























ампициллина


























цефазолина


























Рифампицина?


























пенициллина G


























оксациллина

























70

Пациент М., 5 лет, находится на стационарном лечении с диагнозом: Муковисцидоз, смешанная форма, тяжелой степени тяжести. Вторичный хронический бронхит, обострение. Множественные бронхоэктазы (мешотчатые в S1–S6 справа, цилиндрические в S4–S8 слева). Хроническая синегнойная инфекция. Экзокринная недостаточность поджелудочной железы, Какие препараты составят основу рациональной фармакотерапии в данном случае?


























дорназа альфа, ацетилцистеин, тобрамицин, панкреатин


























азитромицин, урсодезоксихолиевая кислота, токоферола ацетат, аминокислоты


























увлажненный кислород, метилпреднизолон, метоклопрамид, морфина гидрохлорид


























бетаметазон, циклофосфамид, холекальцеферол, ондасетрон, тромбоконцетрат


























пиридоксина гидрохлорид, будесонид, аминофиллин, ко-тримоксазол, альбумин

























71

Пациентке 22 лет, в активном периоде родов, предложено обезболивание. Согласие на проведение регионарной анестезии получено. Сразу после эпидурального введения 5 мл 0,5% раствора бупивакаина у роженицы появилось онемение губ, языка, чувство тревоги, нечеткость зрения. АД=80 и 60 мм.рт. ст. О чем свидетельствуют появившиеся симптомы?


























непреднамеренном внутрисосудистом введении


























нормальном течении анестезии


























передозировке анестетика


























недостаточной дозе анестетика


























психоэмоциональной лабильности пациентки

























72

У пациентки 22 лет, через двое суток после родов, диагностирован сепсис, септический шок. АДср =55 мм.рт.ст. Какой препарат составит основу фармакотерапии сосудистой недостаточности в данном случае, согласно нациоанальному клиническому протоколу лечения?


























дигоксин


























норэпинефрин


























гидрокортизон


























левосимендан


























нитроглицерин

























73

Пациентка 38 лет. Беременность 25 недель +3 дня, в экстренном порядке доставлена в приемный покой.Предъявляет жалобы на внезапно возникшую одышку, кашель, кратковременный эпизод потери сознания, боль за грудиной. Объективно: кожа бледная, холодная, акроцианоз. АД=70 и 50 мм.рт.ст. ЧСС=Ps=125 в мин. Эхокардиография: расширение правых отделов сердца, парадоксальное движение межжелудочковой перегородки в диастолу. Какой препарат необходимо назначить в данной ситуации, для улучшения сократительной способности миокарда?


























добутамин


























норадреналина гидротартрат


























натрия хлорид


























метилпреднизолон


























адреналина гидрохлорид

























74

Пациентка 40 лет. Беременность 12-13 недель. Диагностирована тромбоэмболия легочной артерии.Для коррекции артериальной гипотонии назначен препарат адреналина гидрохлорид в дозе 2 мкг/мин. В течении какого времени сохранятся гемодинамические эффекты препарата, после завершения инфузии?


























12-24 часа


























1-5 минут


























1-2 минуты


























4-6 часов


























1-3 секунды

























75

Пациентке с раним послеродовым кровотечением, необходимо снизить трансфузионную нагрузку на систему гемоциркуляции. Лабораторно подтверждена коагулопатия: ПТИ=75%; фибриноген 1,2 г/л. Какой препарат необходимо назначить для коррекции гемостаза в данной ситуации, согласно клиническому протоколу лечения?


























плазма свежезамороженная


























мизопростол


























транексамовая кислота


























криопреципитат


























эритроцитарная масса

























76

Пациентка 37 лет, кесарево сечение в родах. Сопутствующая патология: экзогенно-конституциональное ожирение, андроидный тип, стабильное. ИМТ=43 кг/м2. Какой препарат составит основу профилактики сосудистых событий в данной ситуации, согласно клиническому протоколу и лечения?


























дабигатрана этексилат


























варфарин


























клопидогрел


























эноксапарин натрия


























ацетилсалициловая кислота

























77

Пациентка 23 лет. Диагностирован послеродовый сепсис. Назначена комбинация из 2 антибактериальных препаратов. Какова максимально возможная продолжительность антибиотикотерапии в эмпирическом режиме, до бактериологической идентификации профиля инфекции и определения чувствительности, согласно клиническому протоколу лечения?


























7-14 суток


























3-5 суток


























21-24 дня


























1-2 месяца


























6-12 часов

























78

У пострадавшего с множественными травмами груди и таза зафиксированы явления травматического шока. C целью фармакотерапии болевого синдрома назначен трамадола гидрохлорид. Какие особенности фармакодинамики препарата определяют его способность прерывать шокогенную импульсацию?


























торможение нейронального захвата норадреналина и серотонина, стимуляция опиатных рецепторов


























угнетение синтеза простагландинов, обусловленное ингибированием фермента циклооксигеназы


























блокада опиатных (преимущественно мю-) рецепторов ЦНС, спинного мозга и периферических тканей.


























стабилизация мембраны нейронов, в результате снижения их проницаемости для ионов натрия


























блокада глутаматных NMDA–рецепторов нейронов головного мозга, быстрое образование новых межнейрональных синапсов

























79

Пациенту 65 лет предстоит оперативное вмешательство с целью фиксации прямой мышцы глаза. Какой препарат необходимо назначить с целью профилактики рефлекторной остановки сердца, в результате возникновения окуловисцерального рефлекса?


























атропин


























верапамил


























лидокаин


























дилтиазем


























амиодарон

























80

Пациентка 19 лет. Беременность 18-20 недель, проведено вскрытие и дренирование абсцесса, иссечение пораженной крипты и сфинктеротомия, по поводу межсфинктерного парапроктита. Какой препарат допустимо назначить в данной клинической ситуации с целью антибиотикопрофилактики?


























левофлоксацин


























тетрациклин


























моксифлоксацин


























цефтриаксон


























кларитромицин

























81

Пациент 4 лет прооперирован по поводу деструктивного аппендицита. Послеоперационный период протекает с признаки изотонической дегидратации I степени.
Какой препарат составит основу фармакотерапии нарушений водно-солевого баланса в данном клиническом случае?


























натрия гидрокарбонат 4%


























декстроза 5%


























калия хлорид 7,4%


























гидроксиэтилкрахмал 6%


























натрия хлорид 0,9%

























82

В ходе предоперационного осмотра пациента анестезиологом был выявлен факт наличия у пациента пищевой аллергии на белок куриного яйца и арахис. Назначение какого внутривенного анестетика необходимо исключить в ходе предстоящего анестезиологического пособия, с учетом данных аллергологического анамнеза?


























тиопентала натрия


























кетамина


























галотана


























пропофола


























фентанила

























83

Пациенту предстоит оперативное вмешательство по поводу ампутации нижней правой конечности. Планируется проведение тотальной внутривенной анестезии с применением суксаметония хлорида перед интубацией трахеи, назначение каких антибактериальных препаратов в предоперационном периоде приведет к усилению нервно-мышечной блокады и задержке восстановления проводимости в данном случае?


























амоксициллина, цефазолина, тетрациклина


























гентамицина, ванкомицина, пиперациллина


























левофлоксацина, хлорамфеникола, метронидазола


























кларитромицина, доксициллина, тинидазола


























меропенема, моксифлоксацина, цефепима

























84

Пациентке 43 лет предстоит оперативное вмешательство - холицистэктомия. В анамнезе Бронхиальная астма, средней степени тяжести, контролируемое течение. ДН 0. Какое из нижеперечисленных наркозных средств, обладающий бронходилатирущими свойствами, предпочтительно назначить в данной клинической ситуации?


























пропофол


























кетамин


























фентанил


























тиопентал натрий


























галотан

























85

Пациенту при проведении наркоза анестезиолог с целью антибиотикопрофилактики назначил антибактериальный препарат. У больного развилась остановка дыхания. Какой препарат из нижеперечисленных вероятнее всего вызвал осложнение?


























Гентамицин


























Цефепим


























Ампициллин


























Спиромицин


























Амоксиклав

























86

Пациенту 57 лет, предстоит оперативное вмешательство - металлоостеосинтез костей бедра под ингаляционной анестезией. В анамнезеИБС. Стабильная стенокардия напряжения. ФК 2. Артериальная гипертензия степень 3, группа риска 4 (гипертрофия левого желудочка, стенокардия). Назначение какого препарата для ингаляционного наркоза необходимо исключить в данной клинической ситуации с учетом сопутствующей патологии?


























галотана


























ксенона


























севофлурана


























изофлурана


























десфлурана

























87

Пациенту с проникающим ранением глазного яблока-OD, необходимо оперативное вмешательство в условиях отделения микрохирургии глаза. Какой препарат должен быть назначен в данном случае, с целью проведения терминальной анестезии?


























лидокаин


























прокаин


























бупивакаин


























оксибупрокаин


























ропивакаин

























88

Пациенту с диагностированной порфирией предстоит оперативное вмешательство с применением тотальной внутривенной анестезии. Применение какого препарата необходимо исключить в данной клинической ситуации?


























тиопентала натрия


























кетамина










галотана


























пропофола


























фентанила

























89

Роженица 20 лет, повторные роды. Пациентка настаивает на обезболивании родов. Согласие роженицы на проведение эпидуральной анестезии получено. Какие препараты составят основу рациональной фармакотерапии боли в данной клинической ситуации?


























фентанил+бупивакаина гидрохлорид


























кислород+динитрогена оксид 50:50


























ибупрофен+парацетамол


























пропофол 100 мг+кетамин 50 мг


























прокаин 0,5%+лидокаина гидрохлорид 1%

























90

Роженица 25 лет, первые роды. Жалобы на схваткообразные боли внизу живота, пояснице, крестце и паховых областях. Схватки регулярные: 3 схватки за 10 минут, длительность 20-25 секунд. Положение плода продольное, предлежание головное. ЧССП=120 удв мин. Шейка матки сглажена, раскрытие 3 см. АД=130 и 80 мм.рт.ст. ЧСС=Ps=90 уд в мин., t тела -36,80 Роженица настаивает на обезболивании родов, однако от эпидуральной анестезии отказывается. Какие препараты составят основу рациональной фармакотерапии боли в данной клинической ситуации?


























кислород+динитрогена оксид 50:50?+


























фентанил+бупивакаина гидрохлорид


























ибупрофен+парацетамол


























пропофол 100 мг+кетамин 50 мг


























прокаин 0,5%+ лидокаина гидрохлорид 1%

























91

Какой характер кровотечения служит признаком наружного артериального кровотечения


























Медленное кровотечение


























Кровь сочится по каплям


























Медленное и тягучее кровотечение


























Кровь темно-красного цвета


























Быстрое и пульсирующее кровотечение

























92

Достоверным признаком клинической смерти является




























Отсутствие пульса на сонной артерии




























Сужение зрачков




























Бледность кожных покровов




























Появление трупных пятен




























Понижение АД

























93

Эффективным мероприятием оказания неотложной помощи при отёке лёгких является




























усаживание пациента в положение Фаулера, наложение жгутов на бёдра до исчезновения пульса




























придание пациенту положение с поднятым ножным концом




























подача дыхательных аналептиков через небулайзер




























применение наркотических анальгетиков




























усаживание пациента в положение ортопноэ, наложение жгутов на бёдра и плечо одной руки до исчезновения пульса

























94

Для эффективной реанимации взрослому, частота компрессий грудной клетки должна составлять




























60-80 в минуту




























100-120 в минуту




























более 120 в минуту




























80-120 в минуту




























не менее 90 в минуту

























95

Появление у пациента при кашле мокроты алого цвета указывает на




























отёк лёгких




























лёгочное кровотечение




























желудочное кровотечение




























рак лёгких




























бронхоэктатическую болезнь

























96

Определите диагностическое исследование, которое позволит провести дифференциаьный диагноз. У ребенка 6 лет недомогание, слабость, боль в животе, тошнота рвота, шумное дыхание, запах ацетона в выдыхаемом воздухе.




























Общий анализ крови




























Общий анализ мочи




























Определение уровня сахара в крови




























Ультразвуковое исследование




























Компьютерная томография

























97

Шкала Глазго используется для оценки:




























уровня угнетения сознания




























степени тяжести шока




























выраженности дыхательных расстройств при коме




























состояния новорожденного




























вида комы

























98

Основным ЭКГ признаком некроза участка сердечной мышцы является:




























снижение сегмента ST




























широкий и глубокий зубец Q+




























подъем сегмента ST




























уменьшение зубца R




























отрицательный зубец Т

























99

Укажите раствор для промывания желудка при остром отравлении уксусной кислотой:




























слабый раствор перманганата калия




























1% раствор натрия гидрокарбоната




























слабоконцентрированный раствор лимонной кислоты




























чистая вода




























3% раствор натрия хлорида

























100

У пожилого пациента внезапно появилась острая боль за грудиной, затем сухой кашель, головокружение. Чуть позже развилась одышка инспираторного типа и диффузный цианоз в сочетании с бледностью кожи. Визуально определяется патологическая пульсация в эпигастральной области. Перкуторно отмечается - расширение правой границы сердца, аускультативно - акцент II тона и раздвоение его над легочной артерией. Состояние, возникшее у пациента можно идентифицировать как:




























спонтанный пневмоторакс




























тромбоэмболия легочной артерии




























клапанный пневмоторакс




























острый коронарный синдром




























сухой плеврит
























ВЫБЕРИТЕ ОДИН ИЛИ НЕСКОЛЬКО ПРАВИЛЬНЫХ ВАРИАНТОВ


1.К лекарственным средствам рецептурного отпуска относятся?//
A.ЛС, содержащие наркотические средства, психотропные вещества и прекурсоры, подлежащие контролю в РК//
B.ЛС, предназначенные для парентерального введения//
C.Диагностические лекарственные средства, применяемые только в условиях стационара//
D.ЛС, назначение и дальнейшее употребление которых необходимо проводить при постоянном контроле врача//
+E.Все перечисленное
2. Сколько категорий лекарственных препаратов по степени риска для плода выделяют на основании рекомендации FDA?
A.7
+B.5
C.3
D.4
E.6


3. Выберите утверждение, не соответствующее приказу МЗ РК от 22 мая 2015 года №373 «Об утверждении правил выписывания, учета и хранения рецептов»//
А. Рецепты в бумажной форме заверяются подписью медицинского работника, выписавшего рецепт//
В. Рецепты не выписываются на лекарственные средства, применяемые исключительно в организациях здравоохранения, оказывающих стационарную помощь//
+C. Рецепты на лекарственные средства, не зарегистрированные и не разрешенные к применению в Республике Казахстан, выписываютсяна латинском языке//
D. Рецепты выписываются с учетом возраста больного и характера действия входящих в состав лекарственного средства ингредиентов//
E. Содержание и номер рецепта на бесплатное или льготное получение лекарственных средств отражаются в амбулаторной карте больного
4. Какое количество наименованийлекарственныхсредств указывается на одном рецептурном бланке на получение бесплатно или на льготной основепроизводных 8-оксихинолина,гормональных и анаболических стероидов, клонидина, кодеина, тропикамида, циклопентолата, трамадола, буторфанола?//
+A. одно//
B.не более двух//
C.не более трех//
D.не более четырех//
E.не более пяти
5. Какое количество наименований лекарственныхсредств указывается на одном рецептурном бланке на получение бесплатно или на льготной основеэналаприла, бисопролола, нифедипина, сальбутамола, амоксициллина?//
A. одно//
+B. не более двух//
C. не более трех//
D. не более четырех//
E. не более пяти
6. Выберите правильный ответ. При выписывании лекарственного средства в дозе, превышающей высшую разовую, врач в рецепте обозначает дозу лекарственного средства прописью и …://
А. вопросительным знаком//
+В. восклицательным знаком//
С. опознавательным знаком «Стоп – медикамент высокого риска»//
D. опознавательным знаком «Стоп – медикамент схожий по названию»//
E. опознавательным знаком «Стоп – высшая разовая доза»
7. Как правильно выписываютсярецепты?//
А. на латинском языке под международным непатентованным наименованием лекарственных средств//
B. в случае индивидуальной непереносимости пациента, многокомпонентного состава лекарственного препарата выписываются под торговым наименованием//
C. рецепты выписываются разборчиво с обязательным заполнением всех предусмотренных в бланке граф//
+D. все перечисленное верно//
E. способ применения прописывается в соответствующей графе с указанием дозы, частоты и длительности приема
8. Выберите правильное утверждение://
A. Рецепты на получение лекарственных средств бесплатно или на льготных условиях дополнительно подписываются уполномоченным лицом, определенным приказом медицинской организации и заверяются печатью организации здравоохранения "Для рецептов"//
B. Рецепты на получение лекарственных средств бесплатно или на льготных условиях дополнительно подписываются уполномоченным лицом, определенным приказом медицинской организации и заверяются электронно-цифровой подписью уполномоченного лица//
C. Заведующий фельдшерско-акушерским пунктом не заверяет рецепт на получение лекарственных средств бесплатно или на льготной основе своей подписью//
D.Заведующий фельдшерско-акушерским пунктом не заверяет рецепт на получение лекарственных средств бесплатно или на льготной основе печатью фельдшерско-акушерского пункта или электронно-цифровой подписью//
+E.Рецепты на получение лекарственных средств бесплатно или на льготных условиях дополнительно подписываются уполномоченным лицом, определенным приказом медицинской организации и заверяются печатью организации здравоохранения "Для рецептовили заверяются электронно-цифровой подписью уполномоченного лица "
1. Медицинская функция формулярной системы//
А.обеспечение больных качественным лечением//
+В. определение и разработка методов рациональной фармакотерапиизаболеваний//
С. определение клинически и экономически наиболее эффективных ибезопасных лекарственных препаратов//
D. обеспечение контроляправильности использования препаратов ипринятие мер по предотвращению и исправлениюошибок фармакотерапии//
Е. широкое распространениеобъективной медико-фармацевтическойинформации среди всех участников процесса здравоохранения, основанной на принципах доказательной медицины
2. В формулярном списке обязательно используются наименования лекарственных средств://
А. химические//
В. определенные клиническим фармакологом данного учреждения//
С. торговые//
D. принятые в данном лечебном учреждении//
+E. генерические
3. Формулярный список лекарственных средств является://
А. рекомендательным//
В. jпределительным//
С. законодательным//
+D. ограничительным//
Е. расширительным
4.Основные особенности фармакокинетики лекарственных средств у лиц пожилого возраста, кроме одного: //
А. снижение скорости распределения//
В. замедление выведения лекарственных средств//
С. уменьшение связывания лекарственных средств с белками плазмы//
D. замедление метаболизма//
+Е. снижение скорости абсорбции
5.Разработка формулярного списка лекарственных средств включает, кроме одного://
А. утверждение формулярного списка лекарственных средств//
+В. установление поставщиков лекарственных средств//
С. отбор лекарственных средств для формулярного списка//
D. анализ структуры заболеваемости и состояния лекарственного обеспечения//
Е. сформирование перечень лекарственных средств для бесплатной государственной помощи
1.Что такое формулярная система?//
A. Консультативно-совещательный орган, ответственный за лекарственную политику РК//
B. Перечень лекарственных средств с доказанной клинической эффективностью и безопасностью//
+C.Комплекс информационно-методологических мероприятий организации здравоохранения,обеспечивающая рациональное использование лекарственных средств в рамках ГОБМП и современную,качественную медицинскую помощь для граждан РК//
D.это важный фактор, влияющий на выбор лечения//
E. изучает эффекты лекарств на уровне популяций или больших групп людей//
2. Аббревиатура Казахстанского национального лекарственного формуляра//
A. BNF//
B. FNO//
C. PИЛС//
D.FDA//
+E. KNF
3.Выберитепредседателяформулярнойкомиссии//
+A.главныйврач//
B.заведующийхирургией//
C.старшаямедицинская сестра//
D. реаниматолог//
E.заведующаяаптекой
4. На основании рекомендаций FDA (FederalDragAdministration) к какой категорииотносятсяпрепараты железо и трийодтиронин?//
+A. категория A//
B. категория B//
C. категория C//
D. категорияD//
E.категорияX
5. Укажите препараты, относящиеся к категории B, в соответствии с рекомендациямиFDA(FederalDragAdministration)//
А. диклофенак,доксицилин//
B. гентамицин,фторхинолоны//
C. диазепам,канамицин//
D. карбамазепин,стрептомицин//
+E. инсулин,аспирин,метронидазол
Тест с ответами по тему РИЛС

2. Определите членов, входящих в состав формулярной комиссии в медицинской организации://


A.ответственный по антибиотикам, провизор/фармацевтаптеки, главная медицинская сестра, комитет по оценке использования лекарственных средств//
B.администрация, клинический фармаколог, заведующие отделений, сотрудники ВУЗ(кафедры)//
C.главная медицинская сестра, комитет по оценке использования лекарственных средств//
+D.администрация, клинический фармаколог, заведующие отделениями, сотрудники ВУЗ, ответственный по антибиотикам, провизор/фармацевтаптеки, ответственныйпо оценке использования лекарственных средств//
E.клинический фармаколог, заведующие отделений, сотрудники ВУЗ(кафедры)

3. Для какого из формулярных компонентов характерны следующие определения: «перечень лекарственных средст, наличие которых обязательно в достаточных количествах, сформированный для оказания гарантированного объема бесплатной медицинской помощи с учетом профилчя организации здравоохранения в порядке, определяемом уполномоченным органом»://


A.функции формулярной комиссии//
+B.лекарственный формуляр//
C.формулярной справочник//
D.стандарты фармакотерапии и клинические протоколы ведения больных//
E.системы контроля, оценки и мониторинга использования лекарств и нежелательных
4.Укажите критерии рационального выбора лекарственного средства по рекомендации ВОЗ://
A.Эффективность, безопасность//
B.Приемлемость, стоимость//
+C.Эффективность, безопасность, приемлемость, стоимость//
D.Безопасность, приемлемость, стоимость//
E.Эффективность, безопасность, приемлемость
5. Определите категорию Cпо тератогенности ЛС с назначением FDA (FederalDrugAdministration)://
A.лекарственные средства, входящие в эту группу, безвредны для плода на протяжении всей беременности (препараты железа, трийодтиронин)//
B.применение препаратов сопряжено с определённым риском для плода, однако польза от их применения превосходит возможное побочное действие (диазепам, доксициклин, канамицин, диклофенак)//
C.доказано тератогенное действие препаратов этой группы, их приём противопоказан до и во время беременности (изотретиноин, карбамазепин, стрептомицин)//
+D.в исследованиях на животных выявлено тератогенное или эмбриотоксическое действие препарата, контролируемых испытаний не проводилось, либо действие препарата не изучено (изониазид, фторхинолоны, гентамицин, противопаркинсонические препараты, антидепрессанты)//
E.экспериментальные исследования не выявили тератогенного действия, либо наблюдаемые у животных осложнения не обнаружены у детей, матери которых принимали лекарственные препараты, входящие в эту группу (инсулин, аспирин, метронидазол)

:
Показание для назначения off-lableдля применения препарата Метформин://


Макулярная дегенерация//
Прерывание беременности//
Легочная артериальная гипертензия//
+Для снижения веса//
Для улучшения отхождения мокроты

***
Укажите какого типа НПР вызывает препарат Талидомид, который широко применялся в 1960 годах для седации://


Канцерогенное, пороки развития конечностей//
Дозозависимое, врожденный порок сердца//
+Тератогенное, пороки развития конечностей//
Ульцерогенное, триада Грега//
Тератогенное, врожденный порок сердца

***
В каком возрасте педиатрических пациентов можно назначить взрослую дозировку ЛС://


5 лет//
6 -7 лет//
До 12 лет//
+Старше 12 лет//
16 – 18 лет

***
К лекарственным средствам рецептурного отпуска относятся://


ЛС, содержащие наркотические средства, психотропные вещества и прекурсоры, подлежащие контролю в РК//
ЛС, предназначенные для парентерального введения//
Диагностические лекарственные средства, применяемые только в условиях стационара//
ЛС, назначение и дальнейшее употребление которых необходимо проводить при постоянном контроле врача//
+Все перечисленное

***
Сколько категорий лекарственных препаратов по степени риска для плода выделяют на основании рекомендации FDA://


7//
+5//
3//
4//
6

***
Какое количество наименований лекарственныхсредств указывается на одном рецептурном бланке на получение бесплатно или на льготной основепроизводных 8-оксихинолина,гормональных и анаболических стероидов, клонидина, кодеина, тропикамида, циклопентолата, трамадола, буторфанола://


+одно//
не более двух//
не более трех//
не более четырех//
не более пяти

***

Какое количество наименований лекарственныхсредств указывается на одном рецептурном бланке на получение бесплатно или на льготной основеэналаприла, бисопролола, нифедипина, сальбутамола, амоксициллина://
одно//
+не более двух//
не более трех//
не более четырех//
не более пяти

***
Выберите правильный ответ. При выписывании лекарственного средства в дозе, превышающей высшую разовую, врач в рецепте обозначает дозу лекарственного средства прописью и …://


вопросительным знаком//
+восклицательным знаком//
опознавательным знаком «Стоп – медикамент высокого риска»//
опознавательным знаком «Стоп – медикамент схожий по названию»//
опознавательным знаком «Стоп – высшая разовая доза»

***
Как правильно выписываются рецепты://


на латинском языке под международным непатентованным наименованием лекарственных средств//
в случае индивидуальной непереносимости пациента, многокомпонентного состава лекарственного препарата выписываются под торговым наименованием//
рецепты выписываются разборчиво с обязательным заполнением всех предусмотренных в бланке граф//
+все перечисленное верно//
способ применения прописывается в соответствующей графе с указанием дозы, частоты и длительности приема

***
Выберите правильное утверждение://


Рецепты на получение лекарственных средств бесплатно или на льготных условиях дополнительно подписываются уполномоченным лицом, определенным приказом медицинской организации и заверяются печатью организации здравоохранения "Для рецептов"//
Рецепты на получение лекарственных средств бесплатно или на льготных условиях дополнительно подписываются уполномоченным лицом, определенным приказом медицинской организации и заверяются электронно-цифровой подписью уполномоченного лица//
Заведующий фельдшерско-акушерским пунктом не заверяет рецепт на получение лекарственных средств бесплатно или на льготной основе своей подписью//
Заведующий фельдшерско-акушерским пунктом не заверяет рецепт на получение лекарственных средств бесплатно или на льготной основе печатью фельдшерско-акушерского пункта или электронно-цифровой подписью//
+Рецепты на получение лекарственных средств бесплатно или на льготных условиях дополнительно подписываются уполномоченным лицом, определенным приказом медицинской организации и заверяются печатью организации здравоохранения "Для рецептовили заверяются электронно-цифровой подписью уполномоченного лица "

***
Определите членов, входящих в состав формулярной комиссии в медицинской организации://


ответственный по антибиотикам, провизор/фармацевт аптеки, главная медицинская сестра, комитет по оценке использования лекарственных средств//
администрация, клинический фармаколог, заведующие отделений, сотрудники ВУЗ (кафедры)//
главная медицинская сестра, комитет по оценке использования лекарственных средств//
+главный врач, заместитель главного врача по лечебной работе, клинический фармаколог, заведующие отделениями, сотрудники ВУЗ, ответственный по антибиотикам, провизор/фармацевт аптеки, ответственный по оценке использования лекарственных средств//
клинический фармаколог, заведующие отделений, сотрудники ВУЗ (кафедры)

***

Что такое формулярная система://
Консультативно-совещательный орган, ответственный за лекарственную политику РК//
Перечень лекарственных средств с доказанной клинической эффективностью и безопасностью//
+Комплекс информационно-методологических мероприятий организации здравоохранения, обеспечивающая рациональное использование лекарственных средств в рамках ГОБМП и современную, качественную медицинскую помощь для граждан РК//
это важный фактор, влияющий на выбор лечения//
изучает эффекты лекарств на уровне популяций или больших групп людей

***
Аббревиатура Казахстанского национального лекарственного формуляра://


BNF//
FNO//
PИЛС//
FDA//
+KNF

***

Ктоявляетсяпредседателемформулярной комиссии://
+главныйврачорганизации//
заведующийхирургией//
старшаямедицинская сестра//
реаниматолог//
заведующаяаптекой

***
На основании рекомендаций FDA (FederalDrugAdministration) к какой категории относятся препараты железо и трийодтиронин://


+категория A//
категория B//
категория C//
категория D//
категория X

***

Не является медикаментозной ошибкой://
Врач выписывает назначения препарата без указания способа введения//
Фармацевт выдает неправильный препарат вследствие неверного прочтения выписанного врачом рецепта//
Лица, ухаживающие за пациентом, дают пациенту препарат в неверной дозе//
Использование просроченных лекарственных препаратов//
+Проведение назначений согласно правилам пяти правильных «П»

***
Компоненты формулярной системы://


Формулярная комиссия//
Лекарственный формуляр//
Формулярный справочник//
+Все ответы верны//
Стандарты фармакотерапии и клинические протоколы диагностики и лечения
В разделе инструкции «Фармакологическое действие» описывается://
+механизм действия, локализация действия, виды действия//
лекарственные взаимодействия//
генетические аспекты действия//
процессы всасывания, распределения, метаболизма, выведения//
механизм действия

***
Терапевтическая широта лекарственного средства определяется://


соотношением пороговой и максимальной токсической доз//
соотношением минимальной эффективной и средней терапевтической доз//
+соотношением минимальной эффективной и минимальной токсической дозы//
соотношением средней терапевтической и максимальной безопасной доз//
соотношением пороговой и смертельной дозы

***
Клинический протокол диагностики и лечения – это://


+клиническое руководство - положение, систематически разрабатываемое для помощи практикующему врачу или пациенту при принятии решения относительно оказания соответствующей медицинской помощи при определенных клинических обстоятельствах//
продукт адаптации клинических руководств, с расширением оперативных деталей для применения в местных условиях//
положение, которое выделяет цель оказания помощи в форме критериев, определяющих необходимые ресурсы, этапы, прогнозируемые результаты и ориентирован на заболевание//
положение, которое разрабатывает список основных лекарственных средств//
ограниченные списки ЛС, упорядочивающие использование препаратов

***
При простом слепом клиническом исследовании://


и врач, и пациент знают, какая терапия назначена//
+пациент, в отличие от исследователя, не знает, какое лечение ему назначено//
ни врач, ни пациент не знают назначенной терапии//
ни врач, ни пациент, ни исследователь не знают назначенной терапии//
врач, пациент, исследователь знают назначенную терапию

***
Полифармация (полипрагмазия) – это://


одновременное использование слишком большого числа ЛС у одного больного при лечении основного заболевания//
одновременное использование слишком большого числа ЛС (зачастую необоснованно) у одного больного при лечении сопутствующего заболевания//
одновременное использование слишком большого числа ЛС (зачастую необоснованно) у группы больных//
одновременное использование слишком большого числа ЛС (зачастую необоснованно) у одного больного с учетом физиологического состояния (беременность, возраст)//
+одновременное использование слишком большого числа ЛС (зачастую необоснованно) у одного больного

***
Приказ Министра здравоохранения и социального развития Республики Казахстан от 22 мая 2015 года № 373 "Об утверждении Правил выписывания, учета и хранения рецептов". Выберите неправильное утверждение://


Рецепты выписываются на латинском языке под международным непатентованным наименованием лекарственных средств//
В случае индивидуальной непереносимости пациента рецепт выписывается под торговым наименованием//
Рецепты выписываются разборчиво, способ применения прописывается с указанием дозы, частоты и длительности приема//
Не допускается ограничиваться общими указаниями ("Известно", "Внутреннее", "Наружное")//
+Исправления в рецепте допускаются

***
В инструкции к применению ЛС в разделе «Фармакокинетика» описывается://


Лекарственные взаимодействия//
+Процессы всасывания, распределения, биотрансформации и выведения ЛС//
Механизм действия, локализация действия//
Фармакогенетика//
Показания к применению

***
Официальным источником информации о лекарственных средствах является://


+Инструкции к применению ЛС на сайте республиканского центра развития здравоохранения (РЦРЗ)//
Справочник лекарственных средств под редакцией М.Д.Машковского//
Справочник ВИДАЛЬ//
Инструкции FDA//
Клинический протокол диагностики и лечения РК

***
Назначение лекарственного средства «off-label» это://


Применение ЛС для повышения эффективности другого ЛС//
+Применение ЛС не по показаниям, регламентированным в инструкции к применению ЛС//
Применение ЛС в качестве прикрытия//
Применение ЛС не по стандартам//
Применение ЛС для повышения безопасности

***
Информация из раздела инструкции «Побочное действие» используется для://


Разработки программы контроля за эффективностью//
Выбора индивидуального режима дозирования//
Коррекции режима дозирования при наличии у пациента печеночной патологии//
+Разработки программы контроля за безопасностью//
Коррекции режима дозирования при наличии у пациента печеночной патологии

***
Медикаментозные ошибки ежегодно наносят ущерб мировой экономике в размере://


$ 58 млрд//
+$ 42 млрд//
$ 33 млрд//
$ 65 млрд//
$ 48 млрд

***
Основным критерием для выбора лекарственных средств://


+безопасность//
низкая стоимость//
высокая стоимоcть//
форма выпуска//
комплайнс

***
Рецепты на получение лекарственных средств, в том числе отпускаемых бесплатно и на льготных условиях, действительны в течение://


2 недель//
3 месяцев//
+1 месяца//
4 недель//
1 недели

***
Рациональная фармакотерапия должна учитывать и оптимально соотносить профиль пациента://


профессии пациента//
национальности пациента//
+возрасту пациента//
семейному статусу//
вредным привычкам

***

У какой категории пациентов чаще встречаются медикаментозныеошибки://
лабильные пациенты//
пациенты амбулаторного звена//
пациенты репродуктивного возраста//
+дети и пожилые//
медицинские работники

***
Какая часть всех расходов на здравоохранение уходит на исправление медикаментозных ошибок://


+1%//
5%//
20%//
10%//
40%

Дайте определение Формулярной комиссии://


+Консультативно-совещательный орган, ответственный за лекарственную политику в медицинской организации//
Утвержденный руководителем организации здравоохранения и согласованный в порядке, определяемом уполномоченным органом в области здравоохранения, перечень лекарственных средств (ЛС), сформированный для оказания гарантированного объема бесплатной медицинской помощи с учетом профиля организации здравоохранения, наличие которых обязательно в достаточных количествах//
Утвержденный, ограниченный список ЛС, отобранных уполномоченными экспертами по критериям эффективности, безопасности и результатам фармакоэкономического анализа для обеспечения рационального лечения//
Комплекс информационно-методологических мероприятии организации здравоохранения, обеспечивающая рациональное использование лекарственных средств в рамках ГОБМП и современную, качественную медицинскую помощь для граждан своей страны//
Все ответы верны

***
Функции формулярной комиссии://


Внедрение принципов рациональной фармакотерапии//
Оценка и отбор ЛС для формуляра и обеспечение его регулярного пересмотра//
+Все ответы верны//
Рассмотрение случаев развития побочных реакций на применение ЛС и принятие соответствующих мер//
Решение проблем, связанных с ошибками при проведении лечения

***

К нежелательным реакциям типа В не относится://
анафилактический шок на введение новокаина//
агранулоцитоз при приеме анальгина//
гемолитический криз при приеме сульфаниламидов//
+ острая язва желудка при приеме пироксикама//
лихородка при приеме ибупрофена

***
У больного в анамнезе – анафилактической шок на пенициллин. Применение какого антибактериального препарата ему противопоказано://


+Цефуроксим//
Тетрациклин//
Ципрофлоксацин//
Кларитромицин//
Ко-тримоксазол

***
К нежелательным реакциям типа С относится://


лекарственная зависимость//
+ эмбриотоксичность//
Ксеростомия//
Крапивница//
медикаментозная нейтропения

***
Укажите правильное утверждение://


у больных с аллергической реакций в анамнезе показано проведение аллергических кожных проб перед началом терапии антибиотиками//
пациенту с аллергией на новокаин противопоказано введение лидокаина//
+препараты с узким терапевтическим диапазоном чаще вызывают нежелательные реакции типа А//
тяжесть аллергической реакции зависит от дозы препарата//
плод наиболее чувствителен к действию лекарственных препаратов в III триместре беременности

***
Фотосенсибилизации возможна при приеме://


+фторхинолонов//
аминогликозидов//
фуросемида//
амоксицлиина//
метамизола

***
Принимая ЛС, их следует запивать://


молоком//
фруктовыми соками//
+кипяченой водой//
минеральной водой//
всем вышеуказанным

***
Всасывание питательных веществ из пищи уменьшается под действием://


атропина//
+ слабительных средств//
витамина С//
анальгина//
нифедипина

***

Антибактериальная тактика у больного с абсцедирующей пневмонией://
Аминопеницилины в больших дозах в/м//
Аминогликозиды в/м//
Фторхинолоны внутрь//
Метронидазол и макролиды в/в//
+ Аминогликозиды и цефалоспорины II-III поколения в/в.

***
Серьезное осложнение терапии хронической сердечной недостаточности ингибиторами АПФ://


Сухой кашель//
+ Постуральная гипотензия//
Ангионевротический отек//
Гиперкалиемия//
Аллергия

***
Потенциальное осложнение терапии петлевыми диуретиками при хронической сердечной недостаточности://


+ Гипогликемия//
Гипермагнелия//
Гипокалиемия//
Гипонатриемия//
Гипоуриремия

***
Какие метаболические нарушения наблюдаются при приеме тиазидных диуретиков://


+Гипокалиемия//
Гипогликемия//
Гипоурикемия//
Гипернатриемия//
Гипермагниемия

***
При лечении острого инфаркта миокарда прямыми антикоагулянтами (гепарином) обязательно надо следить за://


временем свертывания крови//
протромбиновым индексом//
временем рекальцификации плазмы//
уровнем фибриноген//
+АЧТВ

***
Абсолютным противопоказанием к назначению каптоприла является://


+Двухсторонний стеноз почечных артерий//
Повышение уровня креатинина в крови выше 2 мг%//
Умеренное повышение АД//
Снижение уровня ренина в плазме крови//
Сахарный диабет

***
Какой из указанных ниже препаратов вызывает индукцию микросомальных ферментов печени://


+Фенобарбитал//
Морфин//
Аминазин//
Фторотан//
Сибазон

***
Прием следующих препаратов может вызвать язвенное поражение слизистой оболочки желудочно-кишечного тракта://


+ преднизолон//
метронидазол//
вентер//
кардиомагнил//
сукральфат

***
При открытом клиническом исследовании://


+врач и пациент знают, какая терапия назначена//
пациент, в отличие от исследователя, не знает, какое лечение ему назначено//
ни врач, ни пациент не знают назначенной терапии//
ни врач, ни пациент, ни исследователь не знают назначенной терапии//
врач, пациент, исследователь знают назначенную терапию

***
При двойном слепом клиническом исследовании://


и врач, и пациент знают, какая терапия назначена//
пациент, в отличие от исследователя, не знает, какое лечение ему назначено//
+ни врач, ни пациент не знают назначенной терапии//
ни врач, ни пациент, ни исследователь не знают назначенной терапии//
врач, пациент, исследователь знают назначенную терапию

***
Каким термином обозначают необычные реакции на лекарственные средства, связанные с генетически обусловленными энзимопатиями://


сенсибилизация//
тахифилаксия//
+идиосинкразия//
абстиненция//
привыкание

***
Практически за два периода полувыведение из организма выводится://


25% препарата//
50% препарата//
+75% препарата//
90% препарата//
100% препарата

***
Какое антибактериальное средство противопоказано при беременности и лактации://


бензилпенициллин//
+тетрациклин//
амоксициллин//
линкомицин//
цефазолин

***
Какие антибактериальные средства противопоказаны у детей до 8 лет://


защищенные аминопенициллины//
+фторхинолоны//
цефалоспорины//
аминогликозиды//
линкосамиды

***
Фармакокинетическая кривая - это://


зависимость концентрации препарата в крови от дозы//
+ зависимость концентрации препарата в крови от времени//
зависимость концентрации препарата в крови от кратности введения//
зависимость скорости выведения препарата от времени//
зависимость метаболизма лекарства от времени

***
Побочные эффекты, выраженность которых не зависит от дозы://


фармакодинамические (прямого действия)//
повреждающие (токсические)//
+аллергические и иммунные//
фармакогенетические//
фармакокинетические

***
ЛС, прекращение приема которых может вызвать синдром абстиненции://


Клонидин//
Нитроглицерин//
Золпидем//
+Морфин//
Пропранолол

***
Нефротоксичность является характерным ПЭ для следующих ЛС://


+аминогликозидов//
антикоагулянтов//
симпатолитиков//
бета-блокаторов//
ингибиторыАПФ

***
Выберите группу препаратов с узким терапевтическим индексом://


бета-блокаторы//
пенициллины//
+сердечные гликозиды//
ингибиторы АПФ//
петлевые диуретики

***
Проведение лекарственного мониторинга требуется при лечении следующей группой лекарственных препаратов://


+противосудорожными//
бета-блокаторами//
антагонистами кальция//
глюкокортикоидами//
М-холинолитиками

***
Укажите методы контроля за эффективностью применения диуретиков при отечном синдроме://


определение концентрации в плазме крови Na+ и K+//
+измерение суточного диуреза и количества выпитой жидкости//
измерение суточного калийуреза//
определение КЩР//
контроль ЭКГ не реже 1 раза в неделю

***
Ранним побочным эффектом кортикостероидов является://


развитие катаракты//
+стероидная язва//
остеопороз//
кушингоидный синдром//
стероидный диабет

***
Ванкомицин является высокоактивным антибиотиком против://


хламидий//
синегнойной палочки//
+стафилококков//
гемофильной палочки//
анаэробов

***
Наиболее безопасный жаропонижающий и анальгетический препарат у детей://


+Парацетамол//
Аспирин//
Амидопирин//
Анальгин//
Индометацин

***
Побочные эффекты эритромицина включают://


+холестатическую желтуху//
агранулоцитоз и тромбоцитопению//
стоматиты//
кожные проявления, подобные герпесу//
вестибулярные нарушения

***
Результатом высокой степени связывания препарата с белками плазмы является://


уменьшение Т 1/2//
повышение концентрации свободной фракции препарата//
+ снижение концентрации свободной фракции препарата//
лучшая эффективность препарата//
все перечисленное

***

Какие препараты больше подвергаются метаболизму в печени://
+ Липофильные//
Гидрофильные//
Липофобные//
имеющие кислую реакцию//
имеющие щелочную реакцию

***
Что такое равновесная концентрация://


+ состояние, когда количество абсорбированного препарата равно количеству выводимого//
максимальная концентрация после введения препарата//
концентрация перед очередным введением препарата//
средняя концентрация после введения препарата//
правильных ответов нет

***
К побочным эффектам, требующим отмены ингаляционных кортикостероидов, относится://


развитие кандидоза полости рта//
дисфония//
спорадический кашель после ингаляции//
+ возникновение бронхоспазма после ингаляции//
сухость во рту

***
Выберите препарат для лечения сепсиса, вызванного MRSA://


ампициллин//
цефазолин//
тиенам//
азитромицин//
+ линезолид

***
Выберите антимикробный препарат с узким терапевтическим коридором://


мидекамицин//
клиндамицин//
+ амикацин//
амоксициллин//
цефтриаксон

***
Какой антимикробный препарат противопоказан детям://


пенициллин//
цефтриаксон//
азитромицин//
+ ципрофлоксацин//
эритромицин

***
Укажите, что не относится к осложнениям инсулинотерапии://


Феномен Самоджи//
Инсулинорезистентность//
Нарушение зрения//
Инсулиновые отеки//
+ Гиперурикемия

***
Низкая биодоступность лекарственного средства может быть связана с://


Усиленной секрецией ЛС//
Хорошим всасыванием ЛС//
Усиленной клубочковой фильтрацией ЛС//
Большой степенью связи ЛС с белками плазмы крови//
+ Эффектом первого прохождения ЛС через печень

***
Биоэквивалентность – это://


Сравнительная эффективность оригинального и генерического препаратов//
Сравнитеьная фармакодинамика оригинального и генерического препаратов//
Сравнительная безопасность оригинального и генерического препаратов//
+ Сравнитеьная фармакокинетика (биодоступность) оригинального и генерического препаратов//
Сравнитеьная фармакогенетика оригинального и генерического препаратов

***
Среди осложнениий терапии ингибиторами 3-гидрокси-3-метилглутарил коэнзима А редуктазы (статинами) наиболее опасным является://


повышение уровня печеночных ферментов//
+ рабдомиолиз//
миопатия//
повышение уровня креатинфосфокиназы//
диспепсические явления

***
Врач клинический фармаколог имеет следующие права://


вносить предложения администрации по вопросам улучшения деятельности подразделения, организации и условий труда//
осуществлять закупку медикаментозных препаратов//
+ консультировать пациентов в сложных случаях фармакотерапии//
проверять знания заведующих клиническими отделениями//
проводить электрокардиографию пациентам с сердечной патологией

***
При каком способе введения лекарств биодоступность будет максимальной://


пероральном//
сублингвальном//
внутримышечном//
+ внутривенном//
ректальном

***
При сублингвальном введениисоздается более высокая концентрация препарата в плазме, чем пероральном потому, что://


+ лекарство не подвергается пресистемному метаболизму//
лекарство не связывается с белками плазмы//
лекарства не связываются с тканями//
увеличивается гидрофильность препарата//
увеличивается липофильность препарата

***
Казахстанский Национальный Лекарственный Формуляр (КНФ) – это://


перечень ЛС с доказанной клинической эффективностью и безопасностью, содержит информацию о ЛС и ценах//
является обязательной основой для разработки лекарственных формуляров организаций здравоохранения//
надежный информационный ресурс по выбору и применению ЛС для врачей, фармацевтов, организаторов здравоохранения, медсестер, слушателей медицинских учебных заведений//
составлен специальной редакторской группой, под руководством МЗ РК при консультировании экспертами Формулярной комиссии и международной группой экспертов//
+ все перечисленное верно

***
Мониторинг безопасности ЛС и медицинских изделий (МИ) осуществляет://


Биоэтический комитет ВУЗов//
+ Национальный центр экспертизы ЛС и МИ РК//
Формулярная комиссия лечебных организаций//
Центр рационального использования ЛС//
Казахстанский национальный формуляр

***
Какой вид карты сообщения о НПР необходимо заполнить на сайте НЦЭЛС (www.ndda.kz) при поступлении пациента в стационар с развившейся до госпитализации нежелательной побочной реакцией://


начальный//
+последующий//
сопутствующий//
конкурирующий//
предшествующий

***
Какой вид карты сообщения о НПР необходимо заполнить на сайте НЦЭЛС (www.ndda.kz) при возникновении нежелательной побочной реакции во время госпитализации пациента://


+начальный//
последующий//
сопутствующий//
конкурирующий//
предшествующий

***
Сколько вопросов содержит шкала Наранжо (для выявления причинно-следственной связи между возникновением НПР и приемом ЛП)://


2//
5//
7//
12//
+10

***
56-летний мужчина с возвратной желудочковой аритмией 5 месяцев назад самостоятельно прекратил прием антиаритмического препарата. В данный момент у больного усилилась одышка, кашель, отмечено повышение температуры тела. СОЭ повышена. На обзорной R-грамме грудной клетки – интерстициальная пневмония. Функциональные легочные тесты: повышено содержание СО. Отметьте препарат, вызывающий подобные изменения://


+ Амиодарон//
Пропранолол//
Прокамиамид//
Хинидин//
Верапамил

***
К какому типу НПР относятся поражения легких, спровоцированные амиодароном://


А тип//
В тип//
С тип//
D тип//
+А тип – при развитии острого респираторного дистресс синдрома, С тип – при развитии интерстициальной пневмонии

***
Б.34 года получает антибактериальный препарат по поводу инфекционного процесса. При внутривенной инфузии препарата отмечается реакция в виде выраженного покраснения кожных покровов верхней половины туловища, лица, шеи; симптомы значительно уменьшаются при снижении скорости инфузии. На какой препарат отмечается такая реакция://


Цефтриаксон//
+ Ванкомицин//
Амикацин//
Ципрофлоксацин//
Азлоциллин
С какой периодичностью проводится внешняя оценка рационального использования лекарственных средств в организациях здравоохранения://

1 раз в год//


2 раза в год//
каждый квартал//
1 раз в 5 лет//
+1 раз в 2 года

***

Б.45лет на фоне применения антибиотика цефтриаксона в течение 10 дней развилась картина псевдомембранозного колита. Какой первый шаг алгоритма врачебной помощи://

отмена антибиотика, назначение флуконазола//


назначение цефепима//
назначение сульфаниламидных препаратов//
+отмена цефтриаксона, назначение ванкомицина или метронидазола//
назначение амикацина

Медицинская функция формулярной системы://


обеспечение больных качественным лечением//
+определение и разработка методов рациональной фармакотерапии заболеваний//
определение клинически и экономически наиболее эффективных и безопасных лекарственных препаратов//
обеспечение контроля правильности использования препаратов и принятие мер по предотвращению и исправлению ошибок фармакотерапии//
широкое распространение объективной медико-фармацевтической информации среди всех участников процесса здравоохранения, основанной на принципах доказательной медицины

***
Формулярный список лекарственных средств является://


рекомендательным//
определительным//
законодательным//
+ограничительным//
расширительным

***
Основные особенности фармакокинетики лекарственных средств у лиц пожилого возраста, кроме одного://


снижение скорости распределения//
замедление выведения лекарственных средств//
уменьшение связывания лекарственных средств с белками плазмы//
замедление метаболизма//
+снижение скорости абсорбции

***
Основные критерии для рационального выбора ЛС://


Эффективность//
Безопасность//
Приемлемость//
Стоимость//
+Все ответы верны

***
Выберите один препарат, который не относится к ЛС рецептурного отпуска://


Наркотические средства//
+Отхаркивающие средства//
ЛС, предназначенные для парентерального введения//
Диагностические ЛС//
Психотропные вещества
Дайте определение «комплайнс»://

увеличение работоспособности в результате переоценки личностных достижений//


эмоциональное истощение, деперсонализация и снижения личностных достижений специалистов//
+достижение приверженности лечению между врачом и пациентом//
использование всех возможных методов лечения для уменьшения симптомов заболевания в кратчайшие сроки//
назначение 5 и более лекарственных средств одному пациенту

***
Врачи какого возраста чаще всего подвержены синдрому эмоционального выгорания://


30-50 лет//


+20-40 лет//
40-50 лет//
50-60 лет//
60 и старше

***
Каким приказом МЗ РК регламентируются правила проведения оценки рационального использования лекарственных средств://


+№67 от 06.05.2019//


№62 от 04.05.2019//
№12 от 08.01.2013//
№461 от 27.07.2018//
№114 от 26.08.2019

***
В каком году и городе было сформировано определение рациональному использованию лекарственных препаратов://


2001г Лондон//


1967г Вашингтон//
1996г Москва//
+1985г Найроби//
2012г Сидней

1.Медикаментозные ошибки ежегодно наносят ущерб мировой экономике в размере://


$ 58 млрд//
$ 42 млрд//+
$ 33 млрд//
$ 65 млрд//
$ 48 млрд
2.Основным критерием для выбора лекарственных средств://
безопасность//+
низкая стоимость//
высокая стоимсоть//
форма выпуска//
комплайнс
3.Рецепты на получение лекарственных средств, в том числе отпускаемых бесплатно и на льготных условиях, действительны в течение:
2 недель//
3 месяцев//
1 месяца//+
4 недель//
1 недели
4.Рациональная фармакотерапия должна учитывать и оптимально соотносить профиль пациента://
профессии пациента//
национальности пациента//
возрасту пациента//+
семейному статусу//
вредным привычкам
Тест по теме РИЛС
1. Не являетсямедикаментозной ошибкой://
А. Врач выписывает назначенияпрепарата без указания способа введения//
В. Фармацевт выдает неправильный препарат вследствие неверного прочтения выписанного врачом рецепта//
С. Лица, ухаживающие за пациентом, дают пациенту препарат в неверной дозе//
Д. Использование просроченных лекарственных препаратов//
+Е. Проведение назначений согласно правилам пяти правильных «П»

2. Компоненты формулярной системы://


А. Формулярная комиссия//
В. Лекарственный формуляр//
С. Формулярный справочник//
Д. Стандарты фармакотерапии и клинические протоколы диагностики и лечения//
+Е. Все ответы верны

3. Дайте определение Формулярной комиссии//


+А. Консультативно-совещательный орган, ответственный за лекарственную политику в медицинской организации//
В. Утвержденный руководителем организации здравоохранения и согласованный в порядке, определяемом уполномоченным органом в области здравоохранения, перечень лекарственных средств (ЛС), сформированный для оказания гарантированного объема бесплатной медицинской помощи с учетом профиля организации здравоохранения, наличие которых обязательно в достаточных количествах//
С. Утвержденный, ограниченный список ЛС, отобранных уполномоченными экспертами по критериям эффективности, безопасности и результатам фармакоэкономического анализа для обеспечения рационального лечения//
Д. Комплекс информационно-методологических мероприятии организации здравоохранения, обеспечивающая рациональное использование лекарственных средств в рамках ГОБМП и современную, качественную медицинскую помощь для граждан своей страны//
Е. Все ответы верны


4. Не входит в состав формулярной комиссии://
А. Клинический фармаколог//
+В. Экономист //
С. Заведующий отделением//
Д. Администрация//
Е. Провизор/фармацевт аптеки


5. Функции формулярной комиссии://
А. Внедрение принципов рациональной фармакотерапии//
В. Оценка и отбор ЛС для формуляра и обеспечение его регулярного пересмотра//
+С. Все ответы верны//
Д. Рассмотрение случаев развития побочных реакций на применение ЛС и принятие соответствующих мер//
Е. Решение проблем, связанных с ошибками при проведении лечения
Тест по РИЛС
1.Основные критерии для рационального выбора ЛС?//
A.Эффективность//
B.Безопасность//
C.Приемлемость//
D.Стоимость//
+E.Все ответы верны
2.Выберите один препарат, который не относится к ЛС рецептурного отпуска://
A.Наркотические средства//
+B.Отхаркивающие средства//
C.ЛС, предназначенные для парентерального введения//
D.Диагностические ЛС//
E.Психотропные вещества
3.Показание для назначения off-lable (использование ЛС по показаниям, не упомянутым в инструкции) применения препарата Метформин://
A.Макулярная дегенерация//
B.Прерывание беременности//
C.Легочная артериальная гипертензия//
+D.Для снижения веса//
E.Для улучшения отхождения мокроты
4.Укажите какого типа НПР вызывает препарат Талидомид, который широко применялся в 1960 годах для седации://
A.Канцерогенное, пороки развития конечностей//
B.Дозозависимое, врожденный порок сердца//
+C.Тератогенное, пороки развития конечностей//
D.Ульцерогенное, триада Грега//
E.Тератогенное, врожденный порок сердца
5.В каком возрасте педиатрических пациентов можно назначить взрослую дозировку ЛС?//
A.5 лет//
B.6 -7 лет//
C.До 12 лет//
+D.Старше 12 лет//
E.16 – 18 лет
У какой категории пациентов чаще встречаются медикаментозным ошибки?//
А. лабильные пациенты//
B. пациенты амбулаторного звена//
C. пациенты репродуктивного возраста//
+D. дети и пожилые//
Е. медицинские работники

Какая часть всех расходов на здравоохранение уходит на исправление медикаментозных ошибок?//


+А. 1%//
B. 5%//
C.20%//
D.10%//
Е.40%

Дайте определение «комплайнс»?//


А. увеличение работоспособности в результате переоценки личностных достижений//
B. эмоциональное истощение, деперсонализация и снижения личностных достижений специалистов//
+C. достижение приверженности лечению между врачом и пациентом//
D. использование всех возможных методов лечения для уменьшения симптомов заболевания в кратчайшие сроки//
Е. назначение 5 и более лекарственных средств одному пациенту

Врачи какого возраста чаще всего подвержены синдрому эмоционального выгорания?//


А. врачи не подвержены этому синдрому//
+B. 20-40 лет//
C. 40-50 лет//
D. 50-60 лет//
E. 60 и старше
Выберите определениеполипрагмазии?//
генетически обусловленная реакция, возникающая у некоторых людей в ответ на определённые неспецифические раздражители//
одновременное применение одним пациентом слишком многих лекарственных средств//
резистентность микроорганизма к определенным лекарственным средствам//
частое применение лекарственных средств, относящихся к одной группе//
использования нескольких препаратов, имеющих один и тот же механизм действия
2. Каким приказом МЗ РК регламентируются правила проведения оценки рационального использования лекарственных средств?//
+А. №67 от 06.05.2019//
B. №62 от 04.05.2019//
C. №12 от 08.01.2013//
D. №461 от 27.07.2018//
E.№114 от 26.08.2019

3. Сколькоуровней в системеформулярнойкомиссии?//


А. 2//
B. 4//
C. 6//
+D. 3//
E. 5

4. В каком году и городе было сформировано определение рациональному использованию лекарственных препаратов?//


А. 2001г Лондон//
B. 1967г Вашингтон//
C. 1996г Москва//
+D. 1985г Найроби//
E. 2012г Сидней

5. С какой периодичностью проводится внешняя оценка рационального использования лекарственных средств в организациях здравоохранения?//


А. 1 раз в год//
B. 2 раза в год//
C. каждый квартал//
D. 1 раз в 5 лет//
+E. 1 раз в 2 года
Экскреция ЛС или его метаболитов из организма://
Абсорбция//
Распределение//
Биотрансформация//
+Элиминация//
Метаболизм

***

Доля ЛС, достигающая системного кровотока из места введения://
Клиренс//
+Биодоступность//
Период полувыведения//
Объем распределения//
Абсорбция

***
Объем плазмы крови, освобождающийся от ЛС за единицу времени://


+Клиренс//
Биодоступность//
Период полувыведения//
Объем распределения//
Абсорбция

***
Объем жидкости организма, необходимый для равномерного распределения всей введенной дозы в концентрации, аналогичной таковой в плазме крови://


Клиренс//
Биодоступность//
Период полувыведения//
+Объем распределения//
Абсорбция

***
Время, необходимое для снижения концентрации ЛС в крови вдвое://


Клиренс//
Биодоступность//
+Период полувыведения//
Объем распределения//
Абсорбция

***
Промежуток времени, необходимый для достижения равновесного состояния, обычно равен://


Одному периоду полувыведения//
Двум периодам полувыведения//
Трем периодам полувыведения//
Пяти периодам полувыведения//
+Четырем-пяти периодам полувыведения
***
Степень связывания ЛС с белками плазмы играет важную роль при://
+Почечной недостаточности//
Дыхательной недостаточности//
Анемии//
Аллергических заболеваниях//
Анафилактическом шоке

***

О хорошем накоплении ЛС в тканях свидетельствует://
Большая степень связи с белками плазмы//
+Большой объем распределения//
Низкая скорость биотрансформации в печени//
Большое количество нежелательных лекарственных реакций//
Низкая эффективность препарата

***
Баланс между терапевтическим эффектом и неблагоприятными побочными реакциями определяется следующим параметром://


Биоэквивалентностью//
Биодоступностью//
+Терапевтическим диапазоном//
Равновесной концентрацией//
Клиренсом

***
ЛС характеризуется большим периодом полувыведения при://


Малом объеме распределения//
+Большом объеме распределения//
Большой концентрации ЛС в плазме крови//
Низкой концентрации ЛС в органах и тканях//
Большом терапевтическом индексе

***
В разделе инструкции «Фармакологическое действие» описывается://


+A.механизм действия, локализация действия, виды действия//


B.лекарственные взаимодействия//
C.генетические аспекты действия//
D.процессы всасывания, распределения, метаболизма, выведения//
E.механизм действия

***
Терапевтическая широта лекарственного средства определяется://


A.соотношением пороговой и максимальной токсической доз//


B.соотношением минимальной эффективной и средней терапевтической доз//
+C.соотношением минимальной эффективной и минимальной токсической дозы//
D.соотношением средней терапевтической и максимальной безопасной доз//
E.соотношением пороговой и смертельной дозы

***
Клинический протокол диагностики и лечения – это://


+A.клиническое руководство - положение, систематически разрабатываемое для помощи практикующему врачу или пациенту при принятии решения относительно оказания соответствующей медицинской помощи при определенных клинических обстоятельствах//
B.продукт адаптации клинических руководств, с расширением оперативных деталей для применения в местных условиях//
C.положение, которое выделяет цель оказания помощи в форме критериев, определяющих необходимые ресурсы, этапы, прогнозируемые результаты и ориентирован на заболевание//
D.положение, которое разрабатывает список основных лекарственных средств//
ограниченные списки ЛС, упорядочивающие использование препаратов

***
При простом слепом клиническом исследовании://


A.и врач, и пациент знают, какая терапия назначена//
+B.пациент, в отличие от исследователя, не знает, какое лечение ему назначено//
C.ни врач, ни пациент не знают назначенной терапии//
D.ни врач, ни пациент, ни исследователь не знают назначенной терапии//
E.врач, пациент, исследователь знают назначенную терапию

***
Полифармация (полипрагмазия) – это://


A.одновременное использование слишком большого числа ЛС у одного больного при лечении основного заболевания//
B.одновременное использование слишком большого числа ЛС (зачастую необоснованно) у одного больного при лечении сопутствующего заболевания//
C.одновременное использование слишком большого числа ЛС (зачастую необоснованно) у группы больных//
D.одновременное использование слишком большого числа ЛС (зачастую необоснованно) у одного больного с учетом физиологического состояния (беременность, возраст)//
+E.одновременное использование слишком большого числа ЛС (зачастую необоснованно) у одного больного
***
Приказ Министра здравоохранения и социального развития Республики Казахстан от 22 мая 2015 года № 373 "Об утверждении Правил выписывания, учета и хранения рецептов". Выберите неправильное утверждение://
A.Рецепты выписываются на латинском языке под международным непатентованным наименованием лекарственных средств//
B.В случае индивидуальной непереносимости пациента рецепт выписывается под торговым наименованием//
C.Рецепты выписываются разборчиво, способ применения прописывается с указанием дозы, частоты и длительности приема//
D.Не допускается ограничиваться общими указаниями ("Известно", "Внутреннее", "Наружное")//
+E.Исправления в рецепте допускаются

***
В инструкции к применению ЛС вразделе «Фармакокинетика» описывается://


A.Лекарственные взаимодействия//
+B.Процессы всасывания, распределения, биотрансформации и выведения ЛС//
C.Механизм действия, локализация действия//
D.Фармакогенетика//
E.Показания к применению

***
Официальным источником информации о лекарственных средствах является://


+A.Инструкции на сайте республиканского центра развития здравоохранения (РЦРЗ) www.rcrz.kz://
B.Справочник лекарственных средств под редакцией М.Д.Машковского//
C.Справочник ВИДАЛЬ//
D.Инструкции FDA//
E.Клинический протокол диагностики и лечения РК

***
Назначение лекарственного средства «off-label» это://


A.Применение ЛС для повышения эффективности другого ЛС//
+B.Применение ЛС не по показаниям, регламентированным в инструкции к применению ЛС//
C.Применение ЛС в качестве прикрытия//
D.Применение ЛС не по стандартам//
E.Применение ЛС для повышения безопасности

***
Информация из раздела инструкции «Побочное действие» используется для://


A.Разработки программы контроля за эффективностью//
B.Выбора индивидуального режима дозирования//
C.Коррекции режима дозирования при наличии у пациента печеночной патологии//
+D.Разработки программы контроля за безопасностью//
E.Коррекции режима дозирования при наличии у пациента печеночной патологии

***
Медикаментозные ошибки ежегодно наносят ущерб мировой экономике в размере://


A.$ 58 млрд//
+B.$ 42 млрд//+
C.$ 33 млрд//
D.$ 65 млрд//
E.$ 48 млрд
***
Основным критерием для выбора лекарственных средств://
+A.безопасность//
B.низкая стоимость//
C.высокая стоимоcть//
D.форма выпуска//
E.комплайнс
***
Рецепты на получение лекарственных средств, в том числе отпускаемых бесплатно и на льготных условиях, действительны в течение:
A.2 недель//
B.3 месяцев//
+C.1 месяца//
D.4,5 недель//
E.1 недели
***
Рациональная фармакотерапия должна учитывать и оптимально соотносить профиль пациента://
A.профессии пациента//
B.национальности пациента//
+C.возрасту пациента//
D.семейному статусу//
E.вредным привычкам

***

У какой категории пациентов чаще встречаются медикаментозным ошибки?//
А. лабильные пациенты//
B. пациенты амбулаторного звена//
C. пациенты репродуктивного возраста//
+D. дети и пожилые//
Е. медицинские работники

***
Какая часть всех расходов на здравоохранение уходит на исправление медикаментозных ошибок?//


+А. 1%//
B. 5%//
C.20%//
D.10%//
Е.40%

***
Дайте определение «комплайнс»?//


А. увеличение работоспособности в результате переоценки личностных достижений//
B. эмоциональное истощение, деперсонализация и снижения личностных достижений специалистов//
+C. достижение приверженности лечению между врачом и пациентом//
D. использование всех возможных методов лечения для уменьшения симптомов заболевания в кратчайшие сроки//
Е. назначение 5 и более лекарственных средств одному пациенту

***
Врачи какого возраста чаще всего подвержены синдрому эмоционального выгорания?//


А. врачи не подвержены этому синдрому//
+B. 20-40 лет//
C. 40-50 лет//
D. 50-60 лет//
E. 60 и старше

***
Каким приказом МЗ РК регламентируются правила проведения оценки рационального использования лекарственных средств?//


+А. №67 от 06.05.2019//
B. №62 от 04.05.2019//
C. №12 от 08.01.2013//
D. №461 от 27.07.2018//
E. №114 от 26.08.2019

***
Сколькоуровней в системеформулярнойкомиссии?//


А. 2//
B. 4//
C. 6//
+D. 3//
E. 5

***
В каком году и городе было сформировано определение рациональному использованию лекарственных препаратов?//


А. 2001г Лондон//
B. 1967г Вашингтон//
C. 1996г Москва//
+D. 1985г Найроби//
E. 2012г Сидней

***
С какой периодичностью проводится внешняя оценка рационального использования лекарственных средств в организациях здравоохранения?//


А. 1 раз в год//
B. 2 раза в год//
C. каждый квартал//
D. 1 раз в 5 лет//
+E. 1 раз в 2 года

***
Медицинская функция формулярной системы//


А. обеспечение больных качественным лечением//
+В. определение и разработка методов рациональной фармакотерапии заболеваний//
С. определение клинически и экономически наиболее эффективных и безопасных лекарственных препаратов//
D. обеспечение контроля правильности использования препаратов и принятие мер по предотвращению и исправлению ошибок фармакотерапии//
Е. широкое распространение объективной медико-фармацевтической информации среди всех участников процесса здравоохранения, основанной на принципах доказательной медицины

***
В формулярном списке обязательно используются наименования лекарственных средств://


А. химические//
В. определенные клиническим фармакологом данного учреждения//
С. торговые//
D. принятые в данном лечебном учреждении//
+E. генерические
***
Формулярный список лекарственных средств является://
А. рекомендательным//
В. определительным//
С. законодательным//
+D. ограничительным//
Е. расширительным

***
Основные особенности фармакокинетики лекарственных средств у лиц пожилого возраста, кроме одного: //


А. снижение скорости распределения//
В. замедление выведения лекарственных средств//
С. уменьшение связывания лекарственных средств с белками плазмы//
D. замедление метаболизма//
+Е. снижение скорости абсорбции

***
Разработка формулярного списка лекарственных средств включает, кроме одного://


А. утверждение формулярного списка лекарственных средств//
+В. установление поставщиков лекарственных средств//
С. отбор лекарственных средств для формулярного списка//
D. анализ структуры заболеваемости и состояния лекарственного обеспечения//
Е. сформирование перечень лекарственных средств для бесплатной государственной помощи
***
Основные критерии для рационального выбора ЛС?//
A.Эффективность//
B.Безопасность//
C.Приемлемость//
D.Стоимость//
+E.Все ответы верны
***
Выберите один препарат, который не относится к ЛС рецептурного отпуска://
A.Наркотические средства//
+B.Отхаркивающие средства//
C.ЛС, предназначенные для парентерального введения//
D.Диагностические ЛС//
E.Психотропные вещества
***
Показание для назначения off-lable (использование ЛС по показаниям, не упомянутым в инструкции) применения препарата Метформин://
A.Макулярная дегенерация//
B.Прерывание беременности//
C.Легочная артериальная гипертензия//
+D.Для снижения веса//
E.Для улучшения отхождения мокроты
***
Укажите какого типа НПР вызывает препарат Талидомид, который широко применялся в 1960 годах для седации://
A.Канцерогенное, пороки развития конечностей//
B.Дозозависимое, врожденный порок сердца//
+C.Тератогенное, пороки развития конечностей//
D.Ульцерогенное, триада Грега//
E.Тератогенное, врожденный порок сердца
***
В каком возрасте педиатрических пациентов можно назначить взрослую дозировку ЛС?//
A.5 лет//
B.6 -7 лет//
C.До 12 лет//
+D.Старше 12 лет//
E.16 – 18 лет
***
К лекарственным средствам рецептурного отпуска относятся?//
A.ЛС, содержащие наркотические средства, психотропные вещества и прекурсоры, подлежащие контролю в РК//
B.ЛС, предназначенные для парентерального введения//
C.Диагностические лекарственные средства, применяемые только в условиях стационара//
D.ЛС, назначение и дальнейшее употребление которых необходимо проводить при постоянном контроле врача//
+E.Все перечисленное
***
Сколько категорий лекарственных препаратов по степени риска для плода выделяют на основании рекомендации FDA?
A.7
+B.5
C.3
D.4
E.6

***
Выберите утверждение, не соответствующее приказу МЗ РК от 22 мая 2015 года №373 «Об утверждении правил выписывания, учета и хранения рецептов»//


А. Рецепты в бумажной форме заверяются подписью медицинского работника, выписавшего рецепт//
В. Рецепты не выписываются на лекарственные средства, применяемые исключительно в организациях здравоохранения, оказывающих стационарную помощь//
+C. Рецепты на лекарственные средства, не зарегистрированные и не разрешенные к применению в Республике Казахстан, выписываются на латинском языке//
D. Рецепты выписываются с учетом возраста больного и характера действия входящих в состав лекарственного средства ингредиентов//
E. Содержание и номер рецепта на бесплатное или льготное получение лекарственных средств отражаются в амбулаторной карте больного
***
Какое количество наименований лекарственныхсредств указывается на одном рецептурном бланке на получение бесплатно или на льготной основепроизводных 8-оксихинолина,гормональных и анаболических стероидов, клонидина, кодеина, тропикамида, циклопентолата, трамадола, буторфанола?//
+A. одно//
B. не более двух//
C. не более трех//
D. не более четырех//
E. не более пяти
***
Какое количество наименований лекарственныхсредств указывается на одном рецептурном бланке на получение бесплатно или на льготной основеэналаприла, бисопролола, нифедипина, сальбутамола, амоксициллина?//
A. одно//
+B. не более двух//
C. не более трех//
D. не более четырех//
E. не более пяти
***
Выберите правильный ответ. При выписывании лекарственного средства в дозе, превышающей высшую разовую, врач в рецепте обозначает дозу лекарственного средства прописью и …://
А. вопросительным знаком//
+В. восклицательным знаком//
С. опознавательным знаком «Стоп – медикамент высокого риска»//
D. опознавательным знаком «Стоп – медикамент схожий по названию»//
E. опознавательным знаком «Стоп – высшая разовая доза»
***
Как правильно выписываютсярецепты?//
А. на латинском языке под международным непатентованным наименованием лекарственных средств//
B. в случае индивидуальной непереносимости пациента, многокомпонентного состава лекарственного препарата выписываются под торговым наименованием//
C. рецепты выписываются разборчиво с обязательным заполнением всех предусмотренных в бланке граф//
+D. все перечисленное верно//
E. способ применения прописывается в соответствующей графе с указанием дозы, частоты и длительности приема
***
Выберите правильное утверждение://
A. Рецепты на получение лекарственных средств бесплатно или на льготных условиях дополнительно подписываются уполномоченным лицом, определенным приказом медицинской организации и заверяются печатью организации здравоохранения "Для рецептов"//
B. Рецепты на получение лекарственных средств бесплатно или на льготных условиях дополнительно подписываются уполномоченным лицом, определенным приказом медицинской организации и заверяются электронно-цифровой подписью уполномоченного лица//
C. Заведующий фельдшерско-акушерским пунктом не заверяет рецепт на получение лекарственных средств бесплатно или на льготной основе своей подписью//
D. Заведующий фельдшерско-акушерским пунктом не заверяет рецепт на получение лекарственных средств бесплатно или на льготной основе печатью фельдшерско-акушерского пункта или электронно-цифровой подписью//
+E. Рецепты на получение лекарственных средств бесплатно или на льготных условиях дополнительно подписываются уполномоченным лицом, определенным приказом медицинской организации и заверяются печатью организации здравоохранения "Для рецептовили заверяются электронно-цифровой подписью уполномоченного лица "

***
Определите членов, входящих в состав формулярной комиссии в медицинской организации://


A.ответственный по антибиотикам, провизор/фармацевт аптеки, главная медицинская сестра, комитет по оценке использования лекарственных средств//
B.администрация, клинический фармаколог, заведующие отделений, сотрудники ВУЗ (кафедры)//
C.главная медицинская сестра, комитет по оценке использования лекарственных средств//
+D.администрация, клинический фармаколог, заведующие отделениями, сотрудники ВУЗ, ответственный по антибиотикам, провизор/фармацевт аптеки, ответственный по оценке использования лекарственных средств//
E.клинический фармаколог, заведующие отделений, сотрудники ВУЗ (кафедры)

***
Для какого из формулярных компонентов характерны следующие определения: «перечень лекарственных средств, наличие которых обязательно в достаточных количествах, сформированный для оказания гарантированного объема бесплатной медицинской помощи с учетом профиля организации здравоохранения в порядке, определяемом уполномоченным органом»://


A.функции формулярной комиссии//
+B.лекарственный формуляр//
C.формулярной справочник//
D.стандарты фармакотерапии и клинические протоколы ведения больных//
E.системы контроля, оценки и мониторинга использования лекарств и нежелательных реакций
***
Укажите критерии рационального выбора лекарственного средства по рекомендации ВОЗ://
A.Эффективность, безопасность//
B.Приемлемость, стоимость//
+C.Эффективность, безопасность, приемлемость, стоимость//
D.Безопасность, приемлемость, стоимость//
E.Эффективность, безопасность, приемлемость
***
Определите категорию C по тератогенности ЛС с назначением FDA (FederalDrugAdministration)://
A.лекарственные средства, входящие в эту группу, безвредны для плода на протяжении всей беременности (препараты железа, трийодтиронин)//
B.применение препаратов сопряжено с определённым риском для плода, однако польза от их применения превосходит возможное побочное действие (диазепам, доксициклин, канамицин, диклофенак)//
C.доказано тератогенное действие препаратов этой группы, их приём противопоказан до и во время беременности (изотретиноин, карбамазепин, стрептомицин)//
+D.в исследованиях на животных выявлено тератогенное или эмбриотоксическое действие препарата, контролируемых испытаний не проводилось, либо действие препарата не изучено (изониазид, фторхинолоны, гентамицин, противопаркинсонические препараты, антидепрессанты)//
E.экспериментальные исследования не выявили тератогенного действия, либо наблюдаемые у животных осложнения не обнаружены у детей, матери которых принимали лекарственные препараты, входящие в эту группу (инсулин, аспирин, метронидазол)
***
Что такое формулярная система?//
A. Консультативно-совещательный орган, ответственный за лекарственную политику РК//
B. Перечень лекарственных средств с доказанной клинической эффективностью и безопасностью//
+C.Комплекс информационно-методологических мероприятий организации здравоохранения, обеспечивающая рациональное использование лекарственных средств в рамках ГОБМП и современную, качественную медицинскую помощь для граждан РК//
D. это важный фактор, влияющий на выбор лечения//
E. изучает эффекты лекарств на уровне популяций или больших групп людей//
***
Аббревиатура Казахстанского национального лекарственного формуляра//
A. BNF//
B. FNO//
C. PИЛС//
D. FDA//
+E. KNF
***
Выберитепредседателяформулярнойкомиссии//
+A. главныйврач//
B. заведующийхирургией//
C. старшаямедицинская сестра//
D. реаниматолог//
E. заведующаяаптекой
***
На основании рекомендаций FDA (FederalDragAdministration) к какой категории относятся препараты железо и трийодтиронин?//
+A. категория A//
B. категория B//
C. категория C//
D. категория D//
E. категория X
***
Укажите препараты, относящиеся к категории B, в соответствии с рекомендациями FDA (FederalDragAdministration)//
А. диклофенак, доксицилин//
B. гентамицин, фторхинолоны//
C. диазепам, канамицин//
D. карбамазепин, стрептомицин//
+E. инсулин, аспирин, метронидазол

***

Не является медикаментозной ошибкой://
А. Врач выписывает назначения препарата без указания способа введения//
В. Фармацевт выдает неправильный препарат вследствие неверного прочтения выписанного врачом рецепта//
С. Лица, ухаживающие за пациентом, дают пациенту препарат в неверной дозе//
Д. Использование просроченных лекарственных препаратов//
+Е. Проведение назначений согласно правилам пяти правильных «П»

***
Компоненты формулярной системы://


А. Формулярная комиссия//
В. Лекарственный формуляр//
С. Формулярный справочник//
Д. Стандарты фармакотерапии и клинические протоколы диагностики и лечения//
+Е. Все ответы верны

***
Дайте определение Формулярной комиссии//


+А. Консультативно-совещательный орган, ответственный за лекарственную политику в медицинской организации//
В. Утвержденный руководителем организации здравоохранения и согласованный в порядке, определяемом уполномоченным органом в области здравоохранения, перечень лекарственных средств (ЛС), сформированный для оказания гарантированного объема бесплатной медицинской помощи с учетом профиля организации здравоохранения, наличие которых обязательно в достаточных количествах//
С. Утвержденный, ограниченный список ЛС, отобранных уполномоченными экспертами по критериям эффективности, безопасности и результатам фармакоэкономического анализа для обеспечения рационального лечения//
Д. Комплекс информационно-методологических мероприятии организации здравоохранения, обеспечивающая рациональное использование лекарственных средств в рамках ГОБМП и современную, качественную медицинскую помощь для граждан своей страны//
Е. Все ответы верны

***
Не входит в состав формулярной комиссии://


А. Клинический фармаколог//
+В. Экономист //
С. Заведующий отделением//
Д. Администрация//
Е. Провизор/фармацевт аптеки

***
Функции формулярной комиссии://


А. Внедрение принципов рациональной фармакотерапии//
В. Оценка и отбор ЛС для формуляра и обеспечение его регулярного пересмотра//
+С. Все ответы верны//
Д. Рассмотрение случаев развития побочных реакций на применение ЛС и принятие соответствующих мер//
Е. Решение проблем, связанных с ошибками при проведении лечения
К производному фенилалкиламина относится антагонист кальция://
Дилтиазем//
Нифедепин//
+ фелодипин//
Амлодипин//
верапамил
***

Назначение антагонистов кальция предпочтительно при сочетании артериальной гипертензии://


с бронхообструктивными заболеваниями//
+ с почечной недостаточностью//
с печеночной недостаточностью//
с сердечной недостаточностью//
с синдромом слабости синусового узла
***

Нерациональной комбинацией антигипертензивных средств, является://


бета-адреноблокаторы и альфа-адреноблокаторы//
+ ингибиторы АПФ и антагонисты кальция//
бета-адреноблокаторы и антагонисты кальция//
ингибиторы АПФ и диуретики//
блокаторы рецепторов ангиотензина II и диуретики
***

Противопоказанием для назначения ингибиторов АПФ, является://


Беременность//
сердечная недостаточность//
сахарный диабет//
синдром Рейно//
+ подагра

***
Потребность в инсулине в дебюте (первые 1-3 года сахарного диабета типа 1) составляет://


+ 1 ед/кг//
менее 0,3 ед/кг//
0,3-0,5 ед/кг//
0,6-0,9 ед/кг//
1,1-1,2 ед/кг

***
Для действия препаратов сульфонилмочевины не характерно://


закрытие АТФ-зависимых калиевых каналов//
открытие кальциевых каналов//
повышение чувствительности периферических тканей к инсулину//
гепатотоксичность//
+ повышение уровня лактата

***
Препаратами первого выбора у больных сахарным диабетом с микроальбуминурией являются://


бета-2-блокаторы//
ингибиторы АПФ//
петлевые диуретики//
+ верапамил//
бета-1-блокаторы

***
Какова суточная доза ранитидина при лечении обострения язвенной болезни://


100 мг 2 раза в сутки//
150 мг 2 раза в сутки//
+300 мг 2 раза в сутки//
400 мг 2 раза в сутки//
500 мг 2 раза в сутки

***
При лечении стафилококкового дисбактериоза используются все препараты, КРОМЕ://


Эритромицин//
+ ампициллин//
Линкомицин//
Канамицин//
метронидазол

***
К противодиарейным средствам относятся все, КРОМЕ://


Лоперамид//
настой дубовой коры//
настой ольховых шишек//
+ смекта//
гуталакс

***
Препарат для лечения врожденного токсоплазмоза://


Эритромицин//
Хлоридин//
Пенициллин//
Цефотаксим//
+стрептомицин

***
Какой препарат применяется при лечении дифтерийного миокардита://


Реополиглюкин//
Эуфиллин//
+корглюкин//
Сульфокамфокаин//
строфантин

***
Какой химиопрепарат показан при лечении неосложненной кори у детей старшего возраста://


не показан//
пенициллин//
гентамицин//
бисептол//
+эритромицин

***
Пропранолол применяется при ишемической болезни сердца потому, что://


снижает потребность миокарда в кислороде//
расширяет коронарные сосуды//
вызывает спазм периферических сосудов//
+увеличивает потребность миокарда в кислороде//
увеличивает сократительную способность миокарда

***
Частые, политопные и ранние желудочковые экстрасистолы в остром периоде инфаркта миокарда является показанием к терапии://


сердечными гликозидами//
лидокаином//
алупентом//
реланиумом//
+верапамилом

***
Противопоказаниями для применения бета-адреноблокаторов при инфаркте миокарда являются://


мерцательная аритмия//
наджелудочковая экстрасистолия//
+ желудочковая экстрасистолия//
атриовентрикулярная блокада//
глаукома

***
В четырехкомпонентную терапию при хроническом гломерулонефрите не входят://


цитостатики (2-3 мг/кг в сутки)//
глюкокортикосткроиды (преднизолон 40-100 мг/сут)//
нестероидные противовосалительные средства (индометаци до 150 мг/сут)//
+прямые антикоагулянты (гепарин 20000 ед/сут)//
антиагреганты (курантил 400-600мг/сут)

***
Для лечения хронического легочного сердца наиболее эффективны://


Антибиотики//
Антикоагулянты//
Кислородотерапия//
+бронхолитики//
глюкокортикостероиды

***
Основу долгосрочного лечения бронхиальной астмы составляют://


ингаляционные симпатомиметики короткого действия//
ингаляционные симпатомиметики пролонгированного действия//
ингаляционные кортикостероиды//
+пероральные кортикостероиды//
пролонгированные метилксантины

***
Базисная терапия при бронхиальной астме включает применение://


+сальбутамола и атровента//
Серетида//
беротек и аминофиллин//
сальбутамол и беротек//
ацетилцистеин и сальбутамол

***
Наиболее эффективной схемой лечения подострого гломерулонефрита является://


пульс-терапия стероидными препаратами с последующим назначением массивных доз преднизолона и цитостатиков//
+плазмаферез, пульс-терапия с последующей четырехкомпонентной терапией (преднизолоном, цитостатиками, гепарином, курантилом)//
плазмаферез, пульс-терапия с последующим назначением индометацина//
азатиоприн, делагил, трентал//
лейкеран, гепарин, фуросемид, дицинон

***
Показанием к назначению цитостатиков при хроническом гломерулонефрите является все, КРОМЕ://


неэффективность глюкокортикостероидов//
наличие осложнений кортикостероидной терапии//
сочетание нефотического ХГН с артериальной гипертензией//
+мезангиопролиферативный гломерулонефрит//
рецидивирующий нефротический синдром

***
Укажите клинические состояния, являющиеся показанием для проведения монотерапии НПВС://


внесуставные ревматические заболевания (миозит, тендовагинит, синовит)//
+подагрический артрит//
дерматомиозит//
склеродермия//
мигрень

***
Одновременный прием ацетилсалициловой и аскорбиновой кислот приводит к://


ослабление клинической эффективности ацетилсалициловой кислоты (АСК)//
+повышение концентрации АСК в плазме крови за счет снижения скорости её выделения почками//
повышение концентрации АСК в плазме крови за счет нарушения её метаболизма в печени//
снижению концентрации АСК в крови за счет нарушения абсорбции в ЖКТ//
снижение клинической эффективности АСК за счет уменьшения связывания её с белками плазмы

***
Укажите селективный бета-2-адреномиметик://


изопреналин (изадрин)//
+эфедрин//
орципреналин (алупент)//
сальметерол//
ипратропиум (атровент)

***
Укажите селективный бета-2-адреномиметик короткого действия://


Сальметерол//
индактерол//
+сальбутамол//
Формотерол//
Ацебуталол

***
Отметить основную локализацию действия дихлотиазида://


на уровне клубочка//
проксимальные канальцы нефрона//
+дистальные канальцы нефрона//
восходящая часть петли Генле//
собирательные трубки

***
Определяющим критерием для назначения медикаментозной терапии артериальной гипертензии является://


степень повышения АД//
длительность заболевания//
принадлежность к группе риска и степень повышения АД//
+возраст больного//
все перечисленное

***
Медикаментозное лечение артериальной гипертензии (впервые выявленной) начинают://


+с минимальных доз одного препарата//
максимальных доз одного препарата//
минимальных доз нескольких антигипертензивных средств//
максимальных доз нескольких антигипертензивных средств//
нет правильного ответа

***
Бета-адреноблокаторы следует отменять://


резко, без предшествующего снижения дозы//
+постепенно с предшествующим снижением дозы//
длительно, в течение нескольких месяцев//
с перерывами в несколько недель//
нет правильного ответа

***
Для устранения острых приступов стабильной стенокардии обычно применяется://


+бетаадреноблокатор//
антагонист кальция//
нитроглицерин//
ацетилсалициловая кислота//
все перечисленное

***
При назначении антиаритмических препаратов необходимо учитывать://


+характер аритмии//
уровень артериального давления//
состояние хроно, дромо, инотропной функции сердца//
наличие нарушений проводимости//
все ответы верны

***
Препарат, обычно используемый для поддержания нормосистолии при мерцательной аритмии://


Строфантин//
Дигоксин//
Резерпин//
+миноксидил//
Лидокаин

***
Для антигеликобактерной терапии рекомендуются следующие ЛС://


метронидазол, кларитромицин, амоксициллин//
линкомицин, фузидин//
+ципрофлоксацин, пефлоксацин//
фуродонин, фурагин//
сукральфат, викалин

***
Указать по убыванию антисекреторного действия антисекреторные препараты://


ингибиторы протонной помпы > Н2-гистаминоблокаторы >М-холинолитики//
Н2-гистаминоблокаторы>М-холинолитики> ингибиторы протонной помпы//
+М-холинолитики> ингибиторы протонной помпы > Н2-гистаминоблокаторы//
ингибиторы протонной помпы >алмагель> Н2-гистаминоблокаторы//
М-холинолитики> алмагель > Н2-гистаминоблокаторы> ингибиторы протонной
помпы

***
Определите, о каком препарате идет речь: действует при пониженном содержании в крови уровня протромбина, при недостатке его в организме развивается кровоточивость, является функциональным антагонистом антикоагулянтов непрямого действия://


+викасол//
Неодикумарин//
Стрептолиаза//
Гепарин//
фибриноген

***
Определите, о каком препарате идет речь: способствует сохранению тромба путем ингибирования фибринолизина, оказывает специфическое кровоостанавливающее действие при кровотечениях, связанных с повышенным фибринолизином, выводится с мочой примерно за 4 часа, применяется в/в или внутрь://


Викасол//
Неодикумарин//
+стрептолиаза//
Гепарин//
Фибриноген

***
Антибиотик с выраженной активностью по отношению к синегнойной палочке://


Амоксициллин//
Эритромицин//
Цефтазидим//
+цефамандол//
Оксациллин

***
Какие препараты необходимо назначать в период острого подагрического приступа://


аллопуринол, алломарон//
+ бенемид, антуран//
колхицин, индометацин, //
бензбромарон, алломарон//
этионамид, диклофенак натрия

***
Какие побочные реакции НПВС корригирует комбинированный препарат, имеющий в своем составе диклофенак натрия и мизопростол://


кровотечение, т.к. воздействует на агрегационные свойства крови//
+«НПВСгастропатии», т.к. ингибирует секрецию НСl в желудке//
апластические анемии//
задержку натрия//
цитопении

***
Укажите время продолжительности максимального бронхолитического эффекта при ингаляционном применении изадрина://


11,5 часа//
+34 часа//
56 часов//
78 часов//
10-12 часов

***
В случаях неосложненной артериальной гипертензии при отсутствии показаний других гипотензивных средсв, следует давать предпочтение к назначению://


+антагонистов кальция//
центральных адреномиметиков//
симпатолитиков//
бета-адреноблокаторов и диуретиков//
Бета-адреноблокаторов

***
Индивидуальный выбор антигипертензивных средств: преимущественное показание назначения диуретиков при артериальной гипертензии с учетом следующей сопутствующей патологии://


+систолическая гипертония в сочетании с сердечной недостаточностью//
Стенокардия//
Дислипидемия//
Подагра//
сахарный диабет

***
Применение изосорбита динитрата больше показано://


+для быстрого устранения симптомов стенокардии//
для лечения острого инфаркта миокарда//
для профилактики приступов стенокардии//
для профилактики тромбоэмболических осложнений//
все ответы верные

***
Противопоказание к назначению нитроглицерина://


+констриктивный перикардит//
нестабильная стенокардия//
артериальная гипертензия//
сердечная недостаточность//
левожелудочковая недостаточность
***
При невозможности контролировать частоту желудочковых сокращений при фибрилляции предсердий (тахисистолия) на фоне приема дигоксина к лечению можно добавить://
строфантин//
+бета-адреноблокатор//
ингибитор АПФ//
лидокаин//
препараты калия

***
Для предотвращения осложнений антикоагулянтной терапии у пациентов с фибрилляцией предсердий (ФП) к лечению можно добавить://


Алмагель//
Ранитидин//
+Пантопразол//
Мизопростол//
Гидрокарбонат натрия

***
Указать преимущественные показания к назначению М-холиноблокаторов при гастродуоденальной патологии://


гиперсекреция соляной кислоты в сочетании с гипермоторными нарушениями желудка и 12-перстной кишки при функциональной неязвенной болезни//
диспепсии//
синдроме Золлингера-Эллисона//
+симптоматической язве//
монотерапии язвенной болезни

***
Указать преимущественные показания к назначению ингибиторов Н+ –К+-АТФазы://


рефлюкс эзофагит (ГЭРБ)//
функциональная неязвенная диспепсия//
хронический холецистит//
+синдром раздраженного кишечника//
нет правильного ответа

***
При каких из следующих ситуаций применение стрептокиназы и аналогичных препаратов противопоказано://


Кровотечение//
послеоперационный и послеродовый период//
беременность//
все ответы верны//
+нет правильного ответа

***
Какие побочные эффекты могут встречаться при применении гепарина://


геморрагические//
рикошетные тромбозы//
остеопороз//
локальное или генерализованное выпадение волос//
+все ответы верны

***
Какой препарат предпочтителен при анаэробном сепсисе://


Фузидин//
+фторхинолоны//
Полимиксины//
метронидазол//
гентамицин

***
Указать механизм диуретического действия спиронолактона://


устраняет влияние альдостерона на функцию почечных канальцев//
угнетает реабсорбцию ионов Na и С1//
+угнетает синтез альдостерона//
создает высокое осмотическое давление//
угнетает центры симпатической нервной системы

***
При желудочковой тахикардии (без падения АД) предпочтительным средством неотложной помощи является://


Верапамил//
Нифедипин//
+ лидокаин//
Пропранолол//
ингибитор АПФ

***
Показания к применению мизопростола (сайтотек, цитотек) в комплексной терапии язвенной болезни://


неосложненное течение язвенной болезни без сопутствующей патологии//
язвенная болезнь у курящих и злоупотребляющих алкоголем, при невосприимчивости к лечению ингибиторам протоновой помпы и Н2-гистаминоблокаторами//
при сочетании язвенной болезни и хронического некалькулезного холецистита//
+язвенная болезнь желудка и двенадцатиперстной кишки, язвы, вызванные применением НПВС (лечение и профилактика)//
при сочетании язвенной болезни и ГЭРБ

***
Укажите факторы риска геморрагических побочных проявлений гепарина и антикоагулянтов непрямого действия://


несоблюдение противопоказаний, неправильный учет факторов риска//
неадекватный контроль за безопасностью применения//
прерывистое применение больших доз//
+длительное /более 2-3 недель/ применение в лечебном диапазоне доз//
все ответы верны

***
Отберите относительные противопоказания к применению гепарина://


тяжелая форма сахарного диабета с распространенными ангиопатиями//
сепсис, бактериальный эндокардит//
выраженный атеросклероз сосудов головного мозга с частыми нарушениями мозгового кровообращения//
тяжелые поражения печени и почек//
+все ответы верны

***
Какой из перечисленных антибиотиков лучше проникает в костную ткань://


Бензилпенициллин//
Ампициллин//
Рифампицин//
Фурадонин//
+Линкомицин

***
Идиосинкразия это://


+повышение чувствительности организма к лекарственному средству//
накопление в организме лекарственного средства//
необычная реакция организма на лекарственное средство//
аллергическая реакция на лекарственное средство//
выведение лекарственного средства из организма

***
Указать наиболее эффективные НПВС при лечении больных болезнью Бехтерева://


ибупрофен, постал//
опирин, кетопрофен//
+фенилбутазон, диклофенак натрия, индометацин//
ацетилсалициловая кислота, ацетаминофен, флугалин//
все вышеперечисленные

***
Показания для назначения базисных препаратов больным ревматоидным артритом (РА)://


течение РА не контролируется НПВС, активность процесса сохраняется в течение 6 месяцев//
прогрессирование заболевания: вовлечение новых суставов, появление признаков разрушения хряща (сужение суставных щелей), костных эрозий при рентгенологическом исследовании//
потребность в больших дозах глюкокртикостероидов//
+ нарастание титров ревматоидного фактора//
все вышеперечисленное

***
Показанием к применению пролонгированных симпатомиметиков является://


+длительная терапия бронхиальной астмы (легкая и средняя степень)//
снятие приступа удушья//
астматический статус//
хронический бронхит//
все ответы верны

***
Фенотерол расширяет бронхи путем://


блокирования альфа рецепторов//
селективного возбуждения бета-2-адренорецепторов//
непосредственного влияния на гладкую мускулатуру бронхов//
стабилизации тучных клеток//
+снижения тонуса блуждающего нерва

***
Указать механизм диуретического действия маннита://


устраняет влияние альдостерона на функцию почечных канальцев//
угнетает реабсорбцию ионов Na и С1//
угнетает синтез альдостерона//
+создает высокое осмотическое давление//
угнетает центры симпатической нервной системы

***
Указать механизм диуретического действия дихлотиазида://


устраняет влияние альдостерона на функцию почечных канальцев//
+снижает реабсорбцию ионов Na//
угнетает синтез альдостерона//
создает высокое осмотическое давление//
угнетает центры симпатической нервной системы

***
Индивидуальный выбор антигипертензивных средств: преимущественное показание назначения бета-адреноблокаторов при артериальной гипертонии с учетом следующей сопутствующей патологии://


перенесенный инфаркт миокарда//
систолическая гипертония//
заболевания периферических сосудов//
+депрессия//
Дислипидемия

***
Индивидуальный выбор антигипертензивных средств: преимущественное показание назначения антагонистов кальция (нифедипина) при артериальной гипертонии с учетом следующей сопутствующей патологии://


стенокардия//
сердечная недостаточность//
перенесенный инфаркт миокарда с признаками сердечной недостаточности (НК II)//
тахиаритмия//
+атриовентрикулярная блокада II и III степени

***
Понятие интервальной терапии для предотвращения толерантности к нитратам://


перерыв в приеме нитратов на 46 часов//
+перерыв в приеме нитратов на 8-12 часов//
перерыв в приеме нитратов бна 24 часа//
перерыв в приемсе нитратов на 3 суток//
все выше перечисленное верно

***
Для купирования тахикардии типа "пируэт" (двунаправленная веретенообразная желудочковая тахикардия) используется блокатор рецепторов://


Амиодарон//
+бета-адреноблокаторы//
Соталол//
Лидокаин//
ингибитор АПФ

***
К нежелательным реакциям типа В не относится ://


анафилактический шок на введение новокаина//
агранулоцитоз при приеме анальгина//
гемолитический криз при приеме сульфаниламидов//
+ острая язва желудка при приеме пироксикама//
лихородка при приеме ибупрофена

***
Развитие нежелательных действий лекарств наиболее вероятно у пациентов://


при хронических заболевании печении//
страдающих бронхиальной астмой//
пожилого возраста//
при нарушении функции почек//
+ при всех вышеуказанных состояниях

***
Выберите не характерный признак для нежелательной лекарственной реакции типа А://


дозозависимость эффекта//
заболевания почек, увеличивающие риск их развития//
+ идиосинкразия к применяемому препарату//
изменение чувствительности рецепторов к лекарственному препарату//
связь с механизмом действия лекарственного препарата

***
Критериями тяжести нежелательной лекарственной реакции являются все, КРОМЕ://


опасности для жизни больного//
+ эмбриотоксического действия//
продления сроков пребывания в больнице//
стойкой потери трудоспособности//
частоты развития

***
У больного в анамнезе – анафилактической шок на пенициллин. Применение какого антибактериального препарата ему противопоказано://


+Цефуроксим//
Тетрациклин//
Ципрофлоксацин//
Кларитромицин//
Ко-тримоксазол

***
К нежелательным реакциям типа С относится://


лекарственная зависимость//
+ эмбриотоксичность//
Ксеростомия//
Крапивница//
медикаментозная нейтропения

***
Укажите правильное утверждение://


у больных с аллергической реакций в анамнезе показано проведение аллергических кожных проб перед началом терапии антибиотиками//
пациенту с аллергией на новокаин противопоказано введение лидокаина//
+препараты с узким терапевтическим диапазоном чаще вызывают нежелательные реакции типа А//
тяжесть аллергической реакции зависит от дозы препарата//
плод наиболее чувствителен к действию лекарственных препаратов в III триместре беременности

***
Фактором риска развития нежелательных действий лекарств является://


наличие хронического синусита//
запивание ЛП водой//
+злоупотребление алкоголем//
возраст 20-45 лет//
короткий курс приема ЛС

***
Синдром Стивенсона – Джонсона наиболее часто может наблюдаться у пациентов, принимающих://


+ сульфаниламиды//
дигоксин//
инсулин//
преднизолон//
дифенгидрамин

***
Фотосенсибилизации возможна при приеме://


+фторхинолонов//
аминогликозидов//
фуросемида//
амоксицлиина//
метамизола

***
Кандидоз полости рта может наблюдаться на фоне применения://


пилокарпина//
+беклометазона//
фенилбутазона//
оральных контрацепетивов//
солей лития

***
Результатом взаимодействия ЛС может быть://


снижение терапевтической эффективности//
развитие токсического нежелательного действия//
повышение терапевтической эффективности//
инактивация одного из взаимодействующих ЛС//
+ все вышеуказанное

***
При сочетанном применении тетрациклина и антацида для предупреждения нежелательного лекарственного взаимодействия следует://


отказаться от применения антацида//
+ принимать антацид до еды, а тетрациклин – после//
интервал между приемом антацида и тетрациклина должен составлять 2 часа//
принимать тетрациклин до еды, а антацид – после//
принимать тетрациклин во время еды

***
При фармакокинетическом взаимодействии://


ЛС вступают в химическое взаимодействие друг с другом//
одно ЛС меняет чувствительность специфических рецепторов к другому ЛС//
два ЛС конкурируют за специфический рецептор//
+ одно изменяет метаболизм другого ЛС//
действие одного ЛС меняет выраженность эффекта другого ЛС

***
Мужчина 46 лет страдает хроническим обструктивным бронхитом, ежедневно принимает 200 мг теофиллина. Врач-стоматолог назначил ему 2,0г эритромицина в сутки по поводу острого периостита. Через 3 дня у пациента появились сердцебиение, перебои в сердце, головная боль, тремор, возбуждение, бессонница. С чем связано развитие этих симптомов://


Аллергическая реакция на эритромицин//
+ Замедление метаболизма, повышение концентрации в крови теофиллина и развитие его токсических эффектов в результате ингибирования микросомальных ферментов печени эритромицином//
Замедление почечного клиренса, повышение концентрации в крови и развитие токсических эффектов эритромицина в результате действие теофиллина на почечный кровоток//
Ускорение метаболизма и выведение теофиллина под влиянием эритромицина и развитие синдрома отмены//
Увеличение всасывания, повышение концентрации в крови и развитие токсического действия теофиллина в результате прокинетического действия эритромицина

***
Каков механизм лекарственного взаимодействия между местным анестетиком и адреналином при их сочетанном использовании во время проведения местной анестезии://


Физико-химическое взаимодействие//
Повышение чувствительности специфических рецепторов к местному анестетику под влиянием адреналина//
+Снижение абсорбции и повышение концентрации местного анестетика в месте введения под влиянием адреналина//
Повышение абсорбции и увеличение системной доступности местного анестетика под влиянием адреналина//
Уменьшение почечного кровотока и снижение элиминации местного анестетика под влиянием адреналина

***
Мужчина 61 года получает ежедневно 6 мг варфарина после операции аортокоронарного шунтирования. При очередном обследовании международное нормализированное отношение (показатель уровня протромбина в крови) равно 8 при целевых значениях этого коэффициента 2-3. Он также получает эналаприл 10 мг/сут и атенолол 100 мг/сут, а за неделю до обследования врач-стоматолог назначил пациенту по поводу кандидоза слизистой оболочки полости рта 4 раза в день аппликации на пораженные места 2% крема с миконазолом. В чем причина такого снижения уровня протромбина в крови://


+Миконазол ингибирует микросомальные ферменты печени, что нарушает метаболизм варфарина и увеличивает его антикоагулянтное действие//
Миконазол ингибирует микросомальные ферменты печени, что нарушает метаболизм атенолола и увеличивает его антикоагулянтное действие//
Эналаприл ингибирует микросомальные ферменты печени, что нарушает метаболизм варфарина и увеличивает его антикоагулянтное действие//
Миконазол увеличивает всасываемость варфарина, что повышает его антикоагулянтное действие//
Прямое угнетающее действие миконазола на синтез протромбина

***
Принимая ЛС, их следует запивать://


молоком//
фруктовыми соками//
+кипяченой водой//
минеральной водой//
всем вышеуказанным

***
Всасывание питательных веществ из пищи уменьшается под действием://


атропина//
+ слабительных средств//
витамина С//
анальгина//
нифедипина

***
У Б.48л.отмечаются приступы стенокардии при умеренных физических нагрузках. В анамнезе коллаптоидное состояние после однократного приема нитроглицерина сублингвально (с тех пор б-ной нитроглицерин не принимал). Сопутствующие заболевания -ГБ (рабочий уровень АД 160/100мм.рт.ст.,гипофункция щитовидной железы. На момент осмотра АД 190/100мм.рт.ст.,ЧСС 72 в минуту. Больному противопоказаны://


Бета-адреноблокаторы//
+ Амиодарон//
Верапамил и дилтиазем//
Каптоприл//
Нифедипин

***
Б.по поводу гипертонического криза вводили натрия нитропруссид в/в капельно в больших дозах (со скоростью 8 мкг/мин). Появились одышка, акроцианоз, давящие боли за грудиной, мышечные подергивания. Какова причина ухудшения состояния б-ного://


+Токсическое действие цианидов//
Отрицательный инотропный эффект нотропруссида натрия//
Ухудшение бронхиальной проходимости//
Застойные явления в малом круге кровообращения//
Все ответы верны

***
У б-ной 28л. с диагнозом СКВ на фоне ХПН появились отеки голеней, увеличение печени. При эхокардиографическом исследовании определяется снижение сердечного выброса. ЧСС 95 в мин, АД 170/100 мм.рт.ст. Какие сердечные гликозиды показаны больной://


Никакие//
+ Дигитоксин//
Дигоксин//
Строфантин//
Коргликон

***
У б-ной 28л. с диагнозом СКВ на фоне ХПН появились отеки голеней, увеличение печени. При эхокардиографическом исследовании определяется снижение сердечного выброса. ЧСС 95 в мин, АД 170/100 мм.рт.ст. Больная принимает дигитоксин. В связи с появлением судорожного синдрома дополнительно назначен фенобарбитал (0,3 г/сут). Когда возникнут изменения в состоянии б-ной при наличии влияния://


+После первого приема фенобарбитала//
Через сутки//
Через 7-14 дней//
Через несколько месяцев от начала терапии//
Все ответы верны

***
Б.41г. поступила в стационар с жалобами на резкую слабость, одышку при ходьбе. При обследовании в анализе крови выявлена анемия (гемоглобин-56 г/л), цветной показатель 1,2, при осмотре языка - глоссит. В пунктате костного мозга выявлен мегалобластный тип кроветворения. Концентрация железа в сыворотке крови в пределах нормы. Диагноз: В 12-дефицитная анамия. Выберите наиболее оптимальный вариант лечения://


+Вит.В12 в дозе 500 мкг/сут через день, фолиевая кислота в дозе 1,5 мг/сут, сульфат железа (80 мгFe2+) 1 раз в сутки//
Вит.В12 в дозе 50 мкг/сут ежедневно, фолиевая кислота по 0,15 мг/сут//
Железа сульфат, фолиевая кислота//
Вит.С 500 мг/сут, железа сульфат//
Все ответы верны

***
Б.54л.в течение 5 лет страдает деформирующим остеоартрозом нижних конечностей с выраженными синовитами. В анамнезе отмечает лекарственную аллергию (на бутадион, гепарин, метиндол, пенициллин, теофиллин).В стационаре б-ной назначены реопирин по 5 мл в/м 1 раз в сутки, гидрокортизона гемисукцинат по 100 мг в полость коленных суставов, тавегил по 0,001г 2 раза в сутки. Через 3 дня у б-ной появились зудящие эритематозные высыпания на коже туловища. Какова наиболее вероятная причина


ухудшения состояния://
Естественное течение основного заболевания//
Кожные проявления не диагностированного ранее системного васкулита//
+ Лекарственная аллергическая реакция//
Грибковое заболевание кожи//
Все ответы верны

***
Пациенту с ревматоидным артритом был назначен метотрексат. Как скоро проявится действие метотрексата://


+Через 2 ч после первого приема//
Через 1 нед//
Через 1 мес//
Через несколько месяцев//
Не ранее чем через год

***
У Б-ой 52л .развилась картина гипертонического криза 2 типа с ЧСС 62 в мин. АД 200/140 мм.рт.ст. В легких большое кол-во влажных мелкопузырчатых хрипов. С какого препарата необходимо начинать купирование криза://


Нифедипин//
+ Фуросемид//
Маннитол//
Клофелин//
Каптоприл

***
Б.43л.в течение 6 лет страдает сахарным диабетом 1 типа, получает инсулин по 54 ЕД/сут, что поддерживает уровень гликемии в пределах 7,0 ммоль/л.В последнее время в связи с повышением АД до 160\90 мм.рт.ст. лечащий врач назначил гипотиазид в суточной дозе 75 мг в комбинации с эналаприлом в дозе 5 мг.Через 10 дней у больного уровень сахара в крови 10,5 ммоль\л, отмечается ухудшение самочувствия. Что является ведущей причиной изменения уровня сахара в крови://


Неправильный режим дозирования обоих препаратов//
Комбинация эналаприла с гипотиазидом//
+ Применение относительно высоких доз диуретика//
Естественное течение заболевания//
Нет правильного ответа

***
У пациента 37 лет развилась судорожная форма гипертонического криза, состояние тяжелое, цифры АД 200\120 мм.рт.ст., ЧСС 120 в мин. С назначения какого препарата необходимо начинать терапию://


Клофелина//
Дроперидола//
+ Диазепама//
Нитропруссида натрия//
Пентамина

***
Б.45л. на фоне применения антибиотика цефтриаксона в течение 10 дней развилась картина псевдомембранозного колита. Какой первый шаг алгоритма врачебной помощи://


отмена антибиотика, назначение флуконазола//
назначение цефепима//
назначение сульфаниламидных препаратов//
+отмена цефтриаксона, назначение ванкомицина или метронидазола//
назначение амикацина

***
Б.58л. по поводу обострения язвенной болезни желудка в комплексе терапии назначен кларитромицин. Назовите основные отличительные признаки препарата от эритромицина://


более высокая активность в отношении хеликобактер и атипичных микобактерий//
лучшая биодоступность//
при почечной недостаточности возможно увеличение периода полураспада//
не применяется при беременности и лактации//
+ все верно

***
У Б.48л. после перенесенной операции на брюшной полости на 4-е сутки развилась левосторонняя нижнедолевая пневмония. Результаты экспресс-анализа показали наличие MRSA, пенициллино- и аминогликозидорезистентных штаммов энтерококков. Препараты выбора://


Цефалоспорины 3 поколения + аминогликозиды//
Азтреонам//
Амикацин + пиперациллин//
+Ванкомицин//
Клиндамицин + метронидазол

***
Б. 45л. находится в отделении интенсивной терапии по поводу синегнойной инфекции. Выберите препараты 1 ряда для лечения://


Цефалоспорины 1 поколения + аминогликозиды//
Фторхинолоны + аминогликозиды//
Оксациллин + метронидазол//
+ Цефтазидим + аминогликозиды//
Ванкомицин + аминогликозиды

***
Б. 40 лет без сопутствующих заболеваний по поводу внебольничной пневмонии в амбулаторных условиях был назначен спирамицин внутрь по 3 млн. МЕ 2 р\с, на 2-е сутки лечения отмечались интенсивные гастралгии, тошнота, однократная рвота. Выберите альтернативный препарат://


+ азитромицин//
кларитромицин//
цефтриаксон//
амикацин//
доксициклин

***
У Б.72л.с хроническим обструктивным бронхитом выявлена пневмония средней тяжести; в амбулаторных условиях назначен амоксиклав внутрь по 625 мг 3 р\с. На 2-е сутки у больного развилась крапивница, бронхоспазм. Назовите альтернативный препарат для лечения пневмонии://


цефуроксим внутрь//
бензилпенициллин в\м//
+ азитромицин внутрь//
моксифлоксацин внутрь//
ампициллин в\м

***
У ВИЧ-инфицированного пациента 44 лет диагностирована пневмоцистная пневмония. Назовите препарат для лечения://


цефепим в\в 2 г\с в течение 14 сут//
имепенем в\в по 0,5 г 4 р\с в течение 14 сут//
ванкомицин в\в по 1 г 2 р\с в течение 12 сут//
+ ко-тримоксазол в\в 20 мг\кг\с 4 р\с в течение 21 сут//
левофлоксацин в\в 1 г\с в течение 21 сут

***
У б-ной 28л. отмечаются ежедневные симптомы бронхиальной астмы, частые обострения, частые ночные симптомы, диагностирована тяжелая персистирующая бронхиальная астма. Назовите препараты базисной терапии://


ингаляционные глюкокортикоиды (меньше или равно 500 мкг беклометазона дипропионата)//
ингаляционные глюкокортикоиды (200-1000 мкг беклометазона дипропионата)//
+ингаляционные глюкокортикоиды (больше 1000 мкг беклометазона дипропионата) + ингаляционные бета-2-агонисты длительного действия//
теофиллины медленного высвобождения//
пероральные глюкокортикоиды

***
Б.57 лет по поводу артериальной гипертензии умеренной степени получает в монорежиме ингибитор АПФ – эналаприл. У больного через 2 года приема препарата отмечается недостаточность эффекта. Какой наиболее приемлемый вариант оптимизации терапии://


+добавление к препарату диуретика (гипотиазида или индапамида)//
отмена препарата, назначение лекарства из другого ряда//
повышение дозы препарата до максимальной//
временная отмена препарата//
нет верного ответа

***
Б.34л. получает антибактериальный препарат по поводу инфекционного процесса. При внутривенной инфузии препарата отмечается реакция в виде выраженного покраснения кожных покровов верхней половины туловища, лица, шеи; симптомы значительно уменьшаются при снижении скорости инфузии. На какой препарат отмечается такая реакция://


Цефтриаксон//
+ Ванкомицин//
Амикацин//
Ципрофлоксацин//
Азлоциллин

***
У беременной женщины отмечается активация ревматического процесса. Какой препарат из группы антикоагулянтов можно назначить беременной://


Фенилин//
Синкумар//
Варфарин//
+ Гепарин//
Нет верного ответа

***
Б.43г. поступил с ж-ми на повышение АД до цифр 150\90 мм.рт.ст. на фоне психоэмоционального перенапряжения, сердцебиение, тревогу, нарушение сна. Год назад выявлен сахарный диабет 2 типа, получает манинил. Назовите препарат выбора для лечения АГ://


Анаприлин//
Гипотиазид//
Атенолол//
+ Капотен//
Нифедипин

***
Б.55л. в связи с симптомами стенокардии и нарушения ритма назначены препараты: анаприлин 200 мг\с и верапамил 240 мг\с в течение длительного времени. Какие возможны побочные реакции://


+Резкое снижение АД//
Развитие AV блокад, брадикардия//
Развитие волчаночного синдрома//
Развитие тромбоза//
все ответы верны

***
Женщина 34 лет принимает эстрогенсодержащие контрацептивные препараты. Лечащий врач назначил доксициклин в дозе 200 мг\с в течение 2 недель. Какое вероятное взаимодействие ожидается://


Повышается риск тромбозов//
Снижается антибактериальный эффект доксициклина//
+ Снижается эффект контрацепции//
Возможно маточное кровотечение//
Нет верного ответа

***
Анестезиолог при введении кетамина с профилактической целью назначил больному диазепам. Профилактика какого состояния проводится таким образом://


+ Судорожных реакций//
Постнаркозных галлюцинаций//
Постнаркозной депрессии//
Гипоксии головного мозга//
Нет верного ответа

***
Б.46 лет поступил с картиной острого деструктивного аппендицита. Препарат выбора для антибиотикопрофилактики://


Цефтриаксон//
Пенициллин//
Цефазолин//
+ Метронидазол//
Нет верного ответа

***
К врачу обратился больной с жалобами на кашель, повышение температуры до 39 С, боли в грудной клетке. Диагностирована правостронняя бронхопневмония. Назначен препарат в течение 3 суток, обладающий постантибиотическим эффектом. Назовите препарат выбора://


Ампициллин//
Ципрофлоксацин//
+ Азитромицин//
Цефазолин//
Нет правильного ответа

***
Б.45л. по поводу острого правостороннего пиелонефрита назначен цефазолин по 2 г\с в течение 10 дней. Назовите наиболее типичную ошибку при выборе данного антибиотика://


Нефротоксичность//
+ Недостаточно высокая активность в отношении грамотрицательной флоры//
Способность вызывать снижение диуреза//
Узкий спектр действия//
Сложность мониторинга

***
Б.41г. одновременно получает в течение 14 дней фторхинолоновый антибиотик – офлоксацин по поводу инфекции мочевыводящих путей и диклофенак натрия в связи с суставным синдромом. Какое вероятное взаимодействие ожидается://


риск нефротоксичности//
повышение риска возбуждения ЦНС и развития судорог//
риск гепатотоксичности//
усиление ототоксичности//
+нет верного ответа

***
У Б.34л.после переохлаждения возникли озноб,повышение температуры тела до 38,6С, кашель с отделением слизисто-гнойной мокроты,боли в левой половине грудной клетки. Клинически и рентгенологически установлен диагноз левосторонней нижнедолевой пневмонии. Б-ной назначено лечение: цефазолин по 1 г 2р.в сутки в\м, гемодез 400 мл в/в капельно, отхаркивающая микстура по 1ст.л.6р.в сутки. На 3-е сутки отмечалась реакция в виде крапивницы, кожного зуда. Выберите антибактериальный препарат для замены://


+Спирамицин//
Ципрофлоксацин//
Цефтриаксон//
Нетилмицин//
Амоксиклав

***
При проведении наркоза анестезиолог с целью антибиотикопрофилактики назначил антибактериальный препарат. У больного развилась остановка дыхания. Какой препарат использовался://


Цефазолин//
Амоксиклав//
Цефуроксим//
+ Гентамицин//
Пефлоксацин

***
Б.по поводу нарушения ритма сердца назначен новокаинамид, одновременно в связи с сезонным аллергическим ринитом назначен цетиризин. Какое вероятное взаимодействие ожидается://


+развитие AV блокад//
развитие судорог//
остановка дыхания//
тяжелые формы аритмии (типа пируэта)//
развитие тромбозов

***
Б.с гипертоническим кризом был назначен препарат фозиноприл. Несмотря на прием адекватной дозы препарата не отмечено снижение уровня АД в ближайшие минуты и часы. Назовите причину://


При кризах не используются депо-препараты//
Не используются ингибиторы АПФ 2-3 поколения//
Должны использоваться ударные дозы//
+ Необходим сублингвальный прием//
Нет верного ответа

***
Б.43л. с целью плановой терапии АГ был назначен апрессин в течение длительного времени. У больного через месяц применения препарата стали отмечаться сердцебиение, стенокардитические боли, снижение эффекта от лечения. Назовите основную причину развившихся явлений://


Неправильный режим дозирования//
Апрессин не назначается для плановой терапии АГ//
Необходимы короткие курсы терапии//
+Необходимо сочетание с диуретиками//
Нет верного ответа

***
Б.42 лет поступил в отделение интенсивной терапии по поводу тяжелого гипертонического криза. В течение 5 суток внутривенно инфузионно вводился нитропруссид натрия. На 6-е сутки у больного развилась картина интоксикации в виде неукротимой рвоты, снижения функции сердечно-сосудистой, дыхательной и выделительной систем. Назовите основную причину развившегося состояния://


Лекарственное взаимодействие с другими препаратами//
+ Передозировка препарата (накопление тиоцианатов в крови)//
Естественное течение заболевания//
Неправильная диагностика//
Нет верного ответа

***
Б.54 лет страдает сахарным диабетом 1 типа, принимает пролонгированный инсулин. Поступила в отделение с картиной острой правосторонней нижнедолевой пневмонии, подтвержденной рентгенологически. Был назначен цефтриаксон,на который у б-ной отмечалась аллергическая реакция. Препарат отменили, выбран другой антибиотик- ципрофлоксацин в сочетании с амикацином. Однако при обследовании у б-ной выявлен низкий уровень клиренса креатинина (30 мл/мин), вследствие чего амикацин был отменен. Каким препаратом следует продолжить лечение://


Пефлоксацин//
Спирамицин//
Имипенем//
+ Азитромицин//
Левомицетин

***
Б.23г.поступила в отделение с ж-ми на боли в низу живота, повышение температуры тела до 39,5 С. Заболела 2 дня назад, на 6-й день после родов. При гинекологическом осмотре – картина острого послеродового эндометрита. Результаты бактериологического посева мазка из цервикального канала: золотистый стафилококк, образующий пенициллиназу, протей. Определите антибиотик первого выбора://


+Цефепим//
Бензилпенициллин//
Ампициллин//
Гентамицин//
Нет верного ответа

***
Б.25 л.поступила в отделение с картиной острого правостороннего пиелонефрита.Заболела 3 дня назад после переохлаждения. Был назначен цефазолин. После 2-й инъекции препарата через 10 мин появились снижение АД, головокружение, тошнота, рвота, непроизвольное мочеиспускание, судорожный синдром. Какое осложнение развилось у пациентки://


Эндотоксиновый шок//
Признаки дальнейшего развития клинической картины//
+ Анафилактическая реакция//
Симптомы острого живота//
Токсическое действие препарата

***
Б-ная 28 л.поступила в отделение с картиной острого правостороннего мастита. Заболела на 12-й день после родов. Б-ная оперирована. На цефазолин у б-ной отмечалась анафилактическая реакция, препарат был немедленно отменен. При посеве отделяемого из раны выделены стафилококк, образующий пенициллиназу, и кандиды. Выберите антибактериальный препарат с учетом бактериальной микрофлоры и особенностей фармакокинетики://


Цефуроксим//
Линкомицин//
+ Оксациллин + флуконазол//
Офлоксацин//
Метронидазол

***
Б.42л.страдает хроническим холециститом. В посеве желчи при обследовании выявлены золотистый стафилококк и кишечная палочка. В анамнезе отмечена аллергия на оксациллин. Препараты выбора://


Ампициллин//
Цефазолин//
Цефтриаксон//
+ Ципрофлоксацин//
Нет верного ответа

***
Б.58 лет обратилась с жалобами на общую слабость, жажду, частое мочеиспускание, зуд кожи и наружных половых органов. При осмотре: масса тела 56 кг при росте 168 см. Содержание глюкозы в крови 12,3 ммоль/л, в моче 1,5%, реакция на ацетон отрицательная. Какие гипогликемические препараты оптимальны в данном случае://


+ Бигуаниды//
Пролонгированные препараты инсулина//
Препараты сульфонилмочевины//
Инсулин простой//
Нет верного ответа

***
Б.53 лет, поступил с жалобами на сердцебиение, перебои, временами одышку. Эти явления стали беспокоить после перенесенного инфаркта миокарда 2 года назад. Прием новокаинамида в течение 3 месяцев принес значительное облегчение. Однако в последнее время самочувствие ухудшилось. Дальнейшая тактика ведения больного://


отмена препарата//
+ полное обследование и выбор препарата//
назначение сердечных гликозидов//
назначение диуретиков//
нет верного ответа

***
Б.33 лет по поводу послеоперационного гнойного перитонита получает комбинацию препаратов: цефтриаксон + амикацин + метронидазол. В анамнезе – желчнокаменная болезнь. Назовите препарат, который не должен назначаться больному://


Амикацин//
Цефтриаксон//
Метронидазол//
+ Амикацин в сочетании с метронидазолом//
нет верного ответа

***
Б.45 лет по поводу кандидозной пневмонии был назначен флуконазол в\в в течение 3 дней, затем внутрь. На 4-е сутки лечения отмечена реакция в виде сильных головных болей, тошноты. Решено было заменить препарат на кетоконазол. Оцените адекватность тактики://


Кетоконазол не является адекватной заменой в силу невыгодных фармакокинетических характеристик//
интраконазол//
Временная отмена препарата//
тербинафин//
+нет верного ответа

***
Б.42 лет, поступил с жалобами на выраженное сердцебиение, повышение цифр АД до 240\140 мм.рт.ст. При обследовании выявлено значительное повышение в крови уровня катехоламинов. Назовите препарат выбора для купирования криза://


Фуросемид в\в//
Пентамин//
Сернокислая магнезия//
+ фентоламин//
нет верного ответа

***
Б.50 лет, поступил с картиной острого левостороннего пиелонефрита. Были назначены цефазолин + гентамицин в среднетерапевтических дозах. У больного при обследовании клиренс креатинина – 50 мл\мин. Какие вероятные последствия терапии://


+ Риск нефротоксичности//
Развитие ХПН//
Развитие отечного синдрома//
Кровотечение//
нет верного ответа

***
Б.48 лет с пароксизмальной над- и желудочковой тахикардией был назначен кордарон в\в в первые сутки, затем внутрь. При обследовании выявлено нарушение функции щитовидной железы, AV блокада 2-3 степени. Дальнейшая тактика://


+отмена препарата, назначение новокаинамида//
назначение верапамила//
назначение сердечных гликозидов//
назначение анаприлина//
нет верного ответа

***
Б.50 лет находится в отделении кардиореанимации по поводу острого инфаркта миокарда, получает комплекс терапии. Какие основные параметры мониторинга при назначении прямых антикоагулянтов://


Протромбиновое время, тромбоциты//
+ АЧТВ, время свертывания крови, моча на эритроциты//
Коагулограмма, гематокрит//
определение факторов свертывания крови//
нет верного ответа

***
В отделении находится больной после операции по поводу аппендицита. Лечащий врач назначил гентамицин по 80 мг 3 р\с в\м. Пациент 2 года назад перенес острый гломерулонефрит, в настоящее время клиренс креатинина составляет 50 мл\мин. Какая коррекция требуется://


Отмена препарата//
Снижение доз препарата и введение его 1 раз в сутки//
+ замена на препарат, не обладающий нефротоксичностью//
замена на нетилмицин//
нет верного ответа

***
Б.56 лет, получает дигоксин по 0,25 г\с в течение последнего года. В настоящее время стало отмечаться повышение цифр АД до 180\110 мм.рт.ст. Лечащим врачом назначен лизиноприл в дозе 10 мг\с. Через 3 месяца у пациента диагностирована дигиталисная интоксикация. Дальнейшая тактика://


Отмена обоих препаратов//
+ отмена только дигоксина//
назначение гипотензивного препарата другого ряда//
назначение диуретиков//
нет верного ответа

***
У Б.53 лет диагностирован кандидозный и аспергиллезный менингит. Препараты выбора://


+Флуконазол//
Амфотерицин В//
Интраконазол//
Тербинафин//
нет верного ответа

***
У Б.58 лет, получающего длительное время глюкокортикоиды в комплекс терапии был включен синтетический антибиотик широкого спектра в связи с инфекцией репродуктивной сферы. На 14 сутки совместного применения отмечено тяжелое осложнение в виде разрыва ахиллова сухожилия. Назовите антибиотик, который в сочетании с ГКС вызвал данное осложнение://


Левомицетин//
+Левофлоксацин//
Амикацин//
Ванкомицин//
нет верного ответа

***
В отделение поступил Б.43 лет с картиной острой хламидийной пневмонии. Назовите препараты выбора://


Цефазолин//
Ровамицин//
+Клиндамицин//
Амоксиклав//
Пефлоксацин

***
У больного 24 лет диагностирована неосложненная форма гонореи. Назовите препарат выбора://


Цефтриаксон//
Бициллин -5//
+ Гентамицин//
Линкомицин//
нет верного ответа

***
56 летний мужчина с возвратной желудочковой аритмией 5 месяцев назад самостоятельно прекратил прием антиаритмического препарата. В данный момент у больного усилилась одышка, кашель, отмечено повышение температуры тела. СОЭ повышена. На обзорной R-грамме грудной клетки – интерстициальная пневмония. Функциональные легочные тесты: повышено содержание СО. Отметьте препарат, вызывающий подобные изменения://


+ Амиодарон//
Пропранолол//
Прокамиамид//
Хинидин//
Верапамил

***
62-летний мужчина с обострением хронического заболевания легких начал прием гипотензивных препаратов. 2 недели спустя он отметил усиление одышки, появление свистящих хрипов. Это побочное действие характерно для://


Амиодарон//
+ Пропранолол//
Прокамиамид//
Хинидин//
Верапамил

***
В приемном покое больной, который не знает какой именно сердечный препарат он принимает. ЧСС выше 80 в мин. На ЭКГ интервал PQ и комплекс QRS удлинены. Какой препарат возможно принимает больной://


Лидокаин//
Амиодарон//
Пропранолол//
+ Хинидин//
Новокаинамид

***
Препарат выбора при первичной легочной гипертензии://


Гипотиазид//
+ Нифедипин//
Пропранолол//
Клофелин//
Допегит

***
Антибактериальная тактика у больного с абсцедирующей пневмонией://


Аминопеницилины в больших дозах в/м//
Аминогликозиды в/м//
Фторхинолоны внутрь//
Метронидазол и макролиды в/в//
+ Аминогликозиды и цефалоспорины II-IIIпоколения в/в.

***
Базисная терапия при бронхиальной астме://


Бета-2-адреностимуляторы//
Метилксантины//
лекарственнык средства, разжижающие мокроту//
+ глюкокортикоидные гормоны//
холиноблокаторы

***
Биохимичекие показатели, на которые должен обратить внимание врач при лечении ИАПФ://


Глюкоза //
Билирубин//
Общийбелок//
+ Креатинин//
Мочевая кислота

***
Серьезное осложнение терапии хронической сердечной недостаточности ингибиторами АПФ://


Сухой кашель//
+ Постуральная гипотензия//
Ангионевротический отек//
Гиперкалиемия//
Аллергия

***
Потенциальное осложнение терапии петлевыми диуретиками при хронической сердечной недостаточности://


+ Гипогликемия//
Гипермагнелия//
Гипокалиемия//
Гипонатриемия//
Гипоуриремия

***
Какое осложнение развивается после приема первой дозы альфа-блокатора://


Тахикардия//
Одышка//
Задержка мочи//
+ Постуральная гипотензия//
Эзофагиальный рефлюкс

***
Лучшие кандидаты для лечения бета-блокаторами://


+ Больные с гиперплазией предстательной железы//
Больные мигренью//
Больные с застойной сердечной недастаточнойтью//
Больные со стенокардией//
Больные с гипертрофической кардиомиопатией

***
Для купирования криза при феохромоцитоме в первую очередь надо использовать://


Нифедипин//
Пропранолол//
Нитропруссид//
+Фентоламин//
Празозин

***
Какой антиаритмический препарат надо назначить больному с гипертрофической кардиомиопатией и желудочковой экстрасистолией://


+Пропранолол//
Кордарон//
Пропафенон//
Хинидин//
Лидокаин

***
Какой препарат усиливает проявления сердечной недостаточности://


Лидокаин//
Кордарон//
+Верапамин//
Бретилия тозилат//
каптоприл

***
Какие метаболические нарушения наблюдаются при приеме тиазидных диуретиков://


+Гипокалиемия//
Гипогликемия//
Гипоурикемия//
Гипернатриемия//
Гипермагниемия

***
Укажите кардиоселективный бета-блокатор://


Пропранолол (обзидан, анаприлин)//
+Метопролол (корвитол)//
Пиндолол (вискин)//
Окспренолол (тразикор)//
Соталол (соталекс)

***
Пациент 28 лет в течение полугода отмечает подъем АД. При осмотре выявлена ретинопатия высокой степени, протеинурия 3 г/сут. С какого препарата необходимо начать лечение://


Пропранолол//
Гипотеиазид//
Каптоприл//
+Допегит//
Резерпин

***
Выберите препарат для лечения вазоренальной гипертензии://


Пропранолол//
Празозин//
Фенталамин//
Каптоприл//
+Нифедипин

***
44-летный летчик жалуется на повышение АД до 180/90 мм рт ст. Какой препарат не следует назначать данному пациенту://


+Клофелин//
Пропранолол//
Празозин//
Каптоприл//
Нифедипин

***
У 60-летнего больного на фоне повашения АД проявления отека легких: ЧСС – 100 в минуту, АД – 180/100 мм рт ст. Какой препарат надо назначить больному://


Клофелин//
Пропранолол//
Празозин//
+Нитроглицерин//
Нифедипин

***
У больного 63 лет на фоне атеросклеротического кардиосклероза проявления ХСН IIБ стадии, тахисистолической формы мерцательной аритмии://


+Дигоксин//
Пропранолол//
Верапамил//
Празозин//
Нифедипин

***
Какой препарат улучшает сократительную функцию миокарда://


Триметазидин (предуктал)//
Рибоксин//
+Допамин//
Неотон//
бисопролол

***
После приема салуретиков наблюдаются все побочные эффекты, кроме://


Гипокалиемии//
+Лихорадки//
Экстраcистолии//
Слабости//
Судорог
***
При лечении острого инфаркта миокарда прямыми антикоагулянтами (гепарином) обязательно надо следить за://
Временем свертывания крови//
Протромбиновым индексом//
временем рекальцификации плазмы//
уровнем фибриноген//
+АЧТВ

***
При стенокардии необходимо улучшать кровообращение в миокарде и снижать потребность миокарда в кислороде. Какой ихз указанных ниже препаратов не обладает этими свойствами://


A Нитроглицерин//
Кардикет//
Анаприлин//
+Валидол//
Нифедипин

***
Абсолютным противопоказанием к назначению каптоприла является://


+Двухсторонний стеноз почечных артерий//
Повышение уровня креатинина в крови выше 2 мг%//
Умеренное повышение АД//
Снижение уровня ренина в плазме крови//
Сахарный диабет

***
b- адреноблокаторы показаны при сочетании артериальной гипертонии и ://


+ИБС//
синдрома Рейно//
Облитерирующего эндоартерита//
Сахарного диабета//
Бронхиальной астмы

***
У 46-летнего больного артериальная гипертония и стенокардия напряжения. При осмотре: Кожные покровы обычной окраски, в легких везикулярное дыхание, хрипов нет. Тоны сердца приглушены, шумов нет, АД – 180/100 мм мм рт ст, ЧСС- 86 в мин. На ЭКГ- синусовый ритм, Редкая желудочковая экстрасистолия. Какой препарат следует выбрать://


Клофелин и противоаритмическое ЛС//
Диуретик//
+b- адреноблокатор//
нитраты и противоаритмическое ЛС//
антигонист кальция

***
Выберите рациональную комбинацию для лечения артериальной гипертонии://


Анаприлин + нифедипин//
Анаприлин + гипотиазид//
Каптоприл + гипотиазид//
+Анаприлин + верапамил//
Верапамил + гипотиазид

***
Выберите препарат, не относящийся к первой линии терапии АГ://


Диуретик//
b- адреноблокатор//
Антагонист кальция//
ИАПФ//
+Центральный a-адреномиметик

***
Какой препарат усиливает активность ренина://


Клофелин//
Фуросемид//
+Резерпин//
Пропранолол//
Лизиноприл

***
Препарат выбора для лечения АГ при феохромоцитоме://


Пропранолол//
+Фентоламин//
Миноксонидин//
Фуросемид//
Резерпин

***
При обострении язвенной болезни больному с АГ препаратом выбора будет://


+Амлодипин//
Адельфан//
Резерпин//
Кристепин//
Бринердин

***
При хроническом энтерите какую группу препаратов назначать не следует://


Ферменты//
Обвалакивающие средства//
Витамины//
+Антибиотики//
Биологические препараты (лактобактерин и др..)

***
Препараты какой группы увеличивают AV проводимость, усиливают гликогенолиз, понижают тонус бронхов://


М-холиноблокаторы//
Симпатолитики//
+a-b- адреномиметики//
a- адреномиметики//
ганглиоблокаторы

***
Отметьте побочные действия, ограничивающие прием адреноблокаторов://


b-адреноблокаторы увеличивают внутриглазное давление//
a-адреноблокаторы вызывают атриовентрикулярную блокаду//
a-адреноблокаторы вызывают нарушение периферического кровообращения//
+b-адреноблокаторы вызывают бронхспазм//
b-адреноблокаторы вызвают тахикардию

***
Найдите алколоид, влияющий на центральную и периферическую нервную систему. Ему свойственны седативное и гипотензивное действие, может вызвать депрессию://


Атропин//
Пахикарпин//
Платифилин//
+Резерпин//
Физостигмин

***
Какой из указанных ниже препаратов вызывает индукцию микросомальных ферментов печени://


+Фенобарбитал//
Морфин//
Аминазин//
Фторотан//

Сибазон


***
Как можно отличить кокаиниста от морфиниста://


По уровню артериального давления//
По частоте пульса//
+По величине зрачка//
По частоте дыхания//
По тонусу скелетной мускулатуры

***
У больного с первоначально сниженной функцией внешнего дыхания после ингаляции 200 мкг сальбутамола отмечено увеличение ФВД на 20%. Что это означает://


гиперреактивность бронхов//
рестриктивные изменения в легких//
+обратимую обструкцию бронхов//
необратимую обструкцию бронхов//
отсутствие изменений

***
У 67-летнего больного на фоне постинфарктного кардиосклероза, желудочковой экстрасистолии имеется обострение бронхиальной астмы средней степени тяжести. Каким аэрозолем следует купировать приступ экспиратоной одышки://


Изадрин//
Астмопент//
Бекотид//
Интал//
+Сальбутамол

***
Отметьте длительно действующий селективный агонист b2 – адренорецептора://


Изадрин//
Сальбутамол//
+Серевент (сальметерол)//
Интал//
Атровент

***
Укажите кортко действующий агонист b2 – адренорецептора://


Астмопент//
+Сальбутамол//
Серевент//
Интал//
Атровент

***
Какое лечение следует выбрать пациенту с интермиттирующим течением бронхиальной астмы://


Ингаляции короткодействующих 2- симпатомиметика в плановом порядке//
Средние дозы ингаляционных кортикостероидов в плановом порядке (беклометазон 400-600 мкг/ сут)//
Ингаляции интала в плановом порядке//
Теофилин внутрь в плановом порядке//
+В случае необходимости ингаляции короткодействующих b2-симпатомиметиков, в плановом порядке терапия не требуется

***
Выберите лучший гипотензивный препарат для в/в введения://


Рауседил//
Клофелин//
Лазикс//
Дибазол//
+Диазоксид

***
У больного 45 лет – гипертоническая болезнь II стадия гемодинамического, гиперкинетического типа. С какого антигипертензивного препарата следует начать терапию://


АПФ ингибиторы//
+b-адреноблокаторы//
кальций антагонисты//
диуретики//
a- адреноблокаторы

***
Терапия выбора при реноваскулярной АГ://


Кальций антагонисты//
b-адреноблокаторы//
центральные a- антагонисты//
гидралазин//
+хирургическое лечение

***
Выберите комбинацию препаратов в следующей ситуации: больной 38 лет с диагнозом: Острый инфаркт миокарда, осложненный кардиогенным шоком І стадии. При осмотре: синусовая тахикардия 110 в мин, систолический шум на верхушке, АД – 80/40 мм рт ст.://


в/в b-блокатор + диуретик//
АПФ – ингибитор + диуретик//
в/в сердечный гликозид + диуретик//
в/в Нитроглицерин + диуретик//
+в/в допамин + нитроглицерин

***
Перечисленные побочные эффекты (снижение сократимости миокарда, брадикардия, гипотензия, бронхообструктивный синдром) для какого препарата характерны://


Нитроглицерин//
Дипиридамол//
Верапамил//
+Анаприлин//
Нифедипин

***
При патологии печени больному можно назначить все перечисленные сердечные гликозиды, КРОМЕ://


строфантин//
коргликон//
+дигитоксин//
дигоксин//
изоланид

***
При микробиологическом исследовании мокроты больного острой пневмонией выделена Klebsiella pneumoniaе в титре 108. Определите тактику антибактериальной терапии://


цефалоспорины второго поколения//
полусинтетические пенициллины в больших дозах//
+ цефалоспорины второго или третьего поколения в сочетании с аминогликозидами//
фторхинолоны//
макролиды

***
Хронический бронхит следует лечить антибиотиками://


в осенне-зимний период//
длительно//
не следует применять вообще//
+при выделении гнойной мокроты//
при появлении кровохарканья

***
Основу долгосрочного лечения персистирующей бронхиальной астмы составляют://


ингаляционные симпатомиметики короткого действия//
ингаляционные симпатомиметики длительного действия//
+ ингаляционные кортикостероиды//
блокаторы Н1-рецепторов гистамина//
м-хинолитики

***
Основным возбудителем внебольничных пневмоний является://


+пневмококк//
стафилококк//
гемофильная палочка//
синегнойная палочка//
легионелла

***
У больного 35 лет имеют место нечастые (реже 1 раза в неделю) приступы экспираторного удушья, легко купируемые ингаляцией β2-симпатомиметиков короткого действия. Во время приступов в легких выслушиваются сухие свистящие хрипы, в промежутке между приступами ОФВ1 более 80% от должного. Какое лечение необходимо назначить данному больному://


регулярное применение ингаляционных β2-симпатомиметиков длительного действия//
регулярное применение ингаляционных кортикостероидов в средних дозах (беклометазон 400-600 мкг/сутки)//
регулярное применение динатрия кромогликата//
регулярное применение препаратов теофиллина внутрь//
+применение β2-симпатомиметиков короткого действия по требованию

***
Прием следующих препаратов может вызвать язвенное поражение слизистой оболочки желудочно-кишечного тракта://


+ преднизолон//
метронидазол//
вентер//
кардиомагнил//
сукральфат

***
Какой из перечисленных препаратов следует назначить больному хроническим гастритом с повышенной секреторной функцией желудка://


бетацид//
панзинорм//
+ ранитидин//
преднизолон//
натуральный желудочный сок

***
Какое средство обладает цитопротективным действием при лечении язвенной болезни://


облепиховое масло//
викалин//
платифиллин//
+ сукральфат//
солкосерил

***
Какой препарат наиболее эффективен при лечении язвенной болезни, обусловленной инфицированием Helicobacter pylori://


гастроцепин//
альмагель//
+ де-нол//
платифиллин//
маалокс

***
В какое время следует принимать блокаторы Н2-рецепторов гистамина://


через 1-2 ч после еды//
во время еды//
+ преимущественно на ночь//
за 1 час до еды//
все перечисленное неверно

***
При лечении хронического гастрита типа «В» основными принципами являются://


назначение блокаторов Н2-рецепторов гистамина//
+ликвидация Helicobacter pylori//
назначение ферментных препаратов//
назначение антацидов//
все перечисленное верно

***
К ремиссии НЯК приводит назначение://


левомицетина//
бисептола//
фестала//
+сульфасалазина//
Пробиотики

***
В качестве обезбаливающих средств при хроническом панкреатите можно применять все перечисленные средства, кроме одного://


новокаин//
фентанил//
баралгин//
+ морфин//
анальгин

***
При терапии обострений хронического панкретатита можно применять все перечисленные способы, кроме одного://


альмагель//
голодание//
+тепловые процедуры//
новокаин//
трасисол

***
В первые дни выраженного обострения хронического панкреатита используют все перечисленные средства, кроме одного://


пищеварительные панкреатические ферменты//
+диета, механически и химически щадящая, с резким ограничением жиров и
высоким содержанием белка//
постоянное откачивание желудочного содержимого через назальный зонд//
ингибиторы протеолитических ферментов//
блокаторы Н2-рецепторов гистамина

***
Единственным препаратом, доказавшим свою эффективность при лечении хронических гепатитов С, является://


эссенциале форте//
сандостатин//
преднизолон//
+ α-Интерферон//
гемодез

***
Следующие утверждения справедливы в отношении инфекционного эндокардита, кроме одного://


средний срок антибиотикотерапии – 3-6 нед.//
важным мероприятием по профилактике инфекционного эндокардита является тщательная гигиена полости рта//
кортикостероиды назначаю при наличии иммунных осложнений//
+ лечение начинается с перорального назначения антибиотиков//
у больных с пороками сердца антибиотики необходимо назначать профилактически при стоматологических операциях

***
Для уменьшения боли при остром перикардите средствами выбора являются://


ненаркотические и наркотические анальгетики//
нитроглицерин и нитраты пролонгированного действия//
спазмолитики//
+нестероидные противовоспалительные препараты//
цитостатики

***
При недостаточности кровообращения у больных дилатационой КМП наиболее


эффективны://
оксигенотерапия//
изадрин//
+ периферические вазодилататоры//
препараты калия//
АТФ и кокарбоксилаза

***
Рациональными комбинациями для лечения АГ являются все перечисленные, кроме одной://


β-адреноблокаторы и антагонисты кальция дигидропиридонового ряда//
β-адреноблокаторы и диуретики//
ингибиторы АПФ и диуретики//
+ β-адреноблокаторы и антагонисты Са негидропиридонового ряда//
Антагонисты кальция и диуретики

***
Какой препарат применяют в качестве патогенетического средства при синдроме Конна://


+верошпирон//
фуросемид//
анаприлин//
клофелин//
нифедипин

***
В качестве дезагреганта при лечении заболеваний почек используются все перечисленные препараты, кроме одного://


тиклопидин//
дипиридамол//
ацетилалициловая кислота//
+эпсилон-аминокапроновая кислота//
трентал

***
В так называемую 4-компонентную схему лечения заболеваний почек входя следующие препараты, кроме одного://


преднизолон //
+делагил //
гепарин //
дипиридамол //
циклофосфан

***
Лечение больных с ОГН обязательно предусматривает://


глюкокортикоиды//
цитостатики//
антибактериальную терапию//
+ гипохлоридную диету//
иммуномодуляторы

***
Наиболее эффективными методами лечения амилоидоза являются://


Диуретики //
Глюкокортикоиды //
+Аминохинолиновые препараты //
Иммуносупрессоры (цитостатики) //
НПВС

***
При мегалобластической анемии показаны://


длительное применение препаратов железа//
курсовое название пиридоксина//
длительное применение анаболических стероидов//
+применение цианкобаламина в инъекциях//
повторные трансфузии плазмы

***
Гемолитическую анемию могут вызвать все лекарственные препараты, кроме://


цефалоспорины//
туберкулостатик//
сульфаниламиды//
нитрофураны//
+ преднизолон

***
Каков механизм воздействия аспирина на гемокоагуляцию://


снижает содержание тромбина//
ингибирует образование фибрина//
+ снижает агрегацию тромбоцитов//
повышает содержание антитомбина III//
снижает содержание протромбина

***
Каков механизм действия прямых антикоагулянтов://


инактивирует тканевый тромбопластин//
растворяет фибрин//
тормозит переход фибриногена в фибрин//
+инактивирует тромбин//
ингибирует образование Х фактора свертывания крови

***
Каков механизм действия непрямых антикоагулянтов: //


препятствуют агрегации тромбоцитов//
усиливают тромболитическую антивность крови//
снижают содержание фибриногена//
+подавляют синтез белков протромбинового комплекса//
активируют тканевый тромбопластин

***
Каков механизм действия стрептокиназы://


подавляет синтез фибриногена//
снижает агрегацию тромбоцитов//
снижает синтез протромбина //
+ активирует плазминоген//
непосредственно разрушает фибрин

***
Каков механизм фибринолитического действия урокиназы://


+ активирует плазминоген//
подавляет выработку фибриногена//
подавляет выработку белков протромбинового комплекса//
усиливает распад фибриногена//
снижает содержание тромбоцитов

***
Какой тип кровоточивости характерен для гемофилии://


петехально-пятнистый//
васкулитно-пурпурный//
+гематомный//
ангиоматозный//
смешанный

***
Какой препарат является антагонистом непрямых антикоагулянтов://


Ацетилсалициловая кислота//
Эпсилон-аимнокапроновая кислота//
+Викасол//
Преднизолон//
Хлористый кальций

***
Средства выбора при лечении болезни Вегенера://


кортикостероиды//
аминохинолиновые производные//
метатрексат//
+циклофосфан//
ни одно из перечисленных

***
При остром течении узелкового периартериита следует назначать://


антибиотики//
преднизолон//
+преднизолон и циклофосфамид//
делагил//
ни один из перечисленных препаратов

***
Полифармация (полипрагмазия) – это://


одновременное использование слишком большого числа ЛС (зачастую необоснованно) у одного больного при лечении основного заболевания//
одновременное использование слишком большого числа ЛС (зачастую необоснованно) у одного больного при лечении сопутствующего заболевания//
одновременное использование слишком большого числа ЛС (зачастую необоснованно) у группы больных//
одновременное использование слишком большого числа ЛС (зачастую необоснованно) у одного больного с учетом физиологического состояния (беременность, возраст)//
+одновременное использование слишком большого числа ЛС (зачастую необоснованно) у одного больного

***
Выбрать правильное определение «доказательной медицины»://


+«интеграция лучших исследовательских данных с клиническим опытом и потребностям пациента»//
«интеграция лучших доклинических исследовательских данных в клиническую практику»//
«интеграция мнений ведущих специалистов медицины в клиническую практику»//
«рациональное использование лекарственных средств с учетом личного опыта врача»//
«информационно-методическая доктрина, направленная на отбор основных ЛС»

***
Ранжировать типы испытаний по силе их доказательности://


Описательные исследования; РКИ; Контролируемые исследования без рандомизации; Когортные испытания; Неконтролируемые исследования//
Контролируемые исследования без рандомизации; РКИ; Описательные исследования; Когортные испытания; Неконтролируемые исследования//
+РКИ; Контролируемые исследования без рандомизации; Когортные испытания; Неконтролируемые исследования; Описательные исследования//
Неконтролируемые исследования; Описательные исследования; РКИ; Контролируемые исследования без рандомизации; Когортные испытания//
Когортные испытания; РКИ; Контролируемые исследования без рандомизации; Неконтролируемые исследования; Описательные исследования.

***
Мета-анализ – это://


статистическая обработка одного научного исследования.//
количественный анализ объединенных результатов нескольких клинических испытаний различных вмешательств//
оценка эффективности и безопасности лекарственной терапии//
+количественный анализ объединенных результатов нескольких клинических испытаний одного и того же вмешательства//
проведение клинических исследований с последующей количественной обработкой полученных результатов.

***
Категории доказательства - А основаны на://


РКИ. Доказательства ограниченны, т.к. в конечной части исследования было недостаточно количество больных. Рекомендации могут быть расспространены на ограниченную популяцию//
результатах нерандомизированных исследований//
проведенной экспертами дискуссии, в результате которой был достигнут консенсус//
принятых в ЛПУ подходах к фармакотерапии//
+законченных и хорошо спланированных РКИ, использован совершенный математический аппарат (мета-анализ). Они позволяют делать рекомендации для их использования в определенной популяции.
***
Категории доказательства - В основаны на://
+РКИ. Доказательства ограниченны, т.к. в конечной части исследования было недостаточно количество больных. Рекомендации могут быть распространены на ограниченную популяцию//
результатах нерандомизированных исследований//
проведенной экспертами дискуссии, в результате которой был достигнут консенсус//
принятых в ЛПУ подходах к фармакотерапии//
законченных и хорошо спланированных РКИ, использован совершенный математический аппарат (мета-анализ). Они позволяют делать рекомендации для их использования в определенной популяции.

***
Категории доказательства - С основаны на://


РКИ. Доказательства ограниченны, т.к. в конечной части исследования было недостаточно количество больных. Рекомендации могут быть распространены на ограниченную популяцию.//
+результатах нерандомизированных исследований//
проведенной экспертами дискуссии, в результате которой был достигнут консенсус//
принятых в ЛПУ подходах к фармакотерапии//
законченных и хорошо спланированных РКИ, использован совершенный математический аппарат (мета-анализ). Они позволяют делать рекомендации для их использования в определенной популяции.

***
Категории доказательства – D основаны на://


РКИ. Доказательства ограниченны, т.к. в конечной части исследования было недостаточно количество больных. Рекомендации могут быть распространены на ограниченную популяцию.//
результатах нерандомизированных исследований//
+проведенной экспертами дискуссии, в результате которой был достигнут консенсус//
принятых в ЛПУ подходах к фармакотерапии//
законченных и хорошо спланированных РКИ, использован совершенный математический аппарат (мета-анализ). Они позволяют делать рекомендации для их использования в определенной популяции.

***
Современные клинические исследования предусматривают следующий дизайн испытаний://


+сравнительные, рандомизированные, двойные слепые и проспективные//
несравнительные, рандомизированные, двойные слепые и проспективные//
сравнительные, нерандомизированные, двойные слепые и проспективные//
сравнительные, рандомизированные, ретророспективные//
сравнительные, нерандомизированные, проспективные

***


При открытом клиническом исследовании://
+врач и пациент знают, какая терапия назначена//
пациент, в отличие от исследователя, не знает, какое лечение ему назначено//
ни врач, ни пациент не знают назначенной терапии//
ни врач, ни пациент, ни исследователь не знают назначенной терапии//
врач, пациент, исследователь знают назначенную терапию

***
При простом слепом клиническом исследовании://


и врач, и пациент знают, какая терапия назначена//
+пациент, в отличие от исследователя, не знает, какое лечение ему назначено//
ни врач, ни пациент не знают назначенной терапии//
ни врач, ни пациент, ни исследователь не знают назначенной терапии//
врач, пациент, исследователь знают назначенную терапию

***
При двойном слепом клиническом исследовании://


и врач, и пациент знают, какая терапия назначена//
пациент, в отличие от исследователя, не знает, какое лечение ему назначено//
+ни врач, ни пациент не знают назначенной терапии//
ни врач, ни пациент, ни исследователь не знают назначенной терапии//
врач, пациент, исследователь знают назначенную терапию

***
Задачи первой фазы клинических исследований://


+определение спектра фармакологических действий, производимого разными дозами препарата и его фармакокинетические характеристики//
подтверждение терапевтических эффектов, подбор эффективной дозы и кратности приема препарата, изучении безопасности и переносимости//
оценка эффективности и безопасности, изучение терапевтических преимуществ, фармакоэкономики и влияния изучаемого средства на качество жизни, лекарственных взаимодействий с регистрацией нежелательных реакций, возникающие с частотой более 1% на большой группе пациентов//
дополнительная оценка безопасности и эффективности у большого количества людей. Обнаружение неописанных нежелательных побочных реакций//
проведение ретроспективного анализа эффективности препарата

***
Задачи второй фазы клинических исследований://


определение спектра фармакологических действий, производимого разными дозами препарата и его фармакокинетические характеристики//
+подтверждение терапевтических эффектов, подбор эффективной дозы и кратности приема препарата, изучении безопасности и переносимости//
оценка эффективности и безопасности, изучение терапевтических преимуществ, фармакоэкономики и влияния изучаемого средства на качество жизни, лекарственных взаимодействий с регистрацией нежелательных реакций, возникающие с частотой более 1% на большой группе пациентов//
дополнительная оценка безопасности и эффективности у большого количества людей. Обнаружение неописанных нежелательных побочных реакций//
проведение ретроспективного анализа эффективности препарата.

***
Задачи третьей фазы клинических исследований://


определение спектра фармакологических действий, производимого разными дозами препарата и его фармакокинетические характеристики//
подтверждение терапевтических эффектов, подбор эффективной дозы и кратности приема препарата, изучении безопасности и переносимости//
+оценка эффективности и безопасности, изучение терапевтических преимуществ, фармакоэкономики и влияния изучаемого средства на качество жизни, лекарственных взаимодействий с регистрацией нежелательных реакций, возникающие с частотой более 1% на большой группе пациентов//
дополнительная оценка безопасности и эффективности у большого количества людей. Обнаружение неописанных нежелательных побочных реакций//
проведение ретроспективного анализа эффективности препарата

***
Задачи четвертой фазы клинических исследований://


определение спектра фармакологических действий, производимого разными дозами препарата и его фармакокинетические характеристики//
подтверждение терапевтических эффектов, подбор эффективной дозы и кратности приема препарата, изучении безопасности и переносимости//
оценка эффективности и безопасности, изучение терапевтических преимуществ, фармакоэкономики и влияния изучаемого средства на качество жизни, лекарственных взаимодействий с регистрацией нежелательных реакций, возникающие с частотой более 1% на большой группе пациентов//
+дополнительная оценка безопасности и эффективности у большого количества людей. Обнаружение неописанных нежелательных побочных реакций//
проведение ретроспективного анализа эффективности препарата

***
Фармакоэпидемиология – это наука://


изучающая эффекты ЛС на экспериментальных животных//
изучающая применение ЛС и их эффекты на ограниченном контингенте людей//
изучающая применение ЛС и их эффекты на большой популяции или больших группах здоровых людей//
+изучающая применение ЛС и их эффекты на большой популяции или больших группах людей//
изучающая только фармакоэкономические аспекты использования ЛС

***
Указать тип фармакоэкономического анализа, который позволяет определить и оценить реальную стоимость терапии конкретного заболевания://


+Анализ «стоимости болезни» (COI – cost of illness)//
Анализ «минимизации затрат» (CMA – cost-minimization analysis)//
Анализ «затраты – эффективность» (CEA – cost-effectiveness analysis)//
Анализ «затраты – утилитарность (полезность)» (CUA – cost-utility analysis)//
Анализ «затраты – прибыль (выгода, польза)» (CBA – cost-benefit analysis)

***
Указать тип фармакоэкономического анализа, использующегося для обоснования более дешевого метода лечения://


Анализ «стоимости болезни» (COI – cost of illness)//
+Анализ «минимизации затрат» (CMA – cost-minimization analysis)//
Анализ «затраты – эффективность» (CEA – cost-effectiveness analysis)//
Анализ «затраты – утилитарность (полезность)» (CUA – cost-utility analysis)//
Анализ «затраты – прибыль (выгода, польза)» (CBA – cost-benefit analysis)

***
Указать тип фармакоэкономического анализа, позволяющего учесть и соотнести как расходы, так и эффективность лечебных воздействий://


Анализ «стоимости болезни» (COI – cost of illness)//
Анализ «минимизации затрат» (CMA – cost-minimization analysis)//
+Анализ «затраты – эффективность» (CEA – cost-effectiveness analysis)//
Анализ «затраты – утилитарность (полезность)» (CUA – cost-utility analysis)//
Анализ «затраты – прибыль (выгода, польза)» (CBA – cost-benefit analysis)

***
Указать тип фармакоэкономического анализа, учитывающего не столько достижения определенных клинических эффектов, сколько мнение (суждение) пациента о достигнутых результатах://


Анализ «стоимости болезни» (COI – cost of illness) //
Анализ «минимизации затрат» (CMA – cost-minimization analysis)//
Анализ «затраты – эффективность» (CEA – cost-effectiveness analysis)//
+Анализ «затраты – утилитарность (полезность)» (CUA – cost-utility analysis)//
Анализ «затраты – прибыль (выгода, польза)» (CBA – cost-benefit analysis)

***
Лекарственный формуляр – это://


положение, систематически разрабатываемое для помощи практикующему врачу или пациенту при принятии решения относительно оказания соответствующей медицинской помощи при определенных клинических обстоятельствах//
продукт адаптации клинических руководств, с расширением оперативных деталей для применения в местных условиях//
положение, которое выделяет цель оказания помощи в форме критериев, определяющих необходимые ресурсы, этапы, прогнозируемые результаты и ориентирован на заболевание//
положение, которое разрабатывает список основных лекарственных средств//
+ограниченные списки ЛС, упорядочивающие использование препаратов

***
Клинический протокол диагностики и лечения – это://


клиническое руководство - положение, систематически разрабатываемое для помощи практикующему врачу или пациенту при принятии решения относительно оказания соответствующей медицинской помощи при определенных клинических обстоятельствах//
+продукт адаптации клинических руководств, с расширением оперативных деталей для применения в местных условиях//
положение, которое выделяет цель оказания помощи в форме критериев, определяющих необходимые ресурсы, этапы, прогнозируемые результаты и ориентирован на заболевание//
положение, которое разрабатывает список основных лекарственных средств//
ограниченные списки ЛС, упорядочивающие использование препаратов

***
Клиническое руководство (клинические рекомендации) – это://


+положение, систематически разрабатываемое для помощи практикующему врачу или пациенту при принятии решения относительно оказания соответствующей медицинской помощи при определенных клинических обстоятельствах//
продукт адаптации клинических руководств, с расширением оперативных деталей для применения в местных условиях//
положение, которое выделяет цель оказания помощи в форме критериев, определяющих необходимые ресурсы, этапы, прогнозируемые результаты и ориентирован на заболевание//
положение, которое разрабатывает список основных лекарственных средств//
ограниченные списки ЛС, упорядочивающие использование препаратов

***
Стандарт лечения – это://


положение, систематически разрабатываемое для помощи практикующему врачу или пациенту при принятии решения относительно оказания соответствующей медицинской помощи при определенных клинических обстоятельствах//
+продукт адаптации клинических руководств, с расширением оперативных деталей для применения в местных условиях//
положение, которое выделяет цель оказания помощи в форме критериев, определяющих необходимые ресурсы, этапы, прогнозируемые результаты и ориентирован на заболевание//
положение, которое разрабатывает список основных лекарственных средств//
ограниченные списки ЛС, упорядочивающие использование препаратов

***
Нежелательные побочные реакции типа D - это://


частые, предсказуемые реакции, связанные с фармакологической активностью ЛС, могут наблюдаться у любого индивидуума//
нечастые, непредсказуемые реакции, встречающиеся только у чувствительных людей//
реакции, связанные с длительной терапией (лекарственная зависимость). К таким реакциям относится лекарственная зависимость (токсикомании, наркомании)//
+канцерогенные, мутагенные и тератогенные эффекты ЛС//
реакции, связанные с различными фармацевтическими добавками в препарате

***
Нежелательные побочные реакции типа С - это://


частые, предсказуемые реакции, связанные с фармакологической активностью ЛС, могут наблюдаться у любого индивидуума//
нечастые, непредсказуемые реакции, встречающиеся только у чувствительных людей//
+реакции, связанные с длительной терапией (лекарственная зависимость). К таким реакциям относится лекарственная зависимость (токсикомании, наркомании)//
канцерогенные, мутагенные и тератогенные эффекты ЛС//
реакции, связанные с различными фармацевтическими добавками в препарате

***
Нежелательные побочные реакции типа В - это://


частые, предсказуемые реакции, связанные с фармакологической активностью ЛС, могут наблюдаться у любого индивидуума//
+нечастые, непредсказуемые реакции, встречающиеся только у чувствительных людей//
реакции, связанные с длительной терапией (лекарственная зависимость). К таким реакциям относится лекарственная зависимость (токсикомании, наркомании)//
канцерогенные, мутагенные и тератогенные эффекты ЛС//
реакции, связанные с различными фармацевтическими добавками в препарате

***
Нежелательные побочные реакции типа А - это://


+частые, предсказуемые реакции, связанные с фармакологической активностью ЛС, могут наблюдаться у любого индивидуума//
нечастые, непредсказуемые реакции, встречающиеся только у чувствительных людей//
реакции, связанные с длительной терапией (лекарственная зависимость). К таким реакциям относится лекарственная зависимость (токсикомании, наркомании)//
канцерогенные, мутагенные и тератогенные эффекты ЛС//
реакции, связанные с различными фармацевтическими добавками в препарате

***
Согласно требованиям служб фармаконадзора, о каких нежелательных побочных реакциях следует сообщать://


обо всех нежелательных эффектах только новых ЛС//
обо всех нежелательных эффектах ЛС//
только о серьезных НПР любых препаратов//
только о неизвестных и неожиданных эффектах «старых» препаратов//
+о всех нежелательных эффектах новых препаратов, о неизвестных и неожиданных эффектах «старых» препаратов, о серьезных НПР любых препаратов

***
В качестве начальной терапии любой формы гастроэзофагеальной болезни следует использовать только://


+ингибиторы протонной помпы//
прокинетики//
антациды/алгинаты//
М-холинолитики//
Н2-гистаминоблокаторы

***
Сочетание кларитромицина с амоксициллином в эрадикационных схемах при язвенной болезни предпочтительнее, чем кларитромицина с метронидазолом, так как://


оно может способствовать более эффективной эрадикации//
оно значительно меньше вызывает побочные реакции//
оно может применяться у пожилых//
+оно может способствовать достижению лучшего результата при назначении терапии второй линии – четырехкомпонентной//
оно выгодно отличается с точки зрения экономической эффективности

***
Фармакотерапия холестатической формы хронического гепатита предусматривает применение в первую очередь://


+ урсодезоксихолевой кислоты 15 мг/кг/сут//
метаболических средств//
глюкозовитиминной смеси//
слимарина/силибинина//
орнитина аспартата

***
Выбрать правильное суждение://


Кортикостероиды при тяжелом алкогольном гепатите с нарушением функции печени (при индексе Мадрея более 32) применяются длительно (более 3 месяцев)//
+Кортикостероиды при тяжелом алкогольном гепатите с нарушением функции печени (при индексе Мадрея более 32) улучшают краткосрочный прогноз и применяются 4-6 недель//
Кортикостероиды при тяжелом алкогольном гепатите с нарушением функции печени (при индексе Мадрея более 32) не применяются//
Кортикостероиды при тяжелом алкогольном гепатите с нарушением функции печени (при индексе Мадрея более 32) применяются только при наличии асцита//
Все суждения не правильны

***
Диуретическая терапия асцита при циррозах направлена на уменьшение массы тела у пациентов с периферическими отеками примерно на://


1 кг в течение 5 дней//
2 кг в течение 4 дней//
+ 3 кг в течение 3 дней//
4 кг втечение 2 дней//
5 кг в течение 1 дня

***
Их за риска возникновения нежелательных побочных реакций метоклопрамид нельзя применять совместно с://


ингибиторами протонной помпы//
Н2-гистаминоблокаторами//
М-холинолитиками//
+противопаркинсоническими лекарственными средствами//
дигоксином

***
Указать препарат из группы Н2-гистаминоблокаторов, являющийся наиболее потенциальным ингибитором микросомального окисления печени через систему цитохрома Р - 450://


роксатидин//
низатидин//
ранитидин//
фамотидин//
+циметидин

***
Препарат выбора для лечения эрозивно-язвенных поражений желудка и 12-перстной кишки у беременных://


омепразол//
циметидин//
гастрозепин//
висмута субцитрат//
+сукральфат

***
Указать антисекреторное средство, не рекомендованное для включения в схемы эрадикационной терапии://


омепразол//
ранитидин висмут цитрат//
+фамотидин//
рабепразол//
лансопразол

***
268. Средства выбора для эмпирической антибактериальной терапии острого неосложненного цистита://


бензилпенициллин//
+фторхинолоны (норфлоксацин, офлоксацин, ципрофлоксацин)//
макролиды (эритромитцин, кларитромицин)//
аминогликозиды (канамицин, гентамицин)//
метронидазол (трихапол)

***
Характерные побочные эффекты левомецитина (хлорамфеникол)://


+угнетение кроветворения (гипопластическая анемия)//
нефротоксичность//
гепатотоксичность//
ульцерогенный эффект//
угнетение нейромышечной проводимости

***
Антибактериальный препарат выбора при нозокомиальной инфекции установленной этиологии (K.pneumoniae)://


амоксифиллин/клавуланат//
цефазолин//
+ципрофлоксацин//
цефуроксим//
ванкомицин

***
Антибактериальные препараты выбора при нозокомиальной инфекции установленной этиологии (MRSA)://


амоксифиллин/клавуланат, оксациллин//
цефазолин, цефуроксим//
+цефепим, цефтазидим//
гентамицин, нетилмицин//
эритромицин, азитромицин

***
Указать комбинацию антибактериальных препаратов, рекомендуемые при нозокомиальной инфекции установленной этиологии (Enterococcus spp)://


+ванкомицин + гентамицин//
цефазолин + метронидазол//
линкомицин + эритромицин//
цефазолин + гентамицин//
все варианты правильные

***
Выбрать правильное утверждение://


цефалоспорины I поколения противопоказаны при инфекционных заболеваниях почек//
цефалоспорины I поколения обладают высокой активностью в отношении грамотрицательной флоры при инфекциях мочевыводящих путей//
+цефалоспорины I поколения обладают недостаточной высокой активностью в отношении грамотрицательной флоры при инфекциях мочевыводящих путей. Назначение оправдано только при кортикальном абсцессе, имеющем преимущественно стафилококковую этиологию//
цефалоспорины I поколения не создают терапевтических концентраций в паренхиме почек//
цефалоспорины I поколения не уступают по эффективности цефалоспоринам III поколения при осложненных инфекциях мочевыводящих путей

***
Препарат для стартовой терапии бактериального синусита://


бензилпенициллин//
тетрациклин//
ко-тримоксазол//
гентамицин//
+амоксициллин/клавуланат

***
Антибактериальный препарат выбора для стартовой терапии бактериального синусита://


бензилпенициллин//
тетрациклин//
ко-тримоксазол//
гентамицин//
+цефуроксим

***
Ко-тримоксазол://


обеспечивает эрадикацию бета-гемалитического стрептококка и применяется для лечения острых стрептококковых тонзиллитов, вызванных чувствительными к нему in vitro штаммами//
+не обеспечивает эрадикации бета-гемалитического стрептококка и поэтому его не следует применять для лечения острых стрептококковых тонзиллитов, вызванных даже чувствительными к нему in vitro штаммами//
является препаратом выбора при остром стрептококковом тонзиллите//
применяется только в комплексной антибактериальной терапии при остром стрептококковом тонзиллите//
нет правильного ответа

***
У пациентов с острым стрептококковым тонзиллитом при наличии у них аллергии на b-лактамные антибиотики следует применять://


ко-тримоксазол//
+азитромицин//
гентамицин//
цефазолин//
хлорамфеникол

***
Антибактериальный препарат выбора при хроническом бронхите, протекающего с усилением одышки, увеличением объема и гнойности мокроты у больного в возрасте 65 лет (из анамнеза - обострения бронхита от 4 раз в год)://


амоксициллин//
+амоксициллин/клавуланат//
ко-тримоксазол//
цефазолин//
гентамицин
***
Антибактериальный препарат выбора при хроническом бронхите, протекающего с усилением одышки, увеличением объема и гнойности мокроты у больного в возрасте 65 лет. Обострения бронхита от 4 раз в год://
амоксициллин//
+левофлоксацин//
ко-тримоксазол//
цефазолин//
гентамицин

***
Препарат выбора при нетяжелой внебольничной пневмонии на амбулаторном этапе лечения у больного 50 лет без сопутствующих заболеваний://


цефтриаксон//
+амоксициллин//
ко-тримоксазол//
цефепим//
гентамицин

***
Препарат выбора при нетяжелой внебольничной терапии на амбулаторном этапе лечения у больного 50 лет без сопутствующих заболеваний://


цефтриаксон//
+спирамицин//
ко-тримоксазол//
цефепим//
гентамицин

***
Какое антибактериальное средство может вызвать подострую миелооптическую нейропатию (SMON-синдром)://


нитрофуронтоин//
ципрофлоксацин//
+нитроксолин//
ко-тримоксазол//
клиндамицин

***
При применении каких антибиотиков возможно редкое, но грозное осложнение антибактериальной терапии как псевдомембранозный энтероколит://


бензилпенициллин, ампициллин//
цефазолин, цефалексин//
+линкомицин, клиндамицин//
эритромицин, спирамицин//
метронидазол, тинидазол

***
Указать группу препаратов, относящиеся к вспомогательным лекарственным средствам, применяемым при хронической сердечной недостаточности://


ингибиторы АПФ//
β-адреноблокаторы//
+блокаторы медленных кальциевых каналов//
диуретики//
сердечные гликозиды

***
Указать группу препаратов, относящиеся к дополнительным лекарственным средствам, применяемым при хронической сердечной недостаточности://


ингибиторы АПФ//
β-адреноблокаторы//
антагонисты ангиотензиновых рецепторов//
диуретики//
+антикоагулянты (при мерцательной аритмии)

***
Медикаментозная терапия, потенциально необходимая всем пациентам с симптоматической (функциональный класс по NYHA II-IV) систолической сердечной недостаточностью://


блокаторы медленных кальциевых каналов//
антикоагулянты (при мерцательной аритмии)//
ацетилсалициловая кислота//
+ингибиторы АПФ//
негликозидные инотропные средства

***
Указать группу препаратов, относящихся к основным лекарственным средствам, применяемым при хронической сердечной недостаточности://


блокаторы медленных кальциевых каналов//
статины//
ацетилсалициловая кислота//
+β-адреноблокаторы//
негликозидные инотропные средства

***
К основному механизму положительного действия β-адреноблокаторов при хронической сердечной недостаточности относится://


только блокада β-адренорецепторов//
повышение тонуса периферических сосудов//
+блокирующее действие на нейрогуморальные системы (РАСС, эндотелиновая система, система цитокинов), ответственных за прогрессирование ХСН//
снижение сократимости миокарда//
увеличение сократимости миокарда

***
Выбрать правильный вариант применения диуретиков при хронической сердечной недостаточности (IV ФК, декомпенсация)://


не лечит мочегонными//
тиазидные диуретики, при их неэффективности – петлевые диуретики//
петлевые или тиазидные диуретики + антагонисты альдостерона (спиронолактон)//
+петлевые диуретики + тиазидные диуретики + антагонисты альдостерона (спиронолактон) + ингибиторы карбоангидразы//
тиазидные диуретики + антагонисты альдостерона (спиронолактон) + ингибиторы карбоангидразы

***
У больных с хронической ИБС при невозможности назначить β-адреноблокаторы (непереносимость, противопоказания) как альтернативное средство применяют://


амлодипин//
+верапамил//
амиодарон//
нифедипин//
эналаприл

***
При артериальной гипертензии с сопутствующей сердечной недостаточностью и/или дисфункцией левого желудочка предпочтительно назначение://


амлодипина//
доксазозина//
моксонидина//
клонидина//
+эналаприла

***
При артериальной гипертензии в сочетании хронической ИБС предпочтительно назначение://


+бисопролола//
амлодипина//
нифедипина//
резерпина//
верапамила

***
При артериальной гипертензии с сопутствующей диабетической нефропатией предпочтительно назначение://


метапролола//
+фозиноприла//
резерпина//
клонидина//
доксазозина

***
При артериальной гипертензии с сопутствующей диабетической нефропатией у больных, не переносящих ингибиторы АПФ предпочтительно назначение://


бисопролола//
верапамила//
+лозартана//
гидрохлортиазида//
моксонидина

***
Антигипертензивный прерарат выбора при артериальной гипертензии у беременных://


эналаприл//
гидрохлортиазид//
лозартан//
+метилдопа//
доксазозин

***
Указать рациональную комбинацию антигипертензивных средств://


антагонист кальция + диуретик//
антагонист кальция + α-адреноблокатор//
β-адреноблокатор + препарат центрального действия//
+β-адреноблокатор + диуретик//
все комбинации рациональные

***
Выбрать рациональную комбинацию антигипертензивных средств://


+ингибитор АПФ + тиазидный диуретик//
антагонист кальция + α-адреноблокатор//
β-адреноблокатор + препарат центрального действия//
антагонист кальция + диуретик//
все комбинации нерациональные

***
Показать рациональную комбинацию антигипертензивных средств://


антагонист кальция + α-адреноблокатор//
+β-адреноблокатор + антагонист кальция из группы дигидропиридинов//
β-адреноблокатор + препарат центрального действия//
антагонист кальция + диуретик//
все комбинации нерациональные

***
Применение β-адреноблокаторов при феохромоцитоме до начала лечения α-адреноблокаторами, может привести к://


незначительному снижению артериального давления//
+повышению артериального давления//
значительному снижению артериального давления//
снижению сократительной функции миокарда//
повышению седативного эффекта

***
Какой из указанных препаратов наиболее позволяет контролировать ЧСС как в состояние покоя, так и при физической нагрузке у больных мерцательной аритмии://


дигоксин//
верапамил//
дилтиазем//
+бисопролол//
нет правильного ответа

***
Препарат выбора для профилактики системных тромбоэмболий у больных с постоянной формой мерцательной аритмией старше 65 лет://


ацетилсалициловая кислота//
+варфарин//
дипиридамол//
гепарин//
все указанные препарата

***
Какое взаимодействие возможно при одновременном применении НПВС и ингибиторов АПФ://


+НПВС приводят к снижению гипотензивного эффекта ингибиторов АПФ, благодаря угнетению синтеза вазодилатирующих простагландинов//
НПВС не приводят к снижению гипотензивного эффекта ингибиторов АПФ//
НПС потенцируют гипотензивный эффект эффекта ингибиторов АПФ//
суммация нежелательных побочных действий//
нет правильного ответа

***
Указать эффективные антиаритмические средства для лечения желудочковых аритмий опасных для жизни://


пропранолол//
бисопролол//
верапамил//
+амиодарон//
аденозин

***
Отрицательные стороны применения диуретиков (тиазидные, петлевые) при хронической сердечной недостаточности://


+актвизируют нейрогормоны (РААС)//
объемная разгрузка сердца//
положительный диурез//
увеличение сердечного выброса//
нет правильного ответа

***
Побочный эффект гепарина://


+тромбоцитопения//
лейкопения//
гипокалиемия//
ДВС-синдром//
гипотония

***
Противопоказания к назначению непрямых антикоагулянтов://


церебральный тромбоз//
+беременность//
нестабильная стенокардия//
стабильная стенокардия//
мерцательная аритмия у больных высокого риска

***
Терапия при передозировке тромболитических средств предполагает отмену препаратов и://


внутривенное введение протамина сульфата//
внутримышечное введение конакиона//
переливание тромбоцитарной массы//
+введение аминокапроновой кислоты в сочетании с введением фибриногена//
нет правильного ответа

***
Лечение ДВС-синдрома в гиперкоагуляционной фазе наряду с лечением основного заболевания предполагает следующее://


применение препаратов, воздействующих на сосудистый гемостаз (этамзилат, адроксон)//
+гепаринотерапия, применение свежезамороженной плазмой//
введение протамина сульфата//
применение тромболитиков//
переливание концентрата тромбоцитов

***
Лечение ДВС-синдрома в гипокоагуляционной фазе наряду с лечением основного заболевания предполагает следующее КРОМЕ://


+гепаринотерапия (большие дозы)//
свежезамороженная плазма (криоплазма)//
применение препаратов, воздействующих на сосудистый гемостаз (этамзилат, адроксон)//
переливание концентрата тромбоцитов//
применение ингибиторов протеолитических ферментов (апротинин)

***
Ранжировать НПВС по выраженности противовоспалительного действия://


+индометацин > диклофенак > пироксикам > ибупрофен > метамизол > ацетилсалициловая кислота//
ацетилсалициловая кислота > метамизол > ибупрофен > пироксикам >диклофенак >индометацин//
пироксикам > ибупрофен > метамизол > ацетилсалициловая кислота >индометацин > диклофенак//
диклофенак > ацетилсалициловая кислота > метамизол > пироксикам > индометацин > ибупрофен//
индометацин > ацетилсалициловая кислота > метамизол >пироксикам > диклофенак > ибупрофен

***
Ранжировать НПВС по выраженности анальгетического действия://


ацетилсалициловая кислота > пироксикам > метамизол > индометацин > диклофенак > кеторолак//
ацетилсалициловая кислота > диклофенак > пироксикам > метамизол > индометацин > кеторолак//
+кеторолак > диклофенак > индометацин > метамизол > пироксикам > ибупрофен > ацетилсалициловая кислота//
метамизол > индометацин > диклофенак > пироксикам > кеторолак > ацетилсалициловая кислота > ибупрофен//
метамизол > кеторолак > ацетилсалициловая кислота > диклофенак > индометацин > ибупрофен > пироксикам

***
На основании лабораторных показателей ( рН -7,2; SB – 15 ммоль/л; К – 5,3 ммоль/л ) выбрать препараты для коррекции кислотно-основного состояния://


аммония хлорид, кальция хлорид//
препараты, стимулирующие дыхательный центр//
+бикарбонат, лактат натрия, трисамин//
калийсберегающие диуретики в сочетании с хлоридом аммония//
нет правильного ответа

***
На основании лабораторных показателей (рН - 7,45; SB – 35 ммоль/л; натрий крови – 110 ммоль/л, калий – 3,2 ммоль/л) выбрать препараты для коррекции кислотно-основного состояния://


препараты, стимулирующие дыхательный центр//
+калийсберегающие диуретики (ингибиторы карбоангидразы) в сочетании с хлоридом аммония///
петлевые диуретики//
бикарбонат, лактат натрия, трисамин//
нет правильного ответа

***
Применение спиронолактона вызывает побочный эффект, связанный с электролитными нарушениями://


гипокалиемия//
метаболический алкалоз//
гипомагнезиемия//
+метаболический ацидоз//
не вызывает электролитные нарушения

***
Указать стабилизатор мембран тучных клеток://


фенотерол//
+недокромил//
прометазин (дипразин, пипольфен)//
ипратропия бромид (атровент, тровентол)//
кетотифен (задитен)

***
Расположить адреномиметики по мере убывания длительности их действия://


+сальметерол > фенотерол > сальбутамол > орципреналин > изопреналин > эпинефрин//
фенотерол > сальметерол > орципреналин > сальбутамол > эпинефрин > фенотерол //
изопреналин > фенотерол > сальметерол > сальбутамол > орципреналин > эпинефрин//
эпинефрин > фенотерол > сальбутамол > орципреналин > изопреналин > сальметерол//
эпинефрин > фенотерол > сальбутамол > изопреналин > сальметерол >орципреналин

***
Указать препараты базисной терапии легкой персистирующей бронхиальной астмы (2 ступень)//


не требуются://
+ингаляционные глюкокортикоиды (500 мкг и менее бекломатазона или эквивалент)//
ингаляционные глюкокортикоиды (200-1000 мкг бекломатазона или эквивалент) + ингаляционные β2-агонисты длительного действия//
ингаляционные глюкокортикоиды (1000 мкг и более бекломатазона или эквивалент) + ингаляционные β2-агонисты длительного действия + теофиллины замедленного высвобождения, при необходимости пероральные глюкокортикоиды//
пероральные глюкокортикоиды + теофиллины замедленного высвобождения

***
Указать препараты базисной терапии персистирующей бронхиальной астмы средней степени тяжести (3 ступень)://


не требуются//
ингаляционные глюкокортикоиды (500 мкг и менее бекломатазона или эквивалент)//
+ингаляционные глюкокортикоиды (200-1000 мкг бекломатазона или эквивалент) + ингаляционные β2-агонисты длительного действия//
ингаляционные глюкокортикоиды (1000 мкг и более бекломатазона или эквивалент) + ингаляционные β2-агонисты длительного действия + теофиллины замедленного высвобождения, при необходимости пероральные глюкокортикоиды//
пероральные глюкокортикоиды + теофиллины замедленного высвобождения

***
Указать препараты базисной терапии персистирующей бронхиальной астмы тяжелой степени (4 ступень)://


не требуются//
ингаляционные глюкокортикоиды (500 мкг и менее бекломатазона или эквивалент)//
ингаляционные глюкокортикоиды (200-1000 мкг бекломатазона или эквивалент) + ингаляционные β2-агонисты длительного действия//
+ингаляционные глюкокортикоиды (1000 мкг и более бекломатазона или эквивалент) + ингаляционные β2-агонисты длительного действия + теофиллины замедленного высвобождения, при необходимости пероральные глюкокортикоиды//
пероральные глюкокортикоиды + теофиллины замедленного высвобождения

***
Указать гипнотик (снотворное средство) из группы бензодиазепинов://


+нитразепам (радедорм)//
золпидем//
зопиклон (имован)//
фенобарбитал//
дифенгидрамин (димедрол)

***
Какой транквилизатор имеет дополнительный психостимулирующий эффект://


+ медазепам (рудотель)//
диазепам//
лоразепам//
хлордиазепоксид//
мепробамат

***
Указать транквилизатор с дополнительным психостимулирующим эффектом://


+альпразолам//
диазепам//
лоразепам//
хлордиазепоксид//
мепробамат

***
Укажите потенциальный индуктор микросомального окисления (в гепатоцитах)://


+диазепам//
циметидин//
тетрациклин//
ристомицин//
церебролизин

***
Какой препарат из группы антагонистов кальция используется при сосудистой патологии головного мозга и не вызывает клинически значимых изменений центральной гемодинамики://


верапамил (финоптин)//
дилтиазем//
нифедипин (коринфар)//
+циннаризин//
нет правильного ответа

***
Нейролептики- это://


лекарственные средства, способные уменьшать выраженность чувство тревоги, страха и эмоциональной напряженности//
+лекарственные средства, способные редуцировать психотическую симптоматику и психомоторное возбуждение//
лекарственные средства, сглаживать циркуляторные расстройства аффективной сферы (колебания настроения), предотвращать развитие депрессивной и маниакальной симптоматики//
лекарственные средства, вызывать психомоторную активность, как у больных, так и у здоровых лиц//
лекарственные средства, способные активизировать нейрометаболические и антигипоксические процессы в головном мозге

***
Указать Н1-гистаминоблокаторы II поколения://


дифенгидрамин (димедрол)//
+лоратадин (кларитин)//
мебгидролин (диазолин)//
хлоропирамин (супрастин)//
все перечисленные

***
Каким термином обозначают действие лекарственных средств во время беременности, которое приводит к врожденным уродствам://


мутагенное действие//
фетотоксическое действие//
+тератогенное действие//
эмбриотоксическое действие//
все ответы верны

***
Каким термином обозначают необычные реакции на лекарственные средства, связанные с генетически обусловленными энзимопатиями://


сенсибилизация//
тахифилаксия//
+идиосинкразия//
абстиненция//
привыкание

***
Терапевтическая широта лекарственного средства определяется://


соотношением пороговой и максимальной токсической доз//
соотношением минимальной эффективной и средней терапевтической доз//
+соотношением минимальной эффективной и минимальной токсической дозы//
соотношением средней терапевтической и максимальной безопасной доз//
соотношением пороговой и смертельной дозы.

***
К синтетическим реакциям метаболизма лекарственных средств относятся все, КРОМЕ://


конъюгация с аминокислотами//
конъюгация с глюкуроной кислотой//
+ароматическое гидроксилирование//
ацетилирование//
метилирование

***
При курении метаболизм некоторых лекарственных средств (теофиллин) ускоряется за счет://


ускорения всасывания лекарственных средств//
ингибирования оксидаз ферментной системы цитохрома Р-450 печени//
+индуцирования оксидаз ферментной системы цитохрома Р-450 печени//
аутоиндукции//
нет правильного ответа

***
Практически за два периода полувыведение из организма выводится://


25% препарата//
50% препарата//
+75% препарата//
90% препарата//
100% препарата

***
При пероральном одновременном назначении тетрациклинов и антацидов (содержащих алюминий) возможно://


усиление всасывания в желудочно-кишечном тракте//
+образование нерастворимых комплексов (хелатов)//
потенцирование действий друг друга//
антабусное действие//
нет правильного ответа

***
Указать гипотензивные средства, рекомендуемые при гипертонии беременных://


ингибиторы ангиотензинпревращающего фермента//
α-адреноблокаторы//
тиазидные диуретики//
антагонисты ангиотензиновых рецеторов 2//
+метилдопа, β-адреноблокаторы (при отсутствии угрозы беременности)

***
Какое антибактериальное средство противопоказано при беременности и лактации://


бензилпенициллин//
+тетрациклин//
амоксициллин//
линкомицин//
цефазолин

***
Какие антибактериальные средства противопоказаны на последних неделях 3 триместра беременности://


+ко-тримоксазол//
пенициллины//
макролиды//
линкомицин//
цефалоспорины

***
Какие антибактериальные средства противопоказаны у детей до 8 лет://


защищенные аминопенициллины//
+фторхинолоны//
цефалоспорины//
аминогликозиды//
линкосамиды

***
Какие антибиотики противопоказаны у детей до 8 лет://


защищенные пенициллины//
цефалоспорины//
аминогликозиды//
+тетрациклины//
линкосамиды

***
Применение бензодиазепинов в последние недели беременности может привести к://


гипертермии у новорожденного//
анемии у новорожденного//
гипертензии у новорожденного//
стимуляции дыхания у новорожденного//
+угнетению дыхания у новорожденного

***
Гипервитаминоз какого из указанных средств может привести к тератогенному эффекту://


витамин В1 //
витамин В6 //
витамин В12 //
витамин Е//
+витамин А

***
Средства выбора для эмпирической антибактериальной терапии пиелонефрита у беременных://


+амоксициллин//
хлорамфеникол//
ципрофлоксацин//
гентамицин//
тетрациклин

***
Противопоказание к назначению калийсберегающих диуретиков://


+беременность//
метаболический алкалоз//
гипокалиемия//
гиперальдостеронизм//
артериальная гипертония

***
Механизм действия метоклопрамида (церукал)://


блокирует М-холинорецепторы//
+блокирует дофаминовые рецепторы и подавляет высвобождение ацетилхолина//
воздействует на серотониновые рецепторы//
прямое спазмолитическое действие//
блокирование ацетилхолина в нервно-мышечном сплетении

***
Основной механизм действия слабительных из производных антрахинов (препараты сенны, крушины)://


задерживают воду в кишечнике//
увеличивают объем содержимого кишечника//
размягчают каловые массы//
+вызывают химическое раздражение рецепторного аппарата толстой кишки (стимуляция перистальтики)//
все ответы верны

***
Механизм слабительного действия лактулозы://


увеличивают объем содержимого кишечника//
+осмотическое действие (задерживают воду в кишечнике)//
размягчают каловые массы//
вызывают химическое раздражение рецепторного аппарата толстой кишки (стимуляция перистальтики)//
все ответы верны

***
В разделе инструкции «Фармакологическое действие» описывается://


+ механизм действия, локализация действия, виды действия//
лекарственные взаимодействия//
генетические аспекты действия//
процессы всасывания, распределения, метаболизма, выведения//
механизм действия

***
Противопоказания к применению мизопростола (сайтотек, цитотек)://


почечная недостаточность//
+ беременность//
поражение ЦНС//
аллергия в анамнезе//
все ответы верны

***
Показания к применению салазопиридазина (салазодин)://


хронический некалькулезный холецистит//
хронические запоры//
язвенная болезнь желудка//
+болезнь Крона, язвенный колит//
все ответы правильные

***
Сроки эрадикационной терапии при НР-ассоциированной язвенной болезни://


3-5 дней//
5 дней//
+7-14 дней//
16-20 дней//
28 дней

***
Указать противопоказания к назначению Н2-гистаминоблокаторов://


+почечная и печеночная недостаточность//
хронический эрозивный гастрит//
гипертоническая болезнь//
ожирение//
язвенная болезнь

***
Противопоказания к назначению метоклопрамида (церукала) и домперидона (мотилиума)://


хронический колит//
хронический гастрит//
+механическая непроходимость кишечника//
рвота органического и функционального происхождения//
детский возраст (старше 3 лет)

***
Медикаментозное лечение при неязвенной диспепсии с язвенноподобным вариантом течения://


фамотидин + коллоидный субцитрат висмута + тетрациклин//
ранитидин + коллоидный субцитрат висмута + тетрациклин+ метронидазол//
пилорид + кларитромицин+ метронидазол//
метоклопрамид + антацид//
+пирензепин (гастрозепин) + антацид

***
Метронидазол://


плохо проникает в материнское молоко и через плацентарный барьер//
+хорошо проникает в материнское молоко и через плацентарный барьер//
плохо проникает через плаценту//
плохо проникает через гематоэнцефалический барьер//
нет правильного ответа

***
Антибактериальное средство с высокой активностью по отношению к синегнойной палочке://


цефуроским//
налидиксовая кислрота//
+ ципрофлоксацин//
тетрациклин//
гентамицин

***
Какой из указанных цефалоспоринов обладает наиболее высокой активностью против синегнойной палочки://


цефтриаксон//
цефамандол//
+цефтазидим//
цефуроксим//
цефалотин

***
Указать препараты выбора для эмпирической антибактериальной терапии пиелонефрита://


+амоксициллин, ципрофлоксацин//
гентамицин, нетилмицин//
ампициллин, оксациллин//
цефазолин, цефалексин//
эритромицин, азитромицин

***
Указать возможное взаимодействие при одновременном назначении фуросемида и гентамицина://


+усиление ототоксичности//
усиление гематологических осложнений//
ослабление эффекта фуросемида//
брадикардия//
нет взаимодействия

***
Какое антибактериальное средство может приводить к псевдомембранозному энтероколиту://


амоксициллин/клавуланат//
цефазолин//
+клиндамицин//
метронидазол//
ципрофлоксацин

***
Антибактериальное средство выбора для лечения урогенитального хламидиоза://


+азитромицин//
цефазолин//
фуразолидон//
амоксицилллин/клавуланат//
гентамицин

***
Препарат выбора для эмпирической антибактериальной терапии острого холецистита://


+цефотаксим//
кларитромицин//
нитроксолин//
амоксициллин//
хлорамфеникол

***
Средства выбора для эмпирической антибактериальной терапии острого инфекционного эндокардита (наиболее вероятный возбудитель S.aureus)://


амоксициллин, ампициллин//
бензипенициллин, феноксиметилпенициллин//
+оксациллин, цефалоспорины (I и II поколение), ванкомицин//
метронидазол, тинидазол//
норфлоксацин, офлоксацин

***
Средства выбора для эмпирической антибактериальной терапии абсцесса легкого (наиболее часто вызывается анаэробной флорой)://


+амоксициллин/клавуланат + метронидазол//
гентамицин + бензилпенициллин//
ванкомицин + азитромицин//
нитрофураны + нитроксолин//
эритромицин + линкомицин

***
Укажите побочные эффекты аминофиллина (эуфиллин)://


+тахикардия, экстрасистолия//
антидиуретический эффект, отеки лодыжек///
бронхоспазм, пневмонит//
нарушение проводимости, брадикардия//
нефротоксичность, нейротоксичность

***
Какой бронходилататор показан при синдроме бронхиальной обструкции в сочетании с ИБС стенокардия напряжения ФК II, НКI://


орципреналин (алупент, астмопент)//
изопреналин (новодрин, изадрин)//
+ипратропиум бромид (атровент)//
аминофиллин (эуфиллин)//
все перечисленные средства

***
Укажите продолжительность действия формотерола://


3-4 часа//
6-8 часов//
8-10 часов//
+10-12 часов//
16-18 часов

***
Почему препараты теофиллина короткого действия в настоящее время не рекомендуются к применению://


вызывают сильное диуретическое действие//
в связи с их медленным всасыванием//
+в связи с их быстрым всасыванием и наступлением побочных эффектов//
малоэффективные средства//
нет правильного ответа

***
Препарат противопоказанный при бронхообструктивном синдроме://


эналаприл//
+амиодарон//
верапамил//
нифедипин//
гидрохлортиазид

***
Наиболее часто встречающийся побочный эффект сальбутамола://


+тремор, головная боль, незначительная тахикардия//
наджелудочковые и желудочковые экстрасистолы//
замедление атриовентрикулярной проводимости//
повышение артериального давления, кризы//
не правильного ответа

***
Какие антиангинальные средства показаны при ИБС, умеренной стабильной стенокардии без признаков сердечной недостаточности://


нитрат длительного действия (в сочетании ингибиторами АПФ)//
+прием нитроглицерина при приступах и регулярный прием бета-адреноблокатора//
антагонист кальция+ бета-адреноблокатор+ нитрат длительного действия+ (ингибитор АПФ)//
антагонисты кальция+ бета-адреноблокаторы//
используются все указанные средства

***
Дозировка ацетилсалициловой кислоты при стабильной стенокардии://


+75-160 мг 1 раз в сутки//
320-500 мг в сутки//
500 мг в сутки//
500 мг 2 раза в сутки//
1500 мг в сутки

***
Противопоказания к применению нитроглицерина://


+гипотензивные состояния и гиповолемия//
хроническая сердечная недостаточность//
нестабильная стенокардия//
наджелудочковая экстрасистолия//
ИБС, стенокардия напряжения

***
Когда нитраты считаются противопоказанными при лечении инфаркте миокарда://


сохраняющаяся ишемия//
сердечная недостаточность//
артериальная гипертензия//
+ЧСС менее 50 мм.рт.ст., САД менее 90 мм.рт.ст.//
все ответы верны

***
Противопоказания к назначению β-адреноблокаторов при инфаркте миокарда://


+умеренная и тяжелая левожелудочковая недостаточность//
ЧСС более 60 мин//
САД более 100 мм.рт.ст.//
отсутствие признаков левожелудочковой недостаточности//
все ответы верны

***
Какое противопоказание к назначению бета-адреноблокаторов при инфаркте миокарда://


+атривентрикулярная блокада II и III степени//
ЧСС более 60 мин//
САД более 100 мм.рт.ст.//
отсутствие признаков левожелудочковой недостаточности//
все ответы верны

***
Препарат выбора из группы антагонистов кальция у больных, перенесших инфаркт, при невозможности использования β-адреноблокаторов (когда имеются противопоказания к их применению)://


нифедипин//
+верапамил//
амлодипин//
лацидипин//
циннаризин

***
Длительное применение ингибиторов АПФ у больных, перенесших инфаркт миокарда, показано://


у больных без дисфункций миокарда левого желудочка//
у больных с дисфункцией миокарда левого желудочка только в случаях клинических проявлений сердечной недостаточности//
+у больных с дисфункцией миокарда левого желудочка в случаях бессимптомной, так и сопровождающейся клиническими проявлениями сердечной недостаточности//
нет правильного ответа//
все ответы верны

***
Указать β-адреноблокаторы с периферическим вазодилатирующим действием://


пиндолол//
атенолол//
пропранолол//
+карведилол//
все перечисленные
***
Выберите состояние, противопоказанное к назначению нитроглицерина://
+кровоизлияние в мозг//
нестабильная стенокардия//
артериальная гипертензия//
сердечная недостаточность//
левожелудочковая недостаточность

***
Как предупредить толерантность к нитратам://


+назначать их только в то время суток, когда возникновение приступов стенокардии наиболее вероятно//
увеличить дозировку нитратов//
использовать накожные пластыри с нитратами//
увеличить кратность приема//
все ответы верны

***
Главным критерием для назначения медикаментозной терапии артериальной гипертензии является://


степень повышения АД//
длительность заболевания//
+принадлежность к группе риска и степень повышения АД//
возраст больного//
все перечисленное

***
Индивидуальный выбор антигипертензивных средств: преимущественное показание назначения диуретиков при артериальной гипертензии с учетом сопутствующей патологии и состояний://


+систолическая гипертония в сочетании с сердечной недостаточностью//
стенокардия//
дислипидемия//
подагра//
сахарный диабет

***
Индивидуальный выбор бета-блокаторов: преимущественное показание назначения β-адреноблокаторов при артериальной гипертонии с учетом сопутствующей патологии и состояний://


+стенокардия//
систолическая гипертония//
заболевания периферических сосудов//
депрессия//
дислипидемия

***
Назначение антигипертензивных средств: преимущественное показание назначения β-адреноблокаторов при артериальной гипертонии с учетом сопутствующей патологии и состояний://


систолическая гипертония//
заболевания периферических сосудов//
депрессия//
+тахиаритмии//
дислипидемия

***
Индивидуальный выбор антигипертензивных средств: преимущественное показание назначения β-адреноблокаторов при артериальной гипертонии с учетом сопутствующей патологии и состояний://


+перенесенный инфаркт миокарда//
систолическая гипертония у пожилых//
заболевания периферических сосудов//
депрессия//
дислипидемия

***
Индивидуальный выбор антигипертензивных средств: преимущественное показание назначения антагонистов кальция (нифедипина) при артериальной гипертонии с учетом сопутствующей патологии и состояний://


+диабетическая нефропатия//
сердечная недостаточность//
перенесенный инфаркт миокарда с признаками сердечной недостаточности (НК II)//
атриовентрикулярная блокада II и III степени//
тахиаритмия

***
Индивидуальный выбор антигипертензивных средств: преимущественное показание назначения антагонистов кальция (верапамил) при артериальной гипертонии с учетом сопутствующей патологии и состояний://


+тахиаритмии//
сердечная недостаточность//
перенесенный инфаркт миокарда с признаками сердечной недостаточности (НК II)//
атриовентрикулярная блокада II и III степени//
брадикардия

***
Индивидуальный выбор антигипертензивных средств: преимущественное показание назначения ИАПФ при АГ с учетом сопутствующей патологии://


гиперкалиемия//
+сердечная недостаточность//
тахиаритмии//
хронический пиелонефрит//
наджелудочковая экстрасистолия

***
Индивидуальный выбор антигипертензивных средств: преимущественное показание назначения ингибиторов ангиотензинпревращающего фермента при артериальной гипертонии с учетом сопутствующей патологии и состояний://


гиперкалиемия//
тахиаритмии//
хронический пиелонефрит с ХПН//
наджелудочковая экстрасистолия//
+диабетическая нефропатия

***
Указать группу препаратов, способные потенциально маскировать симптомы гипогликемии://


+β –адреноблокаторы//
α-адреноблокаторы//
антагонисты кальция//
ингибиторы АПФ//
антагонисты рецепторов ангиотензина II

***
Противопоказания к назначению ингибиторов ангиотензинпревращающего фермента://


бронхиальная астма//
недостаточность кровообращения//
+стеноз обеих почечных артерий//
хронический пиелонефрит без нарушения функции почек//
артериальная гипертензия

***
Какой препарат из группы тиазидных и тиазидоподобных диуретиков обладает меньшим отрицательным влиянием на углеводный обмен у больных сахарным диабетом://


гидрохлортиазид//
хлорталидон//
+индапамид//
фуросемид//
спиронолактон

***
Для купирования тахикардии типа "пируэт" (двунаправленная веретенообразная желудочковая тахикардия) используется://


амиодарон//
+препараты магния//
соталол//
лидокаин//
эналаприл

***
Указать препараты из I группы антиаритмических средств (классификация в соответствии их влияния на электрические процессы в миокарде)://


+прокаинамид, лидокаин//
бисопролол, атенолол//
амиодарон, соталол//
дилтиазем, верапамил//
дигоксин, строфантин

***
Указать препараты из III группы антиаритмических средств (классификация в соответствии их влияния на электрические процессы в миокарде)://


прокаинамид, лидокаин//
пропранолол, атенолол//
+ амиодарон, соталол//
дилтиазем, верапамил//
дигоксин, строфантин

***
Средством выбора для купирования пароксизмальной наджелудочковой тахикардии является://


+аденозин//
строфантин//
хинидин//
лидокаин//
магния сульфат

***
Препараты, используемые при суправентрикулярных нарушениях ритма сердца://


тримекаин//
лидокаин//
+верапамил//
дифенин//
магния сульфат

***
Укажите препарат, используемый при суправентрикулярных нарушениях ритма сердца у больных с бронхиальной астмой://


бета-адреноблокаторы//
лидокаин//
+верапамил//
дифенин//
магния сульфат

***
Указать эффективное антиаритмическое средство у больных с суправентрикулярными тахикардиями://


+верапамил//
лидокаин//
тримекаин//
мексилетин//
дифенин

***
Указать эффективное антиаритмическое средство у больных с желудочковыми тахиаритмиями://


верапамил//
дилтиазем//
+лидокаин//
аденозин//
строфантин

***
Особенности повышения доз бета-адреноблокаторов (начало терапии предполагает 1/8 средней терапевтической дозы) при лечении хронической сердечной недостаточности://


+увеличение в 2 раза не чаще чем через 2 нед//
увеличение в 4 раза не чаще чем через 2 нед//
увеличение в 4 раза не чаше чем через 1 нед//
доведение до среднетерапевтической дозы в течение недели//
нет правильного ответа

***
Сердечные гликозиды://


оказывают положительный хронотропный эффект//
оказывают отрицательный инотропный и отрицательный хронотропный эффекты//
оказывают положительный инотропный и положительный хронотропный эффекты//
оказывают положительный дромотропный эффект//
+оказывают положительный инотропный и отрицательный хронотропный эффекты

***
Выбор диуретика при вторичном гиперальдостеронизме в сочетании с гипокалиемией://


петлевые диуретики//
тиазидные диуретики//
акворетики//
+спиронолактон//
осмотические диуретики

***
Побочный эффект фуросемида://


гиперкалиемия//
ацидоз//
+метаболический алкалоз//
понижение уровня мочевой кислоты//
повышение АД

***
Положительные стороны применения диуретиков (тиазидные, петлевые) при хронической сердечной недостаточности://


активация нейрогормонов (РААС)//
+объемная разгрузка сердца//
отрицательный диурез//
снижение сердечного выброса//
все перечисленные

***
В качестве дополнительного средства при длительном назначении мощных диуретиков для восстановления рН и чувствительности к петлевым диуретикам назначают://


маннитол//
буметамид//
гидрохлоротиазид//
+ацетазоламид//
все перечисленные

***
Риск геморрагических осложнений при применении гепарина возрастает при сочетании с://


ингибиторами АПФ//
витамином К//
протамином сульфат//
+нестероидными противовоспалительными средствами//
ристомицином

***
Показания к применению непрямых антикоагулянтов://


+периферический артериальный тромбоз//
длительная профилактика и лечение тромбоза глубоких вен//
бактериальный эндокардит//
острый инфаркт миокарда//
нестабильная стенокардия

***
Показания к применению варфарина, неодикумарола://


периферический артериальный тромбоз//
бактериальный эндокардит//
острый инфаркт миокарда//
нестабильная стенокардия//
+профилактика тромбоэмболических осложнений при мерцательной аритмии у больных высокого риска

***
Показания к применению стрептокиназы://


расслаивающаяся аневризма//
нарушения мозгового кровообращения//
нестабильная стенокардия//
предупреждение тромботических осложнений при периферическом атеросклерозе//
+острый инфаркт миокарда

***
Начало действия витамина К и его синтетических аналогов://


через 1-2 часа//
через 3-6 часов//
через 8-10 часов//
+через 12-18 часов//
нет правильного ответа

***
Механизм действия нестероидных противовоспалительных средств://


+торможение синтеза медиаторов воспаления – простагландинов//
усиление выработки макроэргических фосфатов//
влияние на систему цитокинов//
антигистаминное действие//
стабилизация мембран тучных клеток

***
Какой из нестероидных противовоспалительных средств чаще вызывают побочные действия со стороны нейросенсорной сферы (головокружение, головные боли, чувство утомления, расстройства сна)://


ацетилсалициловая кислота//
ибупрофен//
+индометацин//
кетопрофен//
диклофенак

***
Антипсихотическое действие нейролептиков://


+связывают с блокадой Д2-дофаминовых рецепторов, изменением дофаминергической нейротрансмиссии//
связывают с действием на специфические бензодиазепиновые рецепторы, которые связаны с ГАМК- 4-рецепторами//
связывают с повышением активности серотонинергической и норадренергической нейропередачи//
связывают с блокадой норадренергических рецепторов//
нет правильного ответа

***
Указать эффект, не характерный для анксиолитиков://


транквилизирующий (анксиолитический)//
седативное действие//
миорелаксирующее действие//
противосудорожное действие//
+дезингибирующее антипсихотическое действие

***
Какой транквилизатор показан при неврозах с адинамией, вялостью и заторможенностью://


+медазепам (рудотель, мезапам)//
нитразепам (радедорм)//
диазепам (реланиум, сибазон, валиум)//
оксазепам (нозепам, тазепам)//
феназепам

***
Преимущества Н1-гистаминоблокаторов II поколения перед Н1-гистаминоблокаторами I поколения://


+не проходят через ГЭБ в терапевтических дозах//
блокада других рецепторов//
оказывают седативный эффект//
эффективны при бронхиальноф астме (инфекционнозависимой)//
все ответы верны

***
Указать Н1-гистаминоблокаторы I поколения://


лоратадин (кларитин)//
акревастин (семпрекс)//
эбастин (кестин)//
+хлоропирамин (супрастин)//
цетиризин (зиртек)

***
Фармакокинетическая кривая - это://


зависимость концентрации препарата в крови от дозы//
+ зависимость концентрации препарата в крови от времени//
зависимость концентрации препарата в крови от кратности введения//
зависимость скорости выведения препарата от времени//
зависимость метаболизма лекарства от времени

***
Определить вид синергии: взаимодействие фуросемида и этакриновой кислоты://


сенситизирующее действие//
аддитивное действие//
+суммация//
потенцирование//
нет правильного ответа

***
Пациент 68 лет, жалобы на общую слабость. При осмотре – нерегулярный пульс. На ЭКГ: фибрилляция предсердий. Ранее эпизодов аритмии не наблюдалось. В анамнезе: 4 месяца назад был установлен стент с лекарственным покрытием в переднюю межжелудочковую артерию сердца по поводу стеноза 70% (без ОКС). В анамнезе: АГ, гиперлипидемия, СД 2тип. Объективно: Очаговых неврологических симптомов нет. ЧДД – 18 в мин. В легких везикулярное дыхание, хрипов нет. Тоны сердца ритм неправильный, ЧСС – 72 в мин., пульс – 75 в мин., АД – 125/80 мм рт.ст., отеков нет. Проводимая терапия: атенолол 50 мг, лизиноприл 5 мг, метформин, аторвастатин 20 мг, ацетилсалициловая кислота 100 мг, клопидогрель 75 мг. Каково количество баллов по CHA2DS2-VASc у пациента://


менее 2 баллов//
2 балла//
+ 4 балла//
6 баллов//
8 баллов

***
Пациент 68 лет, жалобы на общую слабость. При осмотре – нерегулярный пульс. На ЭКГ: фибрилляция предсердий. Ранее эпизодов аритмии не наблюдалось. В анамнезе: 4 месяца назад был установлен стент с лекарственным покрытием в переднюю межжелудочковую артерию сердца по поводу стеноза 70% (без ОКС). В анамнезе: АГ, гиперлипидемия, СД 2тип. Объективно: Очаговых неврологических симптомов нет. ЧДД – 18 в мин. В легких везикулярное дыхание, хрипов нет. Тоны сердца ритм неправильный, ЧСС – 72 в мин., пульс – 75 в мин., АД – 125/80 мм рт.ст., отеков нет. Проводимая терапия: атенолол 50 мг, лизиноприл 5 мг, метформин, аторвастатин 20 мг, ацетилсалициловая кислота 100 мг, клопидогрель 75 мг. Каково число баллов по шкале HAS-BLED у пациента://


0 баллов//
1 балл//
+ 2 балла//
4 балла//
9 баллов
***
Пациент 68 лет, жалобы на общую слабость. При осмотре – нерегулярный пульс. На ЭКГ: фибрилляция предсердий. Ранее эпизодов аритмии не наблюдалось. В анамнезе: 4 месяца назад был установлен стент с лекарственным покрытием в переднюю межжелудочковую артерию сердца по поводу стеноза 70% (без ОКС). В анамнезе: АГ, гиперлипидемия, СД 2тип. Объективно: Очаговых неврологических симптомов нет. ЧДД – 18 в мин. В легких везикулярное дыхание, хрипов нет. Тоны сердца ритм неправильный, ЧСС – 72 в мин., пульс – 75 в мин., АД – 125/80 мм рт.ст., отеков нет. Проводимая терапия: атенолол 50 мг, лизиноприл 5 мг, метформин, аторвастатин 20 мг, ацетилсалициловая кислота 100 мг, клопидогрель 75 мг. Каково Ваше решение о назначении данному пациенту антикоагулянтной терапии://
Данный пациент не нуждается в назначении антикоагулянтной терапии//
Данный пациент нуждается в назначении антикоагулянтной терапии. Выбор в пользу новых оралных антикоагулянтов (НОАК). При невозможности их приема – варфарин под контролем МНО (международного нормализованного отношения), целевое МНО – 2,5-3,5//
Данный пациент нуждается в назначении антикоагулянтной терапии. Выбор – в пользу новых оралных антикоагулянтов (НОАК). При невозможности их приема – варфарин под контролем МНО (международного нормализованного отношения), при трудности контроля МНО – ацетилсалициловая кислота//
+ Данный пациент нуждается в назначении антикоагулянтной терапии. Выбор в пользу новых оралных антикоагулянтов (НОАК). При невозможности их приема – варфарин под контролем МНО (международного нормализованного отношения), целевое МНО – 2-3//
Данный пациент нуждается в назначении ацетилсалициловой кислоты в качестве средства первичной профилактики

***
Пациентка 68 лет, госпитализирована с признаками ТЭЛА. Ранее венозных тромбоэмболических осложнений не было. Явного провоцирующего фактора, а также наличия злокачественных новообразований не выявлено. Начато лечение антикоагулянтами. Выберите правильное утверждение://


Антикоагулянты можно отменить при выписке из стационара//
Антикоагулянты можно отменить через 3 месяца//
Антикоагулянты можно отменить через 6 месяца//
+ Антикоагулянты можно отменить через 12 месяцев//
Антикоагулянты следует принимать длительно (без определенной даты прекращения)

***

Пациентка 68 лет, госпитализирована с признаками ТЭЛА. 2 года назад перенесла тромбоз правой общей бедренной вены. Явного провоцирующего фактора, а также наличия злокачественных новообразований не выявлено. Начато лечение антикоагулянтами. Выберите правильное утверждение://
Антикоагулянты можно отменить при выписке из стационара//
Антикоагулянты можно отменить через 3 месяца//
Антикоагулянты можно отменить через 6 месяца//
Антикоагулянты можно отменить через 12 месяцев//
+ Антикоагулянты следует принимать длительно (без определенной даты прекращения)

***
Признаки передозировки дигоксина://


головные боли, запоры//
урежение пульса//
боли за грудиной, снижение давления//
+ отсутствие аппетита, тошнота, урежение пульса, нарушения зрения, корытообразная депрессия сегмента ST на ЭКГ//
тошнота, рвота, боли в животе

***
Наиболее частыми признаками внутреннего кровотечения при использовании пероральных антикоагулянтов являются://


+ резкая слабость, бледность кожных покровов, тахикардия, снижение АД//
головные боли; повышение артериального давления//
повышение температуры тела, кашель, боли в грудной клетке//
боли в животе, тошнота, рвота//
нарушение сознания, головные боли

***
Следует назначить пероральный антикоагулянт при://


+ числе баллов 2 и более по шкале CHA2DS2-VASс//
числе баллов более 2 по шкале CHA2DS2-VASс//
числе баллов более 3 по шкале CHA2DS2-VASс//
числе баллов более 1 по шкале CHA2DS2-VASс//
числе баллов 0 по шкале CHA2DS2-VASс

***
К критериям качества жизни пациентов относятся://


положительные эмоции, мышление, изучение, концентрация, самооценка, внешний вид, переживания//
личные взаимоотношения, общественная ценность субъекта, сексуальная активность//
+ физические, психологические, уровень независимости, общественная жизнь, окружающая среда, духовность//
религия, личные убеждения//
сила, энергия, усталость, боль, дискомфорт, сон, отдых

***
Исследование качества жизни в медицине – важный компонент современных клинических исследований и клинической практики, позволяющий://


+ получить представление об общих закономерностях реакции больного на патологический процесс, оценить эффективность лечения у конкретного пациента//
изучить эффекты ЛС на большой популяции людей//
определить прямую выгоду от применения лекарственных средств//
произвести учет затрат, понесенных медицинским учреждением при проведении диагностики и лечения определенного заболевания//
изучить применение ЛС и их эффекты на ограниченном контингенте людей

***
Оценка качества жизни пациентов производится с помощью специальных методик://


Интервьюирования//
Анкетирования//
социологического опроса//
телефонного консультирования//
+ Специализированных опросников

***
Информация из раздела «Побочные действия лекарственных средств» использутся для://


контроля за эффективностью//
+ контроля за безопасностью//
выбора индивидуального режима дозирования//
коррекции режима дозирования у пациента с печеночной недостаточностью//
коррекции режима дозирования у пациента с почечной недостаточностью

***
При применении сердечных гликозидов у больных с застойной сердечной недостаточностью не отмечают://


увеличение систолического и минутного объема сердца//
уменьшение венозного давления//
уменьшение отеков//
+тахикардию//
уменьшение одышки

***
Укажите показания для назначения нифедипина://


профилактика и лечение стенокардии//
стенокардия, кардиофобия, кардиалгия//
+гипертензия, стенокардия//
острая фаза инфаркта миокарда//
гипотоническая дисциркуляторная дистония

***
При гликозидной интоксикации следует назначить://


+КCL + дифенин + раствор глюкозы//
КCL + новодигал+ реополюгликин//
КCL + допамин + гипертонин раствор NaCL//
КCL + амрион + клифт//
КCL + строфантин + аспаркам

***
438. Укажите побочные эффекты нифедипина://


стенокардия, кардиофобия, кардиалгия//
парадоксальная реакция, гипотония, метгемоглобинемия//
гипертензия, жировойгепатоз, амблиопия//
+тахикардия, гипотония//
цианоз, гипертензия

***
При недостаточности витамина РР (никотинамида) развивается://


+дерматит, деменция, диарея//
рахит, остеопроз//
иммунодефицит, цинга//
злокачественная анемия//
неврит, парез

***
Чем отличается оксодолин от дихлотиазида://


+диуретический эффект развивается через 2-4 часа и продолжается до 3 суток//
диуретический эффект развивается через 24 часа и продолжается до 10 суток//
является самым высокоактивным диуретиком//
является низкоактивным диуретиком//
может применяться при отеке легких

***
Побочные эффекты клопамида и оксодолина сходны с побочными эффектами://


+дихлотиазида//
мочевины//
фуросемида//
спиронолактона//
триамтерена

***
Правильное утверждение://


действие фуросемида развивается медленно//
маннит назначается только внутрь//
+совместно с дихлотиазидом можно назначать триамтерен//
клопамид действует кратковременно//
маннит действует в области собирательных трубок

***
Фуросемид может вызывать алкалоз. Как изменяется при этом его эффективность://


снижается//
повышается//
+не изменяется//
потенциируется//
суммируется

***
Как влияет дихлотиазид на диурез при несахарном мочеизнурении://


увеличивает//
+уменьшает//
не изменяет//
изменяет//
усиливает

***
К слабо действующим диуретикам относятся://


буметанид, ксипамид, пиретанид, фуросемид, этакриновая к-та //
бендрофлутиазид, гидрохлоротиазид, клопамид, хлорталидон//
+амилорид, калия канреонат, триамтерен, спиронолактон, диакарб//
манит, мочевина, концентрированные растворы глюкозы, глицерин//
уросемид, этакриновая к-та, триамтерен, спиронолактон, диакарб

***
Нежелательные эффекты нитроксолина://


+головная боль, головокружение, парестезии, полинейропатия, миелооптическаянейропатия, атрофия зрительного нерва, заторможенность, ретроградная амнезия, тошнота, рвота, потеря аппетита, аллергии, тахикардия//
тошнота, рвота, диарея, холестаз, гепатит, сыпь, эозинофилия, лихорадка, артралгия, миалгия, волчаночноподобный синдром, редко - анафилактический шок,пневмонит, бронхоспазм, кашель, боль в грудной клетке, головокружение, головная боль, общая слабость, сонливость, периферическиеполинейропатии, лейкопения, мегалобластная или гемолитическая анемия.//
гипокалиемия, задержка в организме мочевой кислоты: гиперурикемия, гипохлоремический алкалоз, гипергликемия и обострение сахарного диабета, ортостатическая гипотония,повышение в крови холестерина, интерстициальный нефрит, образование в почках оксалатных камней, диспепсические расстройства//
выраженная гипокалиемия, гипомагниемия, гиперурикемия, гипергликемия, диспептические расстройства, панкреатит, поражение ЦНС//
диспептические расстройства; гиперкалиемия; метаболический ацидоз; сонливость, головная боль, атаксия; гормональные расстройства; кожная сыпь, тромбоцитопения

***
Угнетают проникновение вируса в клетку и депротеинизацию://


+амантадин, ремантадин//
зидовудин, ацикловир//
ацикловир, видарабин//
тетрациклин, оксолин//
тетрациклин, идоксуридин

***
Действуют на ДНК-содержащие вирусы://


+идоксуридин, ацикловир, видарабин//
амантадин, ремантадин,мидантан//
зидовудин, ацикловир, мидантан//
тетрациклин, оксолин, ремантадин//
тетрациклин, идоксуридин, оксолин

***
Оказывают противоспирохетозное действие за счет блокады тиоловых ферментов://


+бисмоверол, пентабисмол, бийохинол//
бициллины, бензилпенициллины//
тетрациклины, макролиды//
тетрациклин, оксолин//
тетрациклин, идоксуридин

***
При лейшманиозе применяют://


Пиперазин, пирантел//
Аминоакрихин, празиквантель//
+Солюсурьмин, мономицин//
Метронидазол, тинидазол //
Левамизол, хлорохин

***
Действуют на амеб любой локализации://


Пиперазин, пирантел//
Аминоакрихин, празиквантель//
Солюсурьмин, мономицин//
+Метронидазол, тинидазол //
Левамизол, хлорохин

***
Влияние аминазина на АД://


+ снижает//
повышает//
не изменяет//
делает неустойчивым//
вызывает гипертонический криз

***
Антидепрессант, избирательный ингибитор нейронального захвата норадреналина://


+Мапротилин//
Амитриптилин//
Моклобемид//
Флуоксетин//
Имизин

***
Какой из предложенных режимов антибактериальной терапии наиболее подходит больному с легким течением внебольничной пневмонии ранее не болевшему и не принимавшему антибиотики://


амоксициллин-клавулановая кислота//
амоксициллин-клавулановая кислота плюс азитромицин//
цефтриаксон//
+азитромицин//
левофлоксацин

***
Что характеризует скорость выведения ЛС из организма://


Т1/2//
+общий клиренс//
биоэквивалентность//
Биодоступность//
объем распределения

***
Величина биодоступности важна для определения://


+пути введения ЛС//
величины поддерживающей дозы//
кратности приема//
скорости выведения//
эффективности препарата

***
Что характеризует параметр "кажущийся объем распределения"://


скорость всасывания препарата//
скорость выведения препарата//
скорость распада препарата//
эффективность препарата//
+способность препарата проникать в органы и ткани

***
Объем распределения зависит от://


+интенсивности кровотока в органах и тканях//
скорости всасывания препарата//
Т1/2//
скорости выведения препарата//
пути введения ЛС

***
Какие из перечисленных препаратов снижают скорость опорожнения желудка://


Бикарбонат натрия//
Дигоксин//
Церукал//
+Атропин//
Фурасемид

***
Индукция микросомального окисления в печени характерна для://


+Фенобарбитал//
Пенициллин//
Пропранолол//
Фуросемид//
Кортизон

***
Cнижениe метаболической активности печени существенно увеличивает Т 1/2://


Cтрофантина//
Лидокаина//
+Дигитоксина//
Пропранолола//
Гентаницина

***
В кислой моче увеличивается выведение://


Барбитураты//
+Морфин//
Стептомицин//
Бутадион//
Сульфаниламиды

***
Какие из перечисленных препаратов вызывают ощелачивание мочи://


аскорбиновая кислота//
ацетилсалициловая кислота//
тиазидные диуретики//
+бикарбонат натрия//
Индометацин

***
При циррозах печени существенно увеличивается Т1/2://


Строфантин//
+Пропранолол//
Пенициллин//
Тетрациклин//
Гентаницин

***
К благоприятным взаимодействиям относятся://


трудность контроля терапевтического эффекта//
утяжеление состояния больного//
учащение известных для каждого лекарства ПЭ и осложнений //
появление нового ПЭ, не характерного для каждого из сочетаемых ЛС//
+увеличение терапевтической эффективности

***
К основным видам взаимодействия ЛС является://


Антагонистическое//
Конкурентное//
+Фармакодинамическое//
Целесообразное//
Физиологическое

***
К фармакокинетическому взаимодействию относится://


влияние препарата на взаимодействие ЛС с рецептором //
+влияние препарата на всасывание другого ЛС//
изменение чувствительности ткани к ЛС//
влияние на медиатор//
конкуренция за рецепторы

***
К фармакодинамическому взаимодействию относится://


+конкуренция за рецепторы//
влияние препарата на всасывание другого ЛС//
влияние на распределение ЛС в организме//
влияние на связывание ЛС с белками, элементами крови и тканями//
влияние на биотрансформацию ЛС

***
Побочные эффекты, выраженность которых не зависит от дозы://


фармакодинамические (прямого действия)//
повреждающие (токсические)//
+аллергические и иммунные//
фармакогенетические//
фармакокинетические

***
Побочные эффекты, выраженность которых легко предсказуемa://


+повреждающие (токсические)//
аллергические и иммунные//
фармакогенетические//
мутагенные, онкотератогенные и эмбриотоксические//
парамедикаментозные

***
Выберите фармакодинамические побочные эффекты из следующих://


+сухость во рту при приеме атропина//
зудящая кожная сыпь при приеме эритромицина//
кожный зуд и покраснение лица при приеме бисептола//
нарушение зрения при приеме амиодарона//
желтуха и гепатомегалия при приеме индометацина

***
ЛС, прекращение приема которых может вызвать синдром абстиненции://


Клонидин//
Нитроглицерин//
Золпидем//
+Морфин//
Пропранолол

***
Нефротоксичность является характерным ПЭ для следующих ЛС://


+Аминогликозидов//
Антикоагулянтов//
Симпатолитиков//
бета-блокаторов//
иАПФ

***
Условия, при которых у больных повышается риск развития ПЭ://


гипоальбунемии (для ЛС, имеющих связь с белком < 30%)//
почечная недостаточность (для дигитоксина)//
цирроз печени (для строфантина)//
+выраженная сердечная недостаточность//
гипертиреоз

***
Укажите ЛС, представляющие большую опасность для плода://


Пенициллин//
Парацетамол//
Макролиды//
прямые антикоагулянты//
+Цитостатики

***
В поздние сроки беременности://


максимален риск тератогенного действия ЛС//
+нестероидные противовоспалительные ЛС могут вызывать задержку родов//
опиаты, принимаемые матерью, не оказывают влияния на плод//
Бензодиазепины, принимаемые матерью, не оказывают влияния на плод//
средства общей анестезии, проводимой во время родов, не оказывают влияния на плод

***
Следующие положения касательно анафилактического шока справедливы://


высвобождение гистамина - основная причина шока//
резко расширяются бронхи//
интенсивная терапия включает в/м введение адреналина//
интенсивная терапия требует введения антигистаминных препаратов//
+АД резко снижается

***
Когда наблюдается наибольший риск эмбриотоксического действия ЛС://


+первые 7 дней с момента зачатия (до имплантации)//
первые 3-4 месяца беременности (стадия эмбриогенеза)//
второй триместр беременности//
третий триместр беременности//
7-10 дней перед родами

***
Когда наблюдается наибольший риск тератогенного действия ЛС ://


первые 7 дней с момента зачатия (до имплантации)//
+первые 3-4 месяца беременности (стадия эмбриогенеза)//
второй триместр беременности//
третий триместр беременности//
7-10 дней перед родами

***
Когда токсическое действие ЛС на плод особенно опасно://


первые 7 дней с момента зачатия (до имплантации)//
первые 3-4 месяца беременности (стадия эмбриогенеза)//
второй триместр беременности//
третий триместр беременности//
+7-10 дней перед родами

***
ЛС, необходимость применения которых требует прерывания беременности://


Пенициллины//
витамины группы В//
макролиды//
+противоопухолевые антибиотики//
Финлепсин

***
Aнтибактериальные препараты, которые можно использовать на протяжении всей беременности://


+Пенициллины//
Сульфаниламиды//
Бисептол//
Нитрофураны//
Фторхинолоны

***
Xарактерное действие на ребенка резерпина при приеме его кормящей матерью://


повышение возбудимости, крик, тахикардия, мочегонный эффект//
+насморк, бронхоспазм, угнетение ЦНС, вялость//
увеличение секреции пролактина, размеров молочной железы//
ототоксический эффект у детей с поносами и воспалением слизистой ЖКТ//
геморрагии, нарушение дыхания, синдром Рейе

***
Xарактерное действие на ребенка теофиллина при приеме его кормящей матерью://


+повышение возбудимости, крик, тахикардия, мочегонный эффект//
насморк, бронхоспазм, угнетение ЦНС, вялость//
угнетение дыхания, сонливость//
Ототоксический эффект у детей с поносами и воспалением слизистой ЖКТ//
геморрагии, нарушение дыхания, синдром Рейе

***
Xарактерное действие на ребенка аминогликозидов при приеме их кормящей матерью://


повышение возбудимости, крик, тахикардия, мочегонный эффект//
насморк, бронхоспазм, угнетение ЦНС, вялость//
увеличение секреции пролактина, размеров молочной железы//
+Ототоксический эффект у детей с поносами и воспалением слизистой ЖКТ//
геморрагии, нарушение дыхания, синдром Рейе

***
Xарактерное действие на ребенка этанола при приеме его кормящей матерью://


повышение возбудимости, крик, тахикардия, мочегонный эффект//
насморк, бронхоспазм, угнетение ЦНС, вялость//
+угнетение дыхания, сонливость//
Ототоксический эффект у детей с поносами и воспалением слизистой ЖКТ//
геморрагии, нарушение дыхания, синдром Рейе

***
Xарактерное действие на ребенка салицилатов при приеме его кормящей матерью://


повышение возбудимости, крик, тахикардия, мочегонный эффект//
насморк, бронхоспазм, угнетение ЦНС, вялость//
угнетение дыхания, сонливость//
Ототоксический эффект у детей с поносами и воспалением слизистой ЖКТ//
+геморрагии, нарушение дыхания, синдром Рейе

***
Компенсация сахарного диабета достигнута на фоне бутамида (утром 1 г и вечером 0,5 г). В связи с выявленной стенокардией II ф.к. дополнительно назначили анаприлин (40 мг 3 р/сут) и аспирин (0.125 г в день).К концу 1 недели комплексной терапии больной отметил периодически возникающие приступы слабости, потливости, беспокойства, головокружения, сопровождавшиеся выраженной бледностью кожных покровов. Что послужило причиной развития данного состояния ://


передозировка бутамида//
передозировка анаприлина//
+фармакодинамическое взаимодействие анаприлина и бутамида//
фармакодинамическое взаимодействие анаприлина и аспирина//
фармакодинамическое взаимодействие аспирина и бутамида

***
Больной с жалобами на нервозность, тревогу, эмоциональную напряженность, нарушение сна, частое повышение АД назначен диазепам в суточной дозе 10 мг и раунатин по 1 т 2 раза в день. В анамнезе хронический гепатит. Через 2 недели больная отметила появление галлюцинаций, ярких сновидений, вялость и дрожание пальцев рук. Что привело к появлению побочного действия диазепама://


назначение высокой дозы препарата//
повышение всасывания препарата из ЖКТ//
+замедление процесса биотрансформации препарата в организме//
повышение выведения препарата из организма//
взаимодействие диазепама и раунатина

***
Больному, страдающему ИБС, атеросклеротическим кардиосклерозом, ХНК IIБ ст.


(ЧСС - 96, Ps - 76, АД - 120/80, PQ - 0.21 сек) для устранения частой желудочковой экстрасистолии был назначен лидокаин болюсно (50 мг) с последующим в/в капельным введением (1 мг/мин). Почему использована доза в два раза меньше средней://
из-за тахикардии//
из-за замеделния AV-проводимости//
вследствие замедления выведения лидокаина почками //
из-за гипоальбуминурии, частой у больных с кардиальным циррозом печени//
+из-за снижения объема распределения ЛС у больных с ХНК

***
У больной, страдающей хроническим пиелонефритом, клиренс креатинина соста-


вил 60 мл/мин. Назначение какого ЛС должно производится в меньших дозах://
Фуросемида//
Дигоксина//
Пенициллина//
Аспирина//
+Гентамицина

***
Период полувыведения лекарств - это://


время достижения максимальной концентрации лекарства в плазме//
время, в течение которого лекарство достигает системного кровотока//
время, в течение которого лекарство распределяется в организме//
+время, за которое концентрация лекарства в плазме снижает­ся на 50%//
ремя, за которое половина введенной дозы достигает орга­на-мишени

***
Терапевтический индекс-это://


терапевтическая доза лекарства//
отношение концентрации лекарства в органе или ткани кконцентрации его в плазме крови//
+соотношение между минимальной терапевтической и токси­ческой концентрациями лекарства в плазме//
процент не связанного с белком лекарства//
соотношение между минимальной и максимальной терапевтическими концентрациями лекарства

***
К рецепторным средствам конкурентного действия относятся://


НПВС (нестероидные противовоспалительные средства)//
+бета-адреноблокаторы//
петлевые диуретики//
Нитраты//
Фторхинолоны

***
При назначении каких лекарственных средств следует учиты­вать функцию печени и почек://


липофильных, образующих неактивные метаболиты//
+липофильных, образующих активные метаболиты//
Гидрофильных//
Гепатотоксичных//
нефротоксичных

***
Селективность действия лекарственного вещества зависит от://


периода полувыведения//
способа приема//
связи с белком//
объема распределения//
+ дозы

***
Строго дозозависимой является следующая группа побочных эффектов://


Фармацевтические//
Фармакогенетические//
Аллергические//
Мутагенные//
+синдром отмены

***
Выберите группу препаратов с узким терапевтическим индексом://


бета-блокаторы//
Пенициллины//
+сердечные гликозиды//
ингибиторы АПФ//
петлевые диуретики

***
Проведение лекарственного мониторинга требуется при лечении следующей группой лекарственных препаратов://


+противосудорожными//
бета-блокаторами//
антагонистами кальция//
глюкокортикоидами//
М-холинолитиками

***
Развитие асистолии возможно при комбинации пропранолола с://


Фенобарбиталом//
+Верапамилом//
Каптоприлом//
Фенитоином//
Фуросемидом

***
Риск токсических эффектов увеличивается при комбинации гентамицнна с://


+Фуросемидом//
Пенициллином//
Метилксантинами//
Макролидами//
Глюкокортикоидами

***
При патологии почек возникают следующие изменения фар­макокинетики лекарств, кроме://


нарушения почечной экскреции//
увеличения концентрации лекарств в плазме крови//
уменьшения связывания с белками плазмы//
увеличения Т1/2//
+уменьшения биодоступности

***
Цирроз печени вызывает следующие изменения фармакоки­нетики лекарств, кроме://


снижения пресистемного метаболизма//
уменьшения связывания с белками плазмы//
увеличения Т1/2//
увеличения биодоступности//
+уменьшения объема распределения

***
При сердечной недостаточности наблюдаются следующие изменения фармакокинетики дигоксина, кроме://


снижения абсорбции в ЖКТ на 30%//
уменьшения связывания с белком плазмы//
+усиления метаболизма в печени//
снижения почечной экскреции//
увеличения Т1/2

***
Алкоголь приводит к://


увеличению абсорбции лекарств//
увеличению объема распределения лекарств//
+замедлению метаболизма в печени//
снижению почечной экскреции//
уменьшению Т1/2

***
Никотин приводит к ://


уменьшению абсорбции лекарств//
увеличению объема распределения лекарств//
увеличению связи с белками плазмы//
+ускорению метаболизма в печени//
усилению почечной экскреции лекарств

***
При стенокардии напряжения наиболее показан://


Нифедипин//
+Пропранолол//
Капотен//
Эналаприл//
Клонидин

***
При стенокардии Принцметала (вазоспастической) показан://


+Нифедипин//
Обзидан//
Дипиридамол//
Допегит//
Каптоприл

***
Абсолютным критерием эффективности антиангинального средства является://


прирост времени нарузки на ВЭМ на I мин//
уменьшение количества потребляемого нитроглицерина//
+прирост времени ВЭМ-пробы > 2 мин//
снижение времени нагрузки//
переход больного из II в III функциональный класс стенокардии

***
К антинангинальным средствам относятся://


Аспирин//
Курантил//
Капотен//
+Верапамил//
АТФ


***
Объективные методы оценки гипотензивной эффективности ЛС://
динамика ЧСС в покое и в ходе ВЭН//
+динамика АД в покое и в ходе ВЭН//
динамика толерантности к ВЭН//
динамика диуреза//
динамика времени развития ишемических изменений на ЭКГ

***
Критерии оценки антиангинальной эффективности ЛС://


динамика ЧСС в покое и в ходе ВЭН//
динамика АД в покое и в ходе ВЭН//
динамика толерантности к нитратам//
динамика диуреза//
+динамика времени развития ишемических изменений на ЭКГ

***
Эффект фуросемида (при выраженном отечном синдроме) считается оптимальным при://


Прирост диуреза до 0.5 л //
Прирост диуреза до 1 л//
Прирост диуреза до 1.5 л//
+Прирост диуреза - 1.5 - 2.5 л//
Прирост диуреза - 3 - 4 л
***
Укажите методы контроля за эффективностью применения диуретиков при отечном синдроме://
определение концентрации в плазме крови Na+ и K+//
+измерение суточного диуреза и количества выпитой жидкости//
измерение суточного калийуреза//
определение КЩР//
контроль ЭКГ не реже 1 раза в неделю

***
Укажите причины рефрактерности к диуретикам://


+Гипоальбуминемия//
Гиперкалиемия//
Гипокальциемия//
увеличение почечного кровотока //
повышение глюкокортикоидной функции надпочечников

***
Выберите правильные утверждения для диуретиков://


Частый ПЭ фуросемида у больных с ХНК – гиперкалиемия//
Проявления гипокалиемии - головная боль, психомоторное возбуждение//
Проявления гипокалиемии на ЭКГ - подъем ST, высокий Т в v4-v6.//
Для коррекции гипокалиемии - панангин по 1 т 3 раза в день//
+Для профилактики гипокалиемии назначают фуросемид с верошпироном

***
Выберите правильные утверждения://


Одновременный прием верошпирона и дигоксина повышает риск интоксикации СГ//
+Oдновременный прием фуросемида и амикацина усиливает нефротоксичность//
Oдновременный прием фуросемида и цепорина снижает нефротоксичность//
Oдновременный прием диакарба и этазола усиливает кристаллурию//
Одновременный прием фуросемида и индометацина усиливает диурез

***
При объективной оценке эффективности антибактериальной терапии нельзя использовать следующие критерии://


динамику локальной инфекции//
динамику синдромов лихорадки, интоксикации//
динамику лабораторных показателей активности воспаления//
динамику бактериологических и иммунологических показателей//
+динамику жалоб больного

***
Pежим введения коррегируется при снижении Cl creat < 80 мл/мин у://


+Аминогликозиды//
Ампициллин//
Метициллин//
Оксациллин//
цефалоспорины (кроме цефалоридина)

***
Режим введения меняется при заболеваниях желчевыводящих путей и печени у://


+Эритромицин//
Аминогликозиды//
Ампициллин//
Метициллин//
Цефалоспорины

***
Антибиотики, подавляющие синтез белка и нуклеиновых кислот в микробной клетке://


природные пенициллины //
полусинтетические пенициллины//
цефалоспорины//
+Макролиды//
Аминогликозиды

***
Антибиотики, нарушающие синтез белка и функцию клеточных мембран://


Пенициллины//
Цефалоспорины//
Макролиды//
Тетрациклины//
+ Аминогликозиды

***
Антибиотики, нарушающие синтез мембраны в бактериальной клетке://


+цефалоспорины //
макролиды//
природные тетрациклины//
полусинтетические тетрациклины//
аминогликозиды

***
Цефалоспорины, действующие на синегнойную палочку://


Цефазолин//
Цефтриаксон//
Цефалексин//
Цефамандол//
+Цефаперазолон

***
Ваша тактика у больных, длительно получавших ГК, при амбулаторных операционных вмешательствах://


Полная предварительная отмена ГК//
Уменьшение дозы ГК на 25-30%//
+Увеличение дозы ГК в 1,5-2 раза за день до операции//
Кратковременная отмена за 1 день до операции на 3-4 дня //
Доза ГК не меняется

***
Укажите темпы отмены ГК при длительной терапии://


Уменьшают дозу ГК на 5 мг каждый день//
+Уменьшают дозу ГК на 2,5-5 мг каждые 7-10 дней//
Полностью отменяют ГК в течение нескольких дней//
Уменьшают дозу ГК на 10 мг каждые 7-10 дней//
Уменьшают дозу ГК на 10 мг каждый месяц

***
Укажите клинические признаки "синдрома отмены" глюкокортикоидов://


Артериальная гипертензия//
Гипергликемия//
+Головные боли, боли в пояснице, суставах//
Возврат клинических симптомов основного заболевания//
Снижение резистентности организма к инфекциям, стрессовым ситуациям

***
Какие причины лежат в основе формирования ГК-зависимости://


Гиперкотицизм//
Психический дискомфорт при отмене ГК//
+Гипофункция коры надпочечников//
Снижение выработки половых гормонов//
Снижение функции щитовидной железы

***
Какие клинические симптомы входят в синдром Иценко-Кушинга://


Эррозивные гастриты, язвы желудка//
+Ожирение по "верхнему" типу//
Артериальная гипотония//
Слабость, адинамия, сонливость//
Снижение функции щитовидной железы

***
К побочным эффектам, развивающимся на фоне длительной ГК терапии, относится все кроме://


Атрофия надпочечников//
Эрозивные гастриты, язвы желудка//
+Общая слабость, сонливость, адинамия//
Активация инфекций//
Артериальная гипертензия, отеки

***
Индометацин у беременных вызывает все, кроме://


подавление преждевременных сокращений матки//
преждевременное закрытие Боталлова протока у плода//
концентрация в амниотической жидкости 50-60% от таковой в плазме женщины//
+ индометацин не проникает в амниотическую жидкость//
повышение АД у плода при длительном приеме его беременными

***
Отметить группы психотропных средств, применяющиеся преимущественно при психозах://


+Нейролептики//
анксиолитики(транквилизаторы)//
седативные средства//
психостимуляторы//
ноотропные средства

***
Отметить группы психотропных средств, применяемых преимущественно при психических нарушениях непсихотической природы (неврозы и неврозоподобные состояния)://


+анксиолитики(транквилизаторы)//
антидепрессанты//
соли лития //
антипсихотические средства (нейролептики)//
гипнотики

***
При болях, связанных со злокачественными опухолями применяют://


+промедол //
парацетамол //
кислоту ацетилсалициловую //
фенацетин //
налоксон

***
Для купирования болей при инфаркте миокарда применяют://


+морфин //
парацетамол//
кислоту ацетилсалициловую //
фенацетин //
налоксон

***
По анальгетической активности и способности угнетать дыхание превосходит морфин, действует кратковременно (20-30 мин.), применяется в основном для нейролептанальгезии://


омнопон //
Промедол//
пентазоцин //
+фентанил//
Трамадол

***
Какие препараты не следует применять при фторотановом наркозе во избежание развития сердечных аритмий://


+адреналин //
мезатон//
атропин//
димедрол//
морфин

***
Развитию сердечных аритмий при фторотановом наркозе способствуют://


Атропин//
+Адреналин//
Мезатон//
Морфин//
Димедрол

***
Для повышения артериального давления при гипотензии, вызванной фторотановым наркозом, применяют://


+Мезатон//
Адреналин//
Норадреналин//
Эфедрин//
Атропин

***
Для предупреждения рефлекторных реакций блуждающего нерва на сердце перед наркозом вводят://


Адреналин//
Мезатон//
+Атропин//
Морфин//
Димедрол

***
Чувствительность миокарда к адреналину значительно повышает://


эфир для наркоза//
+Фторотан//
азота закись//
Изофлюран//
Гексенал

***
Для кратковременного наркоза в амбулаторной практике целесообразно применять://


Фторотан//
+Пропанидид//
эфир для наркоза//
натрия оксибутират//
Хлороформ

***
В качестве антигипоксического средства используют://


эфир для наркоза//
Пропанидид//
+натрия оксибутират//
Гексенал//
азота закись

***
Определить препарат: вызывает наркоз через 30-40 мин. после введения в вену, длительность наркоза 1,5-3 часа, обладает седативным, снотворным и антигипоксическим действием, вызывает выраженную миорелаксацию, может вызывать судорожные сокращения скелетных мышц и гипокалиемию://


Пропанидид//
+натрия оксибутират//
Предион//
Гексенал//
тиопентал-натрий

***
Определить препарат: вызывает наркоз через 1 мин. после введения в вену, длительность наркоза 20-30 мин.депонируется в жировой ткани, может вызывать судорожные сокращения скелетных мышц и ларингоспазм, противопоказан при нарушениях функций печени://


Пропанидид//
Кетамин//
натрия оксибутират//
+тиопентал-натрий//
Пропофол

***
Определить препарат: вызывает наркоз через 30-40 сек. после введения в вену, длительность действия около 3 мин., гидролизуется холинэстеразой, оказывает местнораздражающее действие, может вызывать образование тромбов://


натрия оксибутират//
+пропанидид//
гексенал//
тиопентал-натрий//
кетамин

***
Определить препарат: вводится в вену или в мышцу, под влиянием препарата хирургический наркоз не развивается, возникает общее обезболивание с частичной утратой сознания://


тиопентал-натрий//
натрия оксибутират//
+кетамин//
пропанидид//
Гексенал

***
Oпределить препарат: обладает низкой наркотической активностью и в связи с этим обычно применяется в комбинации с активными средствами для наркоза, последействия практически не оказывает, побочных эффектов не вызывает не воспламеняется, но поддерживает горение://


Фторотан//
эфир для наркоза//
+азота закись//
хлороформ//
изофлюран

***
Определить препарат: обладает высокой наркотической активностью, вызывает кратковременную стадию возбуждения, вводится ингаляционно, вызывает брадикардию, снижает артериальное давление, сенсибилизирует миокард к адреналину, безопасен в пожарном отношении://


азота закись//
+фторотан //
хлороформ//
изофлюран//
эфир для наркоза

***
Определить препарат: вызывает наркоз достаточной глубины после выраженной стадии возбуждении, после наркоза вызывает продолжительный вторичный сон, вводится ингаляционно, раздражает слизистые оболочки, огнеопасен://


Фторотан//
+эфир для наркоза//
азота закись//
хлороформ//
изофлюран

***
Применяется как снотворное и противоэпилептическое средство, вызывает индукцию микросомальных ферментов печени://


+фенобарбитал //
Дифенин//
натрия вальпроат //
этосуксимид//
триметин
***
Центральное мышечно-расслабляющее, анксиолитическое и противоэпилептическое действие характерно для://
фенобарбитала //
Триметина//
дифенина //
Этосуксимида//
+сибазона
***
Определите препарат: относится к соединениям алифатического ряда, обладает высокой снотворной активностью, в высоких дозах вызывает наркоз, оказывает местнораздражающее действие, практически не влияет на структуру сна, в печени превращается в активный метаболит://
фенобарбитал //
+хлоралгидрат //
нитразепам //
бромизовал //
сибазон

***
Определите препарат: обладает седативным и снотворным действием, не вызывает наркотического эффекта, вызывает сон главным образом за счет устранения психического напряжения, снотворный эффект связывают с влиянием на лимбическую систему://


фенобарбитал //
+феназепам //
этаминал-натрий //
Бромизовал//
хлоралгидрат

***
Определите препарат: обладает седативными,снотворными и противоэпилептическими свойствами, незначительно метаболизируется в печени и вызывает выраженную индукцию ее микросомальных ферментов, применяется в основном в качестве противоэпилептического и снотворного средства://


этаминал-натрий //
бромизовал //
+фенобарбитал //
Нитразепам//
Хлоралгидрат

***
Ортостатическая гипотония и гипотония при физической нагрузке встречается при использовании: //


Диуретиков//
+альфа-блокаторов//
прямых артериодилататоров//
блокаторов "меделенных" кальциевых каналов//
ингибиторов конвертирующего фермента

***
У больного 43 лет имеется выраженная одышка и нет возможности уточнить связано ли это с бронхиальной или кардиальной астмой. Положительный эффект в обоих случаях могут оказать://


сердечные гликозиды//
+эуфиллин//
адреналин//
глюкокортикоиды//
фуросемид

***
Укажите наиболее безопасные сочетания препаратов у больного c нормальным АД и стенокардией и ЧСС 94: //


нитроглицерин + нифедипин//
верапамил + празозин//
гидралазин + гигротон//
празозин + нифедипин//
+нитраты + верапамил

***
Kакие препараты Вы предпочтете у больного со стенокардией и ХНК IIБ степени://


+Нитраты//
Верапамил//
Дилтиазем//
Празозин//
Анаприлин

***
Если у больного с артериальной гипертензией отмечается бронхиальная астма, то какие из препаратов следует выбрать://


+Коринфар//
Анаприлин//
Клофелин//
Гипотиазид//
Метопролол

***
Если у больного с гипертонической болезнью, ЧСС-110, какие препараты Вы предпочтете://


Эналаприл//
Дихлотиазид//
Празозин//
Коринфар//
+Дилтиазем

***
Если у больного со стабильной стенокардией ЧСС-100, АД - 130/80, то какие препараты следует предпочесть://


Амлодипин//
Нифедипин//
+Дилтиазем//
Дипиридамол//
изосорбид динитрат
***
Если у больного со стабильной стенокардией ЧСС-60, АД-150/80, то какие препараты следует предпочесть://
+Нифедипин//
Дилтиазем//
Верапамил//
Анаприлин//
Метопролол

***
Отметить гипотензивное средство для диагностики феохромоцитомы://


Клофелин//
натрия нитропруссид//
Метилдофа//
+Фентоламин//
Нифедипин

***
Какие гипотензивные средства вызывают брадикардию, повышение моторики кишечника и заложенность носа://


ИАПФ//
антагонисты кальция//
бета-адреноблокаторы //
ганглиоблокаторы//
+симпатолитики

***
При бронхиальной астме и артериальной гипертонии противопоказан://


резерпин //
клофелин //
Дихлотиазид//
+анаприлин //
Каптоприл

***
При язвенной болезни желудка и артериальной гипертонии противопоказан://


+резерпин //
клофелин //
дихлотиазид//
анаприлин //
каптоприл

***
Какие антигипертензивные препараты не следует назначать совместно://


дихлотиазид + эналаприл //
дихлотиазид +анаприлин//
эналаприл+амлодипин//
амлодипин+дихлотиазид//
+анаприлин+верапамил

***
Для систематического лечения гипертонической болезни целесообразно назначать://


пентамин //
резерпин//
фентоламин //
клофелин//
+метопролол

***
Преимущественно при гипертензивных кризах на амбулаторном этапе применяют://


+каптоприл под язык//
бензогексоний в/в//
анаприлин под язык//
пентамин в/в//
резерпин внутрь

***
Для купирования желудочковых аритмий при инфаркте миокарда применяют://


натрия нитропруссид //
пентамин //
новокаинамид//
дибазол //
+лидокаин

***
Какое ЛС понижает венозное давление и уменьшает преднагрузку на сердце://


анаприлин //
верапамил//
+нитроглицерин//
дипиридамол //
амиодарон

***
Какое антиангинальное средство уменьшает потребность сердца в кислороде за счет уменьшения силы и частоты сердечных сокращений://


+анаприлин //
нифедипин//
коринфар//
нитросорбид //
изокет

***
Для купирования желудочковых аритмий, возникающих при инфаркте миокарда, целесообразно назначать://


хинидин //
Анаприлин//
верапамил //
+лидокаин //
Эфедрин

***
Противоаритмические средства, эффективные как при желудочковых, так и при наджелудочковых аритмиях: //


верапамил //
Дилтиазем//
сердечные гликозиды //
+амиодарон //
лидокаин

***
Cредства, применяемые главным образом при наджелудочковых аритмиях://


хинидин //
+верапамил //
Лидокаин//
новокаинамид //
Кордарон

***
Какая лекарственная форма бета-стимуляторов наиболее безопасна у больных с бронхоспастическим синдромом и стенокардией://


Порошки//
Свечи//
Таблетки//
растворы для парентерального введения//
+Аэрозоль

***
Для купирования приступа бронхиальной астмы препаратом первого выбора является://


беклометазон //
+Сальбутамол//
Кетотифен//
эуфиллин внутримышечно//
кромогликат натрия

***
Для купирования приступа стенокардии применяют://


Сустак//
Нитронг//
+Нитроглицерин//
Верапамил//
Дилтиазем
***
Из антиангинальных средств при сочетании ИБС и артери­альной гипертонии показан://
Сустак//
+Верапамил//
Каптоприл//
Курантил//
Эналаприл

***
Наиболее вероятно развитие толерантности при применении://


Тринитролонга//
Сустака//
+изосорбида динитрата//
сублингвального нитроглицерина//
Нитронга

***
Наиболее объективным методом контроля эффективности антиангинальной терапии является://


мониторирование ЭКГ по Холтеру//
контроль уровня липидов крови//
суточное мониторирование АД//
+парные нагрузочные пробы ВЭМ//
оценка частоты ангинозных болей за сутки
***
Препаратом выбора при стенокардии у больного с брадикардией является://
+нифедипин ретард //
пропранолол//
Верапамил//
Дилтиазем//
Метопролол

***
Препаратом выбора при стенокардии у больного с сердечной недостаточностью является://


Верапамил//
Коринфар//
Дилтиазем//
Ацебуталол//
+Нитросорбид

***
Методом контроля за эффективностью гипотензивной терапии является://


суточное мониторирование ЭКГ//
разовые измерения АД//
динамика липидного спектра//
+суточное мониторирование АД//
измерение АД в орто- и клиностазе

***
Укажите побочный эффект нифедипина://


Брадикардия//
Бронхоспазм//
+отеки голеней и стоп//
развитие AV-блокады//
Ульцерогенность

***
Укажите препарат, механизмом гипотензивного действия которого является блокада а-рецепторов://


Клофелин//
+Празозин//
Пропранолол//
Капотен//
Верапамил

***
Препаратом выбора у больного с гипертонией и застойной недостаточностью кровообращения является://


+Эналаприл//
Нитросорбид//
Клофелин//
Адельфан//
Пентамин

***
Назовите группу препаратов, комбинация которых с ингибиторами АПФ противопоказана://


антагонисты кальция//
Нитраты//
мощные диуретики//
+калийсберегающие диуретики//
бета-блокаторы

***
Оптимальный препарат для длительной гипотензивной терапии должен://


влиять на обмен веществ//
иметь рикошетные реакции//
обладать синдромом отмены//
+иметь стабильную концентрацию в крови//
вызывать ортостатические реакции

***
Укажите гипотензивный препарат, не влияющий на активность ренин-ангиотензин-альдостероновой системы://


ингибитор АПФ//
бета-блокатор//
ингибитор альдостерона//
блокатор рецепторов к ангиотензину//
+антагонист кальция

***
Укажите гипотензивный препарат, который необходимо с осторожностью применять при сочетании сахарного диабета и гипертонической болезни://


Верапамил//
Лозартан//
Дилтиазем//
+Гипотиазид//
Эналаприл

***
Укажите предпочтительный путь введения лекарственных препаратов при застойной сердечной недостаточности://


Ректальный//
Сублингвальный//
Внутрь//
+Внутривенный//
Накожный

***
Прямым положительным инотропным эффектом не обладает://


Дигоксин//
Гидралазин//
Норадреналин//
+Эуфиллин//
Допамин

***
Укажите наиболее эффективный диуретик для лечения застойной сердечной недостаточности с развитием вторичного гиперальдостеронизма://


этакриновая кислота (урегит)//
хлорталидон (гигротон)//
ацетазоламид (диакарб)//
+спиронолактон (верошпирон)//
Триампур

***
Укажите рациональную комбинацию диуретиков://


петлевой + умеренный//
петлевой + умеренный + калийсберегающий//
петлевой + осмотический//
+умеренный + калийсберегающий//
осмотический + умеренный

***
Выберите диуретик для лечения отека легких://


+Фуросемид//
Гипотиазид//
Триампур//
Спиронолактон//
Маннитол

***
Укажите основной терапевтический эффект нитросорбида у больных с сердечной недостаточностью://


положительное хронотропное действие//
+расширение преимущественно артериол и снижение постнагрузки//
расширение преимущественно венул и снижение преднагрузки//
прямое положительное инотропное действие//
повышение диуреза и снижение преднагрузки

***
Назовите препарат, оказывающий органопротективный эффект и повышающий выживаемость больных с тяжелой сердечной недостаточностью://


гидралазин (апрессин)://
Верапамил//
Дигоксин//
Нитросорбид//
+Каптоприл

***
Назовите препарат, имеющий холинолитические побочные эффекты://


Лидокаин//
+ Хинидин//
амиодарон (кордарон)//
Верапамил//
Этмозин

***
Укажите группу антиаритмических средств, оказывающих антифибрилляторное действие://


сердечные гликозиды//
антагонисты кальция (IV группа)//
этмозин и другие препараты Iв группы//
+амиодарон, бретилия тозилат (III группа)//
хинидин, прокаинамид и другие препараты Iа группы
***
Назовите препарат, который может спровоцировать приступ предсердной тахиаритмии при синдроме WPW://
Новокаинамид//
Амиодарон//
+Верапамил//
Пропранолол//
Этмозин

***
Укажите препарат, которому свойственен антиаритмический эффект://


Аймалин//
Мексилетин//
Пропафенон//
Пропранолол//
+все перечисленные

***
Назовите препарат, антиаритмическое действие которого связано с удлинением атриовентрикулярной проводимости://


Хинидин//
Лидокаин//
Этмозин//
+Дигоксин//
Прокаинамид

***
Препаратом выбора при наличии атипичных возбудителей (микоплазма, хламидии) является://


+Эритромицин//
Метронидазол//
Гентамицин//
Карбенициллин//
Цефуроксим

***
Препаратами выбора при наличии атипичных возбудителей (микоплазма, хламидии) являются://


+Макролиды//
Пенициллины//
Аминогликозиды//
Цефалоспорины//
Сульфаниламиды

***
Укажите антибактериальный препарат, обладающий наибольшей антианаэробной активностью://


Эритромицин//
Ампициллин//
Тетрациклин//
Гентамицин//
+Цефотетан

***
Дисбактериоз кишечника вызывают все перечисленные антибактериальные препараты, кроме://


полусинтетических пенициллинов//
тетрациклинов//
Фторхинолонов//
оральных цефалоспоринов//
+макролидов

***
Нефротоксичными являются следующие антибактериальные препараты, кроме://


Гентамицина//
Карбенициллина//
+Эритромицина//
Цефалоридина//
Ванкомицина

***
Укажите антибактериальный препарат, малоактивный в отношении пневмококка://


Азитромицин//
Пенициллин//
Цефтриаксон//
+Ципрофлоксацин//
Левомицетин

***
Выберите комбинацию антибактериальных препаратов, обладающих синергизмом действия и безопасностью://


пенициллины + тетрациклины//
пенициллины + цефалоспорины//
пенициллины + макролиды//
+пенициллины + аминогликозиды//
пенициллины + сульфаниламиды

***
Хорошо проникают через гематоэнцефалический барьер следующие антибактериальные препараты://


Пенициллины//
Макролиды//
Тетрациклины//
Аминогликозиды//
+цефалоспорины III генерации

***
Препаратом выбора при крупозной пневмонии является://


Ципрофлоксацин//
Доксициклин//
Метициллин//
Цефотаксим//
+пенициллин

***
Препаратом выбора при фарингите является://


Цефаклор//
Тетрациклин//
Цефтазидим//
Офлоксацин//
+пенициллин

***
Новое поколение макролидных антибиотиков имеет следующие преимущества, кроме://


высокой биодоступности//
широкого спектра антибактериального действия//
бактерицидного действия//
длительного Т1/2 //
+почечного пути экскреции

***
Фторхинолоны отличаются от хинолонов следующими свойствами, кроме://


широкого антибактериального спектра действия//
+бактериостатического действия//
высокой пенетрации в ткани//
постантибактериального эффекта//
перорального способа введения

***
К ингаляционным кортикостероидам относится://


Гидрокортизон//
+Беклометазон//
Преднизолон//
Полькортолон//
Дексаметазон
***
К селективным бета2-агонистам длительного действия относится: //
Флутиказон//
+Сальметерол//
Сальбутамол//
Фенотерол//
Тербуталин

***
Для купирования приступа бронхиальной астмы применяется://


ипратропиум бромид//
теопек//
интал//
+сальбутамол//
Кетотифен

***
Мембраностабилизатором тучных клеток для приема внутрь является://


+Кетотифен//
недокрамил натрия//
хромогликат натрия//
супрастин//
ипратропиума бромид

***
При синдроме «запирания» легких применяют://


Сальбутамол//
Фенотерол//
Теопек//
Эуфиллин//
+адреналин

***
К муколитическим средствам относится://


Кодеин//
хромогликат натрия//
+ацетилцистеин//
сальметерол//
теофиллин

***
При одновременном применении повышает концентрацию теофиллина в крови://


+Ципрофлоксацин//
Пенициллины//
Цефтриаксон//
Гентамицин//
Бисептол

***
При одновременном применении снижает концентрацию теофиллина в крови://


Пефлоксацин//
Цефазолин//
+Рифампицин//
Эритромицин//
Ампиокс

***
При повышенном давлении в легочной артерии у больного с бронхиальной астмой показан://


Верапамил//
+Нифедипин//
Дигоксин//
Интал//
Беклометазон

***
При бронхиальной астме на фоне хронического бронхита показан://


+ипратропиума бромид//
адреналин//
эфедрин//
кетотифен//
супрастин

***
Кортикостероиды усиливают токсичность://


+сердечных гликозидов//
теофиллина//
спиронолактона//
верапамила//
Оксикамов

***
Эффект глюкокортикоидов снижает://


+Фенобарбитал//
Циметидин//
Аспирин//
Ортофен//
Амиодарон

***
Наиболее выраженными анальгетическими свойствами обладает://


Аспирин//
+Кетопрофен//
Бутадион//
Напроксен//
Индометацин

***
Механизмом развития ульцерогенного эффекта неспецифических противовоспалительных средств является://


повышение кислотности желудочного сока//
снижение образования слизи//
+снижение синтеза простагландинов в слизистой оболочке желудка//
снижением репарации слизистой оболочки//
увеличение образования гастрина

***
Наиболее выраженными противовоспалительными свойствами обладает://


Бутадион//
Анальгин//
Пироксикам//
Кетопрофен//
+вольтарен

***
Ранним побочным эффектом кортикостероидов является://


развитие катаракты//
+стероидная язва//
Остеопороз//
кушингоидный синдром//
стероидный диабет

***
Укажите эффект, не характерный для кортикостероидов://


Противовоспалительный//
Противоаллергический//
Противошоковый//
+Анальгетический//
Иммунодепрессивный

***
Выберите препарат, максимально подавляющий секрецию соляной кислоты://


Пирензепин//
Циметидин//
Карбеноксолон//
Антациды//
+Омепразол

***
Максимальное количество побочных эффектов среди Н2-блокаторов вызывает://


+Циметидин//
Роксатидин//
Низатидин//
Ранитидин//
Фамотидин

***
Прокинетиком желудочно-кишечной моторики является://


+Цизаприд//
Пирензепин//
Ранитидин//
Омепразол//
Энпростил

***
Какой побочный эффект не вызывает циметидин://


Диарея//
Гипертрансаминаземия//
Депрессия//
+расстройство зрения//
Гинекомастия

***
Наиболее длительное антисекреторное действие имеет://


Мизопростол//
Гастроцепин//
Циметидин//
Фамотидин//
+омепразол

***
Бактерицидным действием против H.pylori обладает://


сукралфат (вентер)//
+субцитрат висмута (де-нол)//
альмагель//
фамотидин//
карбенексолон

***
Синтетические аналоги простагландинов (энпростил, мизопростол) не вызывают://


антисекреторное действие//
секрецию бикарбонатов//
повышение образования слизи//
репарантное действие//
+бактерицидное действие в отношении H.pylori

***
Угнетает метаболизм других препаратов://


Омепразол//
Карбеноксолон//
+Циметидин//
Фамотидин//
Гастроцепин

***
Продолжительность антисекреторного действия омепразола составляет://


2-4 часа//
8-10 часов//
16-20 часов//
24 часа//
+3 суток

***
Синдром отмены вызывают://


аналоги простагландинов (энпростил)//
антациды//
омепразол//
атропин//
2-блокаторы

***
Укажите препарат, являющийся мощным стимулятором слизеобразования в желудке://


+Карбеноксолон//
Платифиллин//
де-нол//
омепразол//
метоклопрамид

***
При наличии почечной недостаточности требуется коррекция доз://


аналогов простагландинов//
омепразола//
2-блокаторов//
Сукральфата//
Холиноблокаторов

***
Укажите антисекреторный препарат, блокирующий «протоновый насос»://


Метоклопрамид//
Карбеноксолон//
Пирензепин//
Сукральфат//
+омепразол

***
К ингаляционным м-холинолитикам относится://


Сальбутамол//
кромогликат натрия//
+ипратропиум-бромид//
Беклометазон//
Флунисолид

***
К быстрому и пролонгированному бронходилатирующему эффекту приводит сочетанное применение ипратропиум-бромида и ://


кромогликата натрия//
+бета-адреномиметиков//
М-холинолитиков//
Н1-гистаминоблокаторов//
Адреналин

***
При остром отравлении ацетилсалициловой кислотой показано проведение форсированного диуреза с включением://


спиронолактона (верошпирона)//
+ Маннитола//
Фуросемида//
ацетазоламида (диакарба)//
гипотиазида

***
При острой сердечной недостаточности с отеком легких из указанных диуретиков необходимо выбрать://


+Фуросемид//
Маннитол//
спиронолактон (верошпирон)//
ацетазоламид (диакарб)//
гипотиазида

***
Гентамицин является антибиотиком из группы://


Пенициллинов//
+ Аминогликозидов//
Макролидов//
Тетрациклинов//
Гликопептидов

***
Ванкомицин является высокоактивным антибиотиком против://


Хламидий//
синегнойной палочки//
+стафилококков//
гемофильной палочки//
анаэробов

***
Кетоконазол высокоактивен против://


стафилококков и стрептококков//
простейших//
синегнойной палочки//
Анаэробов//
+Грибков

***
Для лечения остеомиелита антибиотиком выбора является://


Пенициллин//
Макролид//
Полимиксин//
+Ципрофлоксацин//
Метронидазол

***
Для лечения перитонита антибиотиком выбора является://


Эритромицин//
Пенициллин//
Рифампицин//
Тетрациклин//
+Гентамицин

***
Для лечения бактериального эндокардита антибиотиком выбора является://


+пенициллин //
Эритромицин//
Цефоперазон//
Имипенем//
Азтреонам

***
Азтреонам высокоактивен только против://


Грибков//
Анаэробов//
всех грамположительных бактерий//
+всех грамотрицательных бактерий//
синегнойной палочки

***
Побочные эффекты в виде аллергических реакций наиболее часто возникают при лечении://


+Пенициллинами//
Аминогликозидами//
Макролидами//
Цефалоспоринами//
левомицетином (хлорамфениколом)

***
Самым частым побочным эффектом всех нестероидных противовоспалительных средств (НПВС) является://


+Гастротоксический//
Гепатотоксический//
Нефротоксический//
Кардиотоксический//
Гематотоксический

***
Из перечисленных нестероидных противовоспалительных средств однократно в сутки применяется только://


Диклофенак//
Ибупрофен//
+Пироксикам//
ацетилсалициловая кислота//
индометацин

***
Основным механизмом действия нестероидных противовоспалительных средств (НПВП) является://


+Антипростагландиновый//
Антибактериальный//
стабилизация мембран лизосом//
торможение реакции "антиген + антитело"//
Антифосфолипазный

***
Какие эффекты характерны для сердечных гликозидов://


учащение сокращений сердца //
+усиление сокращений сердца //
облегчение атриовентрикулярной проводимости //
снижение автоматизма сердца//
снижение возбудимости миокарда

***
При действии сердечных гликозидов снижается://


сила сердечных сокращений //
атриовентрикулярная проводимость//
+автоматизм сердца//
артериальное давление//
возбудимость миокарда

***
Средства, повышающие сократимость миокарда://


+Дофамин//
унитиол //
калия хлорид //
кордиамин//
эуфиллин

***
Показания к применению сердечных гликозидов://


+тахиаритмическая форма мерцательной аритмии предсердий //
отеки почечного происхождения //
острая сердечная недостаточность //
атриовентрикулярная блокада//
фибрилляция желудочков

***
Характерные побочные эффекты сердечных гликозидов: //


+Тошнота//
тахикардия //
Лейкопения//
повышение АД//
отеки на лодыжках

***
При интоксикации сердечными гликозидами назначают://


+калия хлорид //
кальция хлорид //
натрия хлорид //
фуросемид//
дофамин

***
Средство, применяемое при гликозидной интоксикации, связывающее ионы Са2+://


дифенин //
+динатриевая соль этилендиаминтетрауксусной кислоты (ЭДТА)//
унитиол //
аспаркам//
калия хлорид

***
Дигоксин отличается от дигитоксина://


более медленным выведением//
более медленным развитием эффекта //
+меньшей способностью к кумуляции//
большей продолжительностью действия //
более полной абсорбцией в желудочно-кишечном тракте

***
Наиболее полно всасывается в желудочно-кишечном тракте://


+дигитоксин //
строфантин//
коргликон //
дигоксин//
целанид

***
Препарат с наиболее выраженной способностью к кумуляции://


+дигитоксин //
строфантин//
коргликон //
дигоксин //
целанид

***
Кардиальные действия терапевтических доз сердечных гликозидов://


+положительное инотропное//
положительное дромотропное//
отрицательное батмотропное//
положительное хронотропное//
отрицательное инотропное

***
Противопоказания к назначению сердечных гликозидов://


Блокада левой ножки пучка Гиса//
+АV блокада II степени//
Миастении//
Бронхиальная астма//
Пароксизм мерцательной тахиаритмии

***
Абсолютное противопоказание к назначению сердечных гликозидов://


+Дигиталисная интоксикация//
Гипомагниемия//
Алкалоз//
Гипернатриемия//
Выраженный митральный стеноз

***
Укажите противокашлевые средства://


+кодеин//
натрия гидрокарбонат//
трипсин //
калия йодид//
апоморфин

***
Укажите отхаркивающие средства://


+настой травы термопсиса//
кодеин//
либексин//
ацетицистеин//
амроксол

***
Механизм действия бромгексина://


Стимулирует кашлевой механизм//
+Снижает вязкость мокроты//
Действует как отхаркивающее вещество рефлекторного типа//
Снижает активность мукоцилиарного транспорта//
Эффект связан с отщеплением брома

***
Укажите отхаркивающие средства прямого действия://


+трипсин//
настой травы термопсиса//
апоморфин//
корень солодки//
терпингидрат

***
Отхаркивающее средство рефлекторного действия://


трипсин//
калия иодид//
натрия гидрокарбонат//
+настой травы термопсиса//
Дезоксирибонуклеаза

***
Укажите бронхолитическое средство из группы м-холиноблокаторов: //


адреналин //
эуфиллин //
изадрин //
салбутамол//
+атровент

***
Укажите бронхолитические средства из группы адреномиметиков://


атропин //
эуфиллин //
+сальбутамол //
зиулетон//
тиотропия бромид

***
Укажите бронхолитик миотропного действия://


атропин //
aдреналин //
фенотерол //
изадрин//
+теофиллин

***
Препарат, подавляющий высвобождение медиаторов из тучных клеток и используется только для профилактики бронхоспазма://


Адреналин//
Эуфиллин//
+Кромолин-натрия//
Супрастин//
Атропин

***
Укажите препарат, расширяющий бронхи за счет стимуляции бета2 - адренорецепторов://


атропин //
зиулетон//
эуфиллин //
тиотропия бромид//
+фенотерол
***
Укажите препарат, расширяющий бронхи за счет блокады м - холинорецепторов://
зиулетон //
эуфиллин //
+атровент //
Изадрин//
сальбутамол

***
Укажите препарат, устраняющий бронхоспазм за счет миотропного спазмолитического действия://


адреналин //
эфедрин //
атропин //
+эуфиллин //
Изадрин
***
Механизм действия интала и задитена://
Активация бета-2-адренорецепторов//
+Стабилизация мембран тучных клеток//
Блокирование гистаминовых рецепторов//
Блокирование альфа-адренорецепторов//
Блокирование ацетилхолиновых рецепторов

***
Протипоказания к введению адреналина для купирования бронхоспазма://


+Анамнестические сведения о малой эффективности адреномиметиков//
Гормоно-зависимая бронхиальная астма//
Аспириновая астма//
Бронхиальная астма физического усилия//
Астма пищевого генеза

***
Какие ЛС могут вызвать бронхоспазм у больных бронхиальной астмой://


+Обзидан//
Изадрин//
Фентоламин//
Коринфар//
Эналаприл

***
Какие бронхолитики в меньшей степени влияют на деятельность сердца://


Адреналин//
Эфедрин//
Алупент//
+Формотерол//
Эуфиллин

***
Не вызывает выраженную тахикардию://


Адреналин//
изадрин //
эуфиллин//
атропин//
+салбутамол
***
Эффективно используются для купирования бронхоспазма://
Интал//
+Сальбутамол//
Беклометазон//
Прозерин//
Дротаверин

***
ЛС - наиболее эффективные при астме физического усилия://


Гормоны//
+Антагонисты кальция//
Ингибиторы лейкотриеновых рецепторов//
М-холиноблокаторы//
Ингибиторы фосфодиэтеразы

***
Укажите мембраностабилизирующие препараты 1 А группы ://


Лидокаин//
дифенин //
верапамил //
дигоксин //
+хинидин

***
Мембраностабилизирующие препараты 1 Б группы ://


Этмозин//
Верапамил//
+Лидокаин//
Хинидин//
Новокаинамид

***
Укажите блокатор кальциевых каналов - антиаритмик://


новокаинамид //
метопролол //
+верапамил //
хинидин //
Коринфар

***
Укажите средство, ослабляющее адренергические влияния на сердце://


+метопролол //
верапамил //
лидокаин //
хинидин //
Атропин

***
Укажите средство, ослабляющее холинергические влияния на сердце://


изадрин //
анаприлин //
+Атропин//
амиодарон //
Верапамил

***
Какие средства действуют преимущественно на желудочки сердца://


хинидин //
Новокаинамид//
Амиодарон//
+лидокаин //
Анаприлин

***
Укажите средства, действующие на все отделы сердца://


+амиодарон //
Лидокаин//
дифенин //
верапамил//
Дилтиазем

***
Выражено снижает атриовентрикулярную проводимость://


+верапамил //
Кордарон//
Коринфар//
атропин //
лидокаин

***
Атриовентрикулярную проводимость улучшают://


лидокаин //
дифенин//
+атропин //
верапамил//
амиодарон

***
Гипотензия наиболее вероятна при применении://


+хинидина//
этмозина//
дифенина//
Лидокаина//
атропина

***
Синдром системной красной волчанки может возникнуть при применении://


лидокаина//
дигоксина//
+новокаинамида//
верапамила//
амиодарона

***
Отложение кристаллов препарата в роговице - характерный побочный эффект://


Хинидина//
новокаинамида//
дигоксина //
верапамила//
+амиодарона

***
К какой группе ЛС относится каптоприл://


Гипотензивное средство центрального действия//
Гипотензивное средство миотропного действия//
Блокатор кальциевых каналов//
+Ингибитор ангиотензин-конвертирующего фермента//
Диуретик

***
Отметить гипотензивные средства центрального действия://


дибазол//
дихлотиазид//
+метилдофа //
анаприллин//
Каптоприл

***
Отметить гипотензивные средства миотропного действия://


+ дибазол//
дихлотиазид//
метилдофа//
анаприллин//
Каптоприл

***
Верапамил отличается от анаприлина тем, что://


уменьшает частоту сердечных сокращений//
затрудняет атриовентрикулярную проводимость//
уменьшает силу сердечных сокращений//
+увеличивает коронарный кровоток//
уменьшает потребность сердца в кислороде

***
Отметить блокатор альфа1-адренорецепторов://


метопролол//
+празозин//
фентоламин//
метилдофа//
лабеталол

***
Для купирования приступа стенокардии лучше использовать://


изокет//
анаприлин//
+нитроглицерин //
амиодарон//
нитросорбид

***
Отметить блокатор ангиотензиновых рецепторов://


+лозартан//
метопролол//
метилдофа//
коринфар//
каптоприл

***
Какой антигипертензивный препарат вызывает рефлекторную тахикардию://


+коринфар//
Метопролол//
амиодарон//
анаприлин//
Верапамил

***
Отметить препарат, уменьшающий образование и выделение ренина://


Каптоприл//
саралазин //
+Анаприлин//
Апрессин//
фенигидин

***
Фармакодинамика клофелина://


блокирует симпатические ганглии//
усиливает выведение из организма ионов натрия и воды//
блокирует альфа2-адренорецепторы//
+стимулирует альфа2-адренорецепторы ЦНС//
вызывает рефлекторную тахикардию

***
Психическую депрессию может вызвать://


Каптоприл //
верапамил//
+Клофелин //
Дихлотиазид//
Анаприлин

***
Перечислите побочные действия, вызываемые гидрокарбонатом натрия://


повышение внутриглазного давления //
+системный алкалоз //
системный ацидоз//
задержка мочеиспускания//
Диарея

***
Назовите основной механизм действия гастроцепина://


блокада всех М-холинорецепторов//
+селективная блокада мускариновых рецепторов слизистой желудка//
блокада Н2-гистаминорецепторов//
блокада Н-холинорецепторов//
блокада центральных М-холинорецепторов

***
Механизмы терапевтического действия контрикала при остром панкреатите://


номализует отток панкреатического сока//
подавляет секрецию панкреатического сока//
+ингибирует протеолитическую активность ферментов в крови//
подавляет секрецию соляной кислоты//
подавляет секрецию гастрина

***
Характерные признаки солевых слабительных://


раздражают хеморецепторы кишечника//
+раздражают механорецепторы за счет увеличения объема кишечного содержимого//
начало действия через 12 - 16 ч после приема//
действуют на толстый кишечник//
применяют для длительного лечения запоров

***
Слабительные, преимущественно действующие на толстый кишечник://


магния сульфат//
Фенолфталеин//
натрия сульфат//
+кора крушины//
касторовое масло

***
Назовите препараты, обладающие вяжущими свойствами://


Атропин//
Альмагель//
+Танин//
Гастроцепин//
Денол

***
Средства заместительной терапии, применяемые при недостаточности экскреторной функции желудка://


Оксафенамид//
Трипсин//
Контрикал//
+желудочный сок//
Церукал

***
Лекарственные средства, снижающие экскреторную функцию желудка://


Антациды//
М-холиномиметики//
Спазмолитики//
2-гистаминоблокаторы//
ингибиторы фосфодиэстеразы

***
Препараты, усливающие моторику ЖКТ://


Антациды//
+М-холиномиметики//
Спазмолитики//
Н2-гистаминоблокаторы//
ингибиторы фосфодиэстеразы

***
Препараты, снижающие моторику ЖКТ://


Антациды//
М-холиномиметики//
+Спазмолитики//
антихолинэстеразные препараты//
ингибиторы фосфодиэстеразы

***
Отметить холеретик://


магния сульфат//
+Аллохол//
Папаверин//
Атропин//
ксилит

***
Отметить холекинетик://


+магния сульфат//
Холензим//
кислота дегидрохолевая//
Аллохол//
Оксафенамид

***
Какие лекарствение средства могут вызвать задержку мочи://


Ранитидин//
+Атропин//
Омепразол//
Циметидин//
Гастроцепин

***
Отметить антацид с системным действием://


Гидроокись магния//
Окись магния//
Гидроокись алюминия//
+Гидрокарбонат натрия//
Оксид кальция

***
Для стадии возбуждения при остром отравлении спиртом этиловым характерны://


+Эйфория//
улучшение психомоторных реакций//
улучшение памяти//
повышение работоспособности//
угнетение дыхания

***
Для агональной стадии при остром отравлении спиртом этиловым характерны://


Эйфория//
психомоторное возбуждение//
+коматозное состояние//
глубокое и ровное дыхание//
повышение мышечного тонуса

***
Меры помощи при остром отравлении спиртом этиловым://


+промывание желудка//
введение тетурама//
введение средств для наркоза//
введение нейролептиков//
введение ганглиоблокаторов

***
Для хронического отравления спиртом этиловым характерны://


повышение умственной работоспособности//
повышение физической работоспособности//
улучшение памяти//
+психическая и физическая деградация личности//
улучшение функций печени

***
Отметить барбитурат://


Хлоралгидрат//
Нитразепам//
+Фенобарбитал//
Бромизовал//
сибазон

***
Отметить снотворное алифатического ряда://


Сибазон//
этаминал-натрий//
нитразепам//
+Хлоралгидрат//
Фенобарбитал

***
Отметить производное бензодиазепина://


Фенобарбитал//
этаминал-натрия//
бромизовал//
+нитразепам//
Хлоралгидрат

***
Какой эффект не характерен для барбитуратов://


Седативный//
Снотворный//
+Жаропонижающий//
Наркотический//
Транквилизирующий

***
Наиболее выраженной способностью к материальной кумуляции обладает://


+Фенобарбитал//
Хлоралгидрат//
Нитразепам//
Бромизовал//
Сибазон

***
При тяжелом отравлении барбитуратами развиваются://


+коматозное состояние//
эйфория//
повышение рефлекторной активности//
гипертензия//
улучшение дыхательной функции

***
Отметить препарат, применяемый при малых припадках эпилепсии://


Дифенин//
+Этосуксимид//
Фенобарбитал//
Карбамазепин//
Сибазон

***
Средства для ингаляционного наркоза://


Пропанидид//
тиопентал-натрий//
+фторотан//
Кетамин//
натрия оксибутират

***
Средства для неингаляционного наркоза://


азота окись//
Фторотан//
+тиопентал-натрий//
Изофлюран//
Хлороформ

***
Средство для неингаляционного наркоза ультракороткого действия (3 - 10 мин.)://


тиопентал-натрий//
+Пропанидид//
натрия оксибутират//
Гексенал//
закись азота

***
Средства для неингаляционного наркоза короткого действия (15 - 30 мин)://


Калипсол//
+тиопентал-натрий//
Пропанидид//
натрия оксибутират//
закись азота

***
Средство для неингаляционного наркоза длительного действия (90 мин. и более)://


тиопентал-натрий//
+натрия оксибутират//
Пропанидид//
Гексенал//
Кетамин

***
Наиболее продолжительный наркотический эффект вызывает://


Кетамин//
Пропанидид//
Гексенал//
+натрия оксибутират//
тиопентал-натрий

***
Наиболее кратковременный наркотический эффект вызывает://


Гексенал//
тиопентал-натрий//
натрия оксибутират//
+Пропандид//
азота закись

***
Отметить наркотический анальгетик://


+морфин//
анальгин//
фенацетин//
Кеторолак//
Лорноксикам

***
Не является опиоидным наркотиком, но обладает центральной болеутоляющей активностью://


Фентанил//
Пентазоцин//
Трамадол//
+Кетамин//
Промедол

***
Отметить антагонист наркотических анальгетиков://


Бутадион//
Парацетамол//
Анальгин//
+Налоксон//
Трамадол

***
Агонистом опиатных рецепторов является://


+Морфин//
Пентазоцин//
Дроперидол//
Налоксон//
Фенацетин

***
В терапевтических дозах морфин вызывает://


+Эйфорию//
повышение артериального давления//
тахидикардию//
Мидриаз//
понижение тонуса гладкой мускулатуры внутренних органов

***
Налоксон устраняет угнетение дыхания, вызываемое://


средствами для наркоза//
снотворными средствами//
+наркотическими анальгетиками//
спиртом этиловым//
Нейролептиками

***
Основные побочные эффекты морфина://


+угнетение дыхания//
Рвота//
Понос//
повышение артериального давления//
тахикардия

***
При длительном применении наркотичеких анальгетиков развиваются://


+физическая лекарственная зависимость//
кумуляция//
тахифилаксия//
Резистентность//
повышение анальгетической активности

***
Побочные эффекты морфина://


нарушение слуха//
+миоз//
тахикардия//
агранулоцитоз//
изъязвление слизистой оболочки желудка

***
Для острого отравления морфином характерны://


коматозное состояние//
психомоторное возбуждение//
+Мидриаз//
Диарея//
повышение температуры тела

***
К какой группе препаратов относится аминазин://


психостимулирующее средство//
Антидепрессант//
ноотропное средство//
седативное средство//
+антипсихотическое средство (нейролептик)

***
К какой группе препаратов относится галоперидол://


ноотропное средство//
анксиолитик(транквилизатор)//
антидепрессант//
+антипсихотическое средство(нейролептик)//
психостимулирующее средство

***
К какой группе препаратов относится дроперидол://


+антипсихотическое средство(нейролептик)//
антидепрессант//
анксиолитик(транквилизатор)//
психостимулирующее средство//
седативное средство

***
К какой группе препаратов относится aмитриптилин://


антипсихотическое средство (нейролептик)//
анксиолитик (транквилизатор)//
психостимулирующее средство//
+антидепрессант//
ноотропное средство

***
К какой группе препаратов относится феназепам://


ноотропное средство//
психостимулирующее средство//
+анксиолитик(транквилизатор)//
антидепрессант//
седативное средство

***
К какой группе препаратов относится натрия бромид://


Антидепрессантам//
ноотропным средствам//
анксиолитикам(транквилизаторам)//
психостимулирующим средствам//
+седативным средствам

***
К какой группе препаратов относится препараты валерианы и пустырника относятся к://


анксиолитик(транквилизатор)//
ноотропное средство//
+седативное средство//
психостимулирующее средство//
антидепрессант

***
К какой группе препаратов относится кофеин://


ноотропное средство//
+психостимулирующее средство//
седативное средство//
анксиолитик (транквилизатор)//
Антидепрессант

***
К какой группе препаратов относится пирацетам://


+ноотропное средство//
психостимулирующее средство//
седативное средство//
анксиолитик(транквилизатор)//
Антидепрессант

***
При умственной недостаточности, связанной с органическими поражениями головного мозга применяют://


Аминазин//
Галоперидол//
Сиднокарб//
+Пирацетам//
сибазон

***
Сосудистый компонент воспалительной реакции состоит из://


+нарушения микроциркуляции с начальным спазмом и последующей вазодилатацией//
уменьшения проницаемости сосудистой стенки//
взаимодействия иммунных комплексов с рецепторами сосудистой стенки//
нарушения центральных регуляторных механизмов сосудистого тонуса//
повышения активности ренин-ангиотензин-альдостероновой системы

***
Гематологический компонент воспалительной реакции состоит из://


нарушения механизмов свертывания крови//
+миграции лейкоцитов в очаг воспаления//
разрушения эритроцитов//
высвобождения и активации медиаторов воспаления//
снижения активности клеток костного мозга

***
К НПВС относятся://


Дексаметазон//
+Ибупрофен//
Кенакорт//
Метипред//
Лефлюнамид

***
Стероидный противовоспалительный препарат://


Напросин//
+Дексаметазон//
Ибупрофен//
Индометацин//
Ортофен

***
Для НПВС не характерно://


неспецифичность противовоспалительного действия//
+высокая эффективность при системных заболеваниях соединительной ткани//
тормозящее влияние на агрегацию тромбоцитов//
повышенный риск развития эрозий в слизистой оболочке ЖКТ//
противовоспалительное, анальгетическое и жаропонижающее свойства

***
Побочные эффекты – не характерные для НПВП://


диспепсия, гастралгия, головная боль, крапивница//
диспепсия, головная боль, гастралгия, гипохромная анемия//
+запоры, дизурия, желтуха, токсикодермия//
конъюктивит, токсикодермия, анафилактический шок//
гастралгия, артериальная гипертензия, зудящая сыпь, головная боль

***
Наиболее безопасный жаропонижающий и анальгетический препарат у детей://


+Парацетамол//
Аспирин//
Амидопирин//
Анальгин//
Индометацин

***
Аллергические реакции при применении пиразолонов наиболее вероятны от момента начала терапии://


через 2 часа от первого приема//
в течение первых суток//
+через 2-4 суток//
через 7-14 дней от начала лечения//
через 1-2 месяца

***
Наиболее эффективное базисное средство для лечения ревматоиднго артрита://


Азатиоприн//
Делагил//
Кризанол//
+Метотрексат//
Сульфосалазин

***
Наиболее опасное осложнение при терапии декарисом (левамизолом)://


+Агранулоцитоз//
судорожный синдром//
обострение течения хронического гепатита//
псевдомембранозный колит//
обострение течения язвенной болезни желудка

***
Время проявления базисного действия сульфосалазинов://


после первого приема препарата//
через неделю//
через месяц//
+через 2-3 месяца//
через год от начала терапии

***
Фенилбутазон вызывает перекрестные аллергические реакции с://


Циклофосфамид//
Глюкокортикостероиды//
+Амидопирин//
D-пеницилламин//
Кризанол

***
Новообразования могут развиваться при длительном применении://


+Циклофосфамид//
Глюкокортикостероиды//
Сульфасалазин//
D-пеницилламин//
Кризанол

***
Остеопороз при длительной терапии вызывают://


Циклофосфамид//
+Глюкокортикостероиды//
Сульфасалазин//
D-пеницилламин//
Кризанол

***
При возникновении люпус-нефрита используют://


Сульфасалазин//
+Глюкокортикостероиды//
Амидопирин//
D-пеницилламин//
Кризанол

***
При отравлении cалициловой кислотой у детей://


может возникнуть тяжелый метаболический алкалоз//
+может проявляться синдромом Рейе//
эффективен гемодиализ//
необходимо подкислять мочу//
может проявляться синдромом серого ребенка

***
Относительными противопоказаниями к применению глюкокортикостероидов являются://


+язвенная болезнь желудка//
кардиогенный шок//
недавно перенесенный приступ бронхиальной астмы//
Поллиноз//
гипогликемические состояния

***
Механизмы подавления НПВС экссудативных процессов://


Уменьшение активности фибропластов и нарушение синтеза коллагена//
+Снижение проницаемости клеточных и субклеточных мембран//
Усилинение процессов окислительного фосфорилирования//
Hакопление макроэргических соединений в очаге воспаления//
Увеличение выработки простагландинов

***
Механизмы подавления НПВС процессов пролиферации://


+Уменьшение активности фибробластов и нарушение синтеза коллагена//
Уменьшение выработки биологических активных аминов//
Снижение проницаемости клеточных и субклеточных мембран//
Увеличение синтеза нуклеиновых кислот//
Повышение сосудистой проницаемости

***
НПВС, обладающие выраженным антиэкссудативным (противовоспалительным) эффектом://


Аспирин//
Реопирин//
+Индометацин//
Ибупрофен//
Фенацетин

***
ЛС, анальгетический эффект которых превосходит противовоспалительный://


Аспирин//
Индометацин//
+Метамизол//
Вольтарен//
Напроксин

***
Наиболее безопасный НПВС у больных с язвенной болезнью://


Индометацин//
Аспирин//
Бутадион//
Вольтарен//
+Нимесулид

***
НПВС, в наибольшей степени снижающие адгезию тромбоцитов://


+Аспирин//
Амидопирин//
Индометацин//
Реопирин//
Диклофенак

***
Механизм антиагрeгантного эффекта аспирина://


ингибирует фосфолипазу и тормозит образование эндоперекисей тромбоцитов//
+необратимо ингибирует синтез тромбоксана А2//
ингибирует синтез простациклина//
ингибирует липооксигеназу//
ингибирует фосфодиэстеразу

***
В сравнении с индометацином у аспирина более выражено://


анальгетическое действие//
угнетение синтеза простогландинов//
+антиагрегантное действие на тромбоциты//
жаропонижающее действие//
противовоспалительное действие

***
Как влияют глюкокоpтикоиды на углеводный обмен://


+Вызывают гипергликемию//
Увеличивают содержание гликогена в печени//
Угнетают глюконеогенез//
Угнетают гликогенолиз//
Снижают всасывание глюкозы из ЖКТ

***
Как влияют глюкокортикоиды на белковый обмен://


+Стимулируют распад белков в мышечной ткани//
Повышают содержание глобулинов//
Снижают содержание альбуминов//
Стимулируют рост скелета у детей//
Стимулируют процессы гликонеогенеза

***
Фармакодинамические эффекты ГК://


+Подавляют процессы экссудативного воспаления//
Оказывают иммуномодулирующее действие//
Способствуют увеличению мышечной массы//
Стимулируют рост скелета у детей, развитие гиперкальцинозов//
Уменьшают отложение жиров в органах и тканях

***
Наиболее быстро атрофию надпочечников вызывает://


Гидрокортизон//
Метилпреднизолон//
Триамсинолон//
Преднизолон//
+Дексаметазон

***
Похудание часто отмечается на фоне://


Кортизон//
Гидрокортизон//
Метилпреднизолон//
+Триамсинолон//
Преднизолон

***
Показания к назначению ГК при хронических гепатитах и циррозах://


Хронический персистирующий гепатит//
+Цирроз печени с явлениями выраженных аутоиммунных реакций//
Вторичный биллиарный цирроз//
Алкогольный цирроз печени//
Хронический токсический гепатит в стадии ремиссии

***
Перечислите показания к применению ГК при гломерулонефритах://


ХПН//
Паранеопластическая нефропатия//
Диабетическая нефропатия//
+Нефрит при системных заболеваниях (СКВ, геморрагический васкулит)//
Тромбоз почечных вен

***
Суточные дозы преднизолона, используемые для пульс-терапии://


До 100 мг//
100 мг - 200 мг//
200 мг - 500 мг//
+500 мг - 1000мг//
1000 мг - 1500 мг

***
Анаболики://


Прогестерон//
Этинилэстрадиол//
Диэтилстильбэстрол//
Тестостерон//
+Ретаболил

***
Показания к применению минералокортикоидов://


+болезнь Аддисона//
аутоиммуннные заболевания//
Лейкозы//
тяжелые аллергические реакции//
Кахексия

***
Препарат часто вызывающий мышечную слабость://


Кортизон//
Гидрокортизон//
Метилпреднизолон//
+Триамсинолон//
Преднизолон

***
Препарат, наиболее часто вызывающий стероидный диабет ://


Гидрокортизон//
Метилпреднизолон//
Триамсинолон//
Преднизолон//
+Дексаметазон

***
Полиеновый антибиотик местного действия://


+Нистатин//
Амфотерицин//
Фузидин//
Микосептин//
Гризеофульвин

***
Средство для лечения генерализованных микозов://


Нистатин//
Леворин//
+Амфотерицин//
Трихомицин//
Фузидин

***
Пенициллин широкого спектра действия://


Бензилпенициллин//
Метациклин//
+Ампициллин//
Азлоциллин//
Оксациллин

***
Пенициллиназоустойчивый пенициллин://


Бензилпенициллин//
Ампициллин//
Неомицин//
Тетрациклин//
+Оксациллин

***
Характерные признаки сульфаниламидов://


нарушают синтез клеточной стенки микроорганизмов//
являются антогонистами фолиевой кислоты//
+являются конкурентами парааминобензойной кислоты//
нарушают процесс синтеза тетрагидрофолиевой кислоты//
нарушают проницаемость клеточных мембран

***
Cульфаниламиды сверхдлительного действия (T1/2 > 48 ч)://


Сульфапиридазин//
Сульфадиметоксин//
Этазол//
Сульфазин//
+Сульфален

***
Нитрофуран, используемый только при инфекциях мочевыделительной системы://


Фурациллин//
Фуразолидон//
Фуразолин//
+Фурадонин//
Фурагин

***
Опасные комбинации антимикробных препаратов://


сульфаметаксозол + триметаприм//
нитрофураны + тетрациклины//
нитрофураны + полиеновые антибиотики//
нитрофураны + линкомицин//
+цефалоридин + аминогликозиды

***
Тромбоцитопению аллергического генеза вызывают://


+Левомицетин//
Пенициллин//
Цефалоспорины//
Карбепенемы//
Макролиды

***
Чаще всего поражение слухового нерва вызывают://


Пенициллины//
Цефалоспорины//
Карбепенемы//
Макролиды//
+Аминогликозиды

***
Чаще всего поражение вестибулярного аппарата вызывают://


Пенициллины//
Цефалоспорины//
Карбепенемы//
Макролиды//
+Аминогликозиды

***
Гепатотоксическое действие характерно для://


+Тетрациклины//
Аминогликозиды//
Цефалоспорины//
Карбепенемы//
Фторхинолоны

***
Нефротоксическое действие характерно для://


Карбепенемы//
Фторхинолоны//
Макролиды//
Тетрациклины //
+Аминогликозиды

***
Наиболее частое побочное действие бензилпенициллина://


Агранулоцитоз//
Анемия//
+аллергические реакции//
Дисбактериоз//
Глухота

***
Наиболее опасное действие левомицетина://


+Агранулоцитоз, анемия//
Аллергические реакции//
Дисбактериоз//
Глухота//
Нефропатии

***
Антимикробное действие сульфаниламидов ингибируется://


Пенициллиназой//
Диэтилкарбамазином//
+парааминобензойной кислотой//
аскорбиновой кислотой//
Триметопримом

***
Побочные эффекты эритромицина включают://


+холестатическую желтуху//
агранулоцитоз и тромбоцитопению//
Стоматиты//
кожные проявления, подобные герпесу//
вестибулярные нарушения

***
Поражение слухового нерва практически не вызывает://


Стрептомицин//
Неомицин//
Канамицин//
Мономицин//
+Амикацин

***
Диуретические препараты, действующие на уровне клубочка://


Гипотиазид//
Меркузал//
+Теофиллин//
Гигротон//
Урегит

***
Препараты, действующие на уровне проксимального канальцa://


Гипотиазид//
Буметамид//
+Диакарб//
Урегит//
Амилорид
***
Фармакокинетика - это://
+ изучение абсорбции, распределения, метаболизма и выведения лекарств//
изучение биологических и терапевтических эффектов лекарств//
изучение токсичности и побочных эффектов//
методология клинического испытания лекарств//
изучение взаимодействий лекарственных средств

***

Какие существуют виды абсорбции в ЖКТ://
Фильтрация//
Диффузия//
активный транспорт//
пиноцитоз//
+ все виды

***
Результатом высокой степени связывания препарата с белками плазмы является://


уменьшение Т 1/2//
повышение концентрации свободной фракции препарата//
+ снижение концентрации свободной фракции препарата//
лучшая эффективность препарата//
все перечисленное
***

Какие препараты больше подвергаются метаболизму в печени://


+ Липофильные//
Гидрофильные//
Липофобные//
имеющие кислую реакцию//
имеющие щелочную реакцию

***
Фармакодинамика включает в себя изучение следующего://


+ эффекты лекарственных средств и механизмы их действия//
абсорбцию и распределение лекарств//
метаболизм лекарств//
выведение лекарств//
всё перечисленное

***
Какие факторы влияют на кишечную абсорбцию://


рН желудочного сока//
васкуляризация//
моторика//
состояние микрофлоры кишечника//
+ все перечисленные

***
Препараты, оказывающие однонаправленное действие, называются://


Агонистами//
+ синергистами//
Антагонистами//
Миметиками//
литиками

***
Если первичную мочу сделать более щелочной, то://


экскреция слабых кислот уменьшится//
+ экскреция слабых кислот увеличится//
экскреция слабых оснований увеличится//
увеличится экскреция и слабых кислот, и слабых оснований//
экскреция не изменится

***
Какие препараты приводят к индукции ферментов в печени://


+ фенобарбитал//
Циметидин//
Амиодарон//
Эритромицин//
ципрофлоксацин

***
Что такое равновесная концентрация://


+ состояние, когда количество абсорбированного препарата равно количеству выводимого//
максимальная концентрация после введения препарата//
концентрация перед очередным введением препарата//
средняя концентрация после введения препарата//
правильных ответов нет

***
Укажите, где происходит всасывание большей части лекарств://


в ротовой полости//
в пищеводе//
в желудке//
+ в тонком кишечнике//
в толстом кишечнике

***
При заболеваниях печени увеличивается период полувыведения://


Дигоксина//
Гентамицина//
+ теофиллина//
Преднизолона//
ванкомицина

***
В щелочной моче увеличивается выведение://


Морфина//
новокаинамида//
+ барбитуратов//
Хинина//
варфарина

***
Какой из препаратов вызывает ощелачивание мочи://


аскорбиновая кислота//
аспирин//
тиазидные диуретики//
+ бикарбонат натрия//
эуфиллин

***
Укажите механизмы абсорбции препарата://


пиноцитоз//
пассивная диффузия//
активный транспорт//
фильтрация//
+ все перечисленное

***
Всасывание каких препаратов снижается при приеме пищи://


+ ампициллина//
Рамиприла//
Омепразола//
Ципрофлоксацина//
метопролола

***
Укажите антибиотик, вызывающий диарею, обусловленную прокинетическим действием://


+ эритромицин//
Амоксициллин//
Цефтриаксон//
Гентамицин//
ванкомицин

***
Наиболее токсичным из антибиотиков аминогликозидной группы является://


Амикацин//
Тобрамицин//
Гентамицин//
+ Неомицин//
Стрептомицин

***
Укажите наиболее ранний симптом передозировки сердечных гликозидов://


Рвота//
Гинекомастия//
+ Потеря аппетита//
Гиперкалиемия//
Тахикардия

***
Какой диуретик может вызвать развитие гирсутизма и гинекомастии://


гипотиазид//
+ спиронолактон//
Фуросемид//
Диакарб//
индапамид

***
Какие факторы влияют на период полувыведения://


+ почечный и печёночный клиренс//
биодоступность//
скорость распределения//
связь с белком//
экскреция почками

***
От чего зависит биодоступность://


всасывания и связи с белком//
+ всасывания и пресистемного метаболизма//
экскреции почками и биотрансформации в печени//
объёма распределения//
всего вышеперечисленного

***
Синдром «рикошета» - это://


снижение эффекта препарата при его отмене//
увеличения эффекта препарата при его отмене//
ответная реакция организма при отмене препарата//
+ развитие обратного эффекта при продолжении использования препарата//
все перечисленное

***
При патологии почек возникают следующие изменения фармакокинетики лекарств, кроме://


нарушения почечной экскреции//
увеличения концентрации препаратов в плазме//
уменьшения связывания с белками плазмы//
увеличения Т1\2//
+ уменьшения биодоступности

***
Хронический прием алкоголя приводит к://


увеличению абсорбции лекарств//
увеличению объема распределения лекарств//
+ замедлению метаболизма в печени//
снижению почечной экскреции//
снижению Т1\2
***
Развитие недостаточности какого витамина можно ожидать при длительном приеме пероральных контрацептивов://
А//
В1//
В2//
С//
+ фолиевой кислоты

***
Назначение какого витамина при беременности может приводить к порокам развития у ребенка://


+ А//
В1//
В2//
С//
РР

***
Какой препарат не рекомендуют применять при подагре из-за способности вызывать гиперурикемию://


нитроглицерин//
эритромицин//
пропранолол//
+ гипотиазид//
каптоприл

***
Какая из аритмий чаще возникает при интоксикации сердечными гликозидами?//


пароксизмальная наджелудочковая тахикардия//
предсердная экстрасистолия//
мерцание предсердий//
+ желудочковая экстрасистолия//
любая из перечисленных

***
Какой из режимов дозирования верошпирона является наиболее эффективным при ХСН://


доза, разделенная на 2 приёма//
+ вся доза утром однократно//
доза, разделенная на 3 приёма//
через день//
1 раз в неделю

***
При каком ритме сердца повышается эффект сердечных гликозидов?//


+ мерцательная тахикардия//
мерцательная брадикардия//
мерцательная нормосистолия//
синусовая брадикардия//
синусовая аритмия

***
Для какой группы диуретиков наиболее выражены метаболические нарушения://


+ Петлевые//
Ингибиторы карбоангидразы//
К+ - сберегающие//
Тиазидные//
Тиазидоподобные

***
Противоаритмическое действие сердечных гликозидов при мерцательной тахиаритмии обусловлено://


снижением автоматизма//
уменьшением возбудимости//
+ замедлением AV-проведения//
увеличением AV-проведения//
вазодилатацией

***
При развитии рефрактерности к мочегонным при ХСН какова максимальная доза фуросемида://


40 мг//
80 мг//
200 мг//
+ 500 мг//
ограничений доз нет

***
Какая группа лекарственных препаратов требует проведения лекарственного мониторинга://


стабилизаторы мембран тучных клеток//
+ теофиллины//
ингибиторы лейкотриенов//
ингаляционные бета-2-миметики//
все вышеперечисленные

***
Уменьшает выработку бронхиального секрета://


+ спирива//
амброксол//
эуфиллин//
бекламетазон//
пульмикорт

***
Беродуал – это://


фенотерол + будесонид//
+ фенотерол + ипратропия бромид//
будесонид + формотерол//
бекламетазон + ипратропия бромид//
сальбутамол + ипратропия бромид

***
При ингаляции какого препарата часто развивается кандидоз полости рта://


ипратропия бромида//
динатрия хромогликата//
+ беклометазона//
сальбутамола//
фенотерола

***
К побочным эффектам кетотифена относится://


бронхоспазм после приёма препарата//
+ сонливость//
нарушение сна и раздражительность//
раздражение слизистой оболочки дыхательных путей//
раздражение слизистой желудка

***
К побочным эффектам, требующим отмены ингаляционных кортикостероидов, относится://


развитие кандидоза полости рта//
дисфония//
спорадический кашель после ингаляции//
+ возникновение бронхоспазма после ингаляции//
сухость во рту

***
Клинический эффект от назначения ИГКС больным с бронхиальной астмой обычно отмечается через://


1-2 часа//
1-2 дня//
+ 5-7 дней//
4-6 недель//
2 месяца

***
При назначении теофиллина курильщику://


назначается стандартная доза препарата//
+ доза может быть увеличена//
доза может быть уменьшена//
назначения препарата необходимо избегать//
теофиллин курильщику противопоазан

***
Выберите ферментный муколитик://


бромгексин//
синекод//
амброксол//
карбоцистеин//
+ пульмозим

***
Выберите препарат для использования через небулайзер://


ацетилцистеин//
карбоцистеин//
месна//
+ амброксол//
бромгексин

***
Выберите отхаркивающий препарат, противопоказанный при артериальной гипертензии://


подорожник//
душица//
+ солодка//
алтей//
мать-и-мачеха

***
Укажите, что не относится к побочным эффектам всасывающихся антацидов://


метаболический алкалоз//
гипернатриемия//
синдром отмены//
+ гипофосфатемия//
гипертензия

***
Какой препарат подавляет ульцерогенное действие НПВС://


альмагель//
атропин//
фамотидин//
сукралфат//
+ мизопростол

***
Укажите цель назначения статинов в увеличенной дозе в первые дни инфаркта миокарда://


снижение общего холестерина//
снижение холестерина липопротеидов низкой плотности//
+ стабилизация атеросклеротических бляшек//
снижение преднагрузки//
снижение постнагрузки

***
Когда следует назначать статины://


утром перед завтраком//
+ вечером после ужина//
после обеда//
на ночь//
рекомендован прием через день

***
Выберите побочный эффект, характерный для статинов://


миелотоксичность//
угнетение ЦНС//
удлинение QT//
+ миопатии//
гипертензия

***
Для какого антибиотика характерен пенициллиновый тип развития резистентности://


эритромицин//
тетрациклин//
рифампицин//
гентамицин//
+ цефазолин

***
Какой побочный эффект характерен для линкомицина://


анемия//
снижение слуха//
полиневрит//
+ энтероколит//
азотемия

***
Какой из антибиотиков нужно назначать для воздействия на микроорганизмы, продуцирующие бета – лактамазы://


пенициллин//
ампициллин//
цефазолин//
+ амоксиклав//
амоксициллин

***
В чём преимущество карбапенемов по сравнению с другими бета – лактамными антибиотиками://


имеют 2 пути элиминации//
не вызывают аллергических реакций//
способны проникать через гематоэнцефалический барьер//
+ устойчивы к действию бета-лактамаз//
активны в отношении MRSA

***
Выберите препарат для лечения сепсиса, вызванного MRSA://


ампициллин//
цефазолин//
тиенам//
азитромицин//
+ линезолид

***
Выбелите наиболее эффективный антисинегнойный препарат://


амоксициллин//
цефотаксим//
имипенем//
+ меропенем//
гентамицин

***
Выберите цефалоспорин для ступенчатой антибактериальной терапии://


+ цефуроксим//
цефтазидим//
цефотаксим//
цефипим//
цефтриаксон

***
Отметьте антисинегнойный цефалоспорин://


цефазолин//
цефотаксим//
цефтрипаксон//
+ цефтазидим//
цефаклор

***
Выберите антимикробный препарат с узким терапевтическим коридором://


мидекамицин//
клиндамицин//
+ амикацин//
амоксициллин//
цефтриаксон

***
Какой антимикробный препарат противопоказан детям://


пенициллин//
цефтриаксон//
азитромицин//
+ ципрофлоксацин//
эритромицин

***
Выберите фторхинолон с антисинегнойной активностью://


норфлоксацин//
ломифлоксацин//
спарфлоксацин//
+ левофлоксацин//
ципрофлоксацин

***
Выберите сочетание антибиотиков, при котором увеличивается риск кардиотоксичности (увеличение QT)://


цефалоспорины+ макролиды//
цефалоспорины+ аминогликозиды//
фтохинолоны + цефалоспорины//
+ фторхинолоны+ макролиды//
пенициллины + макролиды

***
К какому классу противовирусных средств относится осельтамивир://


противогерпетические//
антиретровирусные//
+ противогриппозные//
противоцитомегаловирусные//
широкого спектра действия

***
Какой антибиотик вызывает гипопротромбинемию://


цефотаксим//
амоксициллин//
азитромицин//
+ цефоперазон//
гентамицин

***
Выберите антибиотик, обладающий дисульфирамоподобным эффектом://


цефазолин//
кларитромицин//
гентамицин//
+ цефоперазон//
ципрофлоксацин

***
Почему назначение НПВС в последнем триместре беременности нежелательно://


+ замедление родовой деятельности//
усиление сократимости миометрия//
мутагенное действие//
нарушение роста костей и зубов плода//
активация свертывающей системы

***
Взаимодействие НПВС с ингибиторами АПФ при лечении артериальной гипертензии приводит к://


+ ослаблению гипотензивного эффекта//
усилению гипотензивного эффекта//
угнетению ЦНС//
снижению гастротоксичности НПВС//
устранению антиагрегантного действия НПВС

***
Наиболее опасным препаратом в плане возникновения интерстициального нефрита является://


парацетамол//
фенацетин//
пироксикам//
напроксен//
+ индометацин

***
Какая связь с альбуминами плазмы у большинства НПВС://


+ высокая//
умеренная//
низкая//
НПВС связаны с глобулинами//
НПВС связаны с эритроцитами

***
Укажите недостаток высокоселективных блокаторов ЦОГ-2://


выраженный гастротоксический эффект//
кровоточивость//
+ ухудшение прогноза при ИБС//
нейро-мышечная блокада//
психические нарушения

***
Выберите неправильное утверждение: Отрицательное влияние ГКС на костно-мышечную систему проявляется://


миопатией//
патологическими переломами//
компрессионными переломами позвонков//
+ остеосклерозом//
остеопорозом

***
Негативное влияние ГКС на метаболизм проявляется://


гипогликемией//
снижением аппетита//
снижением липидов//
+ катаболизмом белков//
повышением либидо

***
Отрицательное влияние ГКС на эндокринную систему может проявляться://


ускорением полового созревания//
ускорением роста у детей//
усилением функции надпочечников//
усилением полового влечения//
+ дис- и аменореей

***
Угнетения гипоталамо-гипофизарно-надпочечниковой системы не происходит при приеме преднизолона в дозе://


30 мг/сутки//
40 мг/сутки//
20 мг/сутки//
10 мг/сутки//
+ 5 мг/сутки

***
Полное восстановление функции коры надпочечников при курсе ГКС 2-3 недели происходит://


сразу после отмены//
через 2-недели//
3-4 недели//
2-3 месяца//
+ 6-12 месяцев

***
В наибольшей степени угнетает гипоталамо-гипофизарно-надпочечниковоую систему://


преднизолон//
метилпреднизолон//
гидрокортизон//
+ дексаметазон//
бекламетазон

***
Системная фармакодинамическая терапия ГКС подразумевает://


10 мг преднизолона утром//
20 мг преднизолона по альтернирующей схеме//
40 мг в 2 приема в первой половине дня//
«пульс-терапия» 1000 мг\сутки в течение трех дней//
+ все перечисленное

***
При проведении пульс-терапии более предпочтителен://


преднизолон//
+ метилпреднизолон//
дексаметазон//
бетаметазон//
флютиказон

***
При назначении на длительный срок предпочтительнее использовать://


дексаметезон//
метилпреднизолон//
+ преднизолон//
триамцинолон//
беклометазон

***
Отметьте продолжительность действия дипроспана при внутримышечном введении://


+ 3-4 недели//
5-6 недель//
2-3 месяца//
0,5 года//
7 дней

***
Выберите характерный побочный эффект для антигистаминных II поколения://


угнетение ЦНС//
+ кардиотоксичность//
атония кишечника//
атония мочевого пузыря//
тахифилаксия

***
К III поколению антигистаминных не относится://


цетиризин//
левоцетиризин//
+ лоратадин//
фексофенадин//
дезлоратадин

***
Риск нарушений ритма при использовании астемизола возрастает при совместном применении с://


азитромицином//
пенициллином//
+ эритромицином//
цефатоксимом//
меропенемом

***
С каким из антигистаминных имеет перекрестную аллергическую реакцию эуфиллин://


цетиризин//
+ супрастин//
фексофенадин//
лоратадин//
димедрол

***
Выберите антигистаминный препарат с наименьшим угнетающим действием на ЦНС://


димедрол//
тавегил//
супрастин//
+ фенкарол//
диазолин

***
Астемизол отличается от других антигистаминных средств 2-го поколения://


+ необратимым связыванием с Н1-гистаминорецепторами//
более выраженным действием в первые дни приема//
возможностью монотерапии при атопической астме//
более выраженным седативным//
не отличается от других препаратов 2 поколения

***
Какой из антигистаминных препаратов имеет максимальный период полувыведения://


терфенадин//
+ астемизол//
лоратадин//
цетиризин//
дезлоратадин

***
Доза какого антигистаминного препарата должна быть снижена у больного с ХПН://


лоратадина//
+ цетиризина//
дезлоратадина//
клемастина//
фексофенадина

***
Какой антигистаминный препарат способен потенцировать эффекты алкоголя://


дезлоратадин//
фексофенадин//
+ цетиризин//
лоратадин//
эбастин

***
Наиболее частый побочный эффект при использовании интраназальной формы азеластина://


сонливость//
носовые кровотечения//
+ чувство горечи во рту//
повышение аппетита//
сердцебиение

***
Какой препарат оказывает максимальное противовоспалительное действие при лечении аллергических ринитов://


азеластин//
кромогликат//
+ беклазон//
лоратадин//
ксилометазолин

***
Основным показанием к назначению интраназального кромогликата является://


бронхиальная астма//
+ аллергический ринит//
рецидивирующая крапивница//
пищевая аллергия//
гнойный гайморит

***
К побочным эффектам интраназальных ГКС относятся все, кроме://


жжения в полости носа//
кандидоза полости носа//
носовых кровотечений//
+ угревой сыпи на лице//
сухости полости носа

***
Средством выбора для лечения сезонного аллергического ринита в период беременности является://


интраназальные ГКС//
интраназальные антигистаминные//
+ интраназальный кромогликат//
фексофенадин//
лоратадин

***
Укажите антигистаминный препарат, обладающий М-холинолитическим действием://


+ дифенгидрамин//
- хифенадин//
- лоратадин//
- фексофенадин//
- цетиризин

***
Выберите антигистаминный препарат, не обладающий седативным действием://


Дифенгидрамин//
Клемастин //
Хлоропирамин //
Прометазин //
+ Лоратадин

***
Выберите препарат, имеющий парентеральную форму://


+ тавегил//
кларитин//
зиртек//
телфаст//
эриус

***
Выберите препарат, выписанный под международным непатентованным названием://


тавегил//
кларитин//
зиртек//
+ лоратадин//
эриус

***
Какой ГКС предпочесть при аллергической реакции://


дексаметазон//
триамцинолон//
бетаметазон//
+ преднизолон//
гидрокортизон

***
В каком случае увеличивается риск появления аритмогенного (проаритмического) действия при использовании препаратов 1 класса://


При использовании меньших доз препарата//
+ При использовании 2-х антиаритмических препаратов//
На фоне гиперкалиемии//
У лиц до 50 лет и фракцией выброса 45%//
У лииц с фракцией выброса 25%

***
Укажите состояние, при котором необходима медикаментозная терапия://


+ Желудочковая экстрасистолия (более 30 экстрасистол за час)//
Ускоренный идиовентрикулярный ритм или ритм атриовентрикулярного соединения с нормальной ЧСС//
Предсердная экстрасистолия без признаков органического заболевания сердца//
АV блокада 1 степени//
SА – блокада

***
Препараты 1 класса являются по механизму действия://


+ Блокаторами натриевых каналов//
Блокаторами быстрых кальциевых каналов//
Блокаторами хлорных каналов//
Блокаторами калиевых каналов//
Блокаторами магниевых каналов

***
Укажите препараты, безопасные при нарушении проводимости в системе Гиса-Пуркинье://


Все препараты 1 класса//
Препараты 1 С подкласса//
+ Препараты 2 класса//
Препараты 1 А подкласса//
Все препараты 3 класса

***
Укажите неправильное утверждение о лидокаине://


Имеет мембраностабилизирующее действие//
Укорачивает эффективный рефрактерный период//
Не дается перорально//
+ Особенно эффективен при наджелудочковых аритмиях//
При стандартных дозах достигает более высоких концентраций в крови больных с ХСН, чем у больных без нее

***
Наиболее эффективный препарат для купирования суправентрикулярной тахикардии://


+ Верапамил//
Аймалин//
Мекситил//
Лидокаин//
Метопролол

***
Укажите антиаритмические препараты, не удлиняющие QT на ЭКГ://


Хинидин, новокаинамид//
Амиодарон, соталол//
Верапамил, хинидин//
+ Пропранолол, лидокаин//
Амиодарон, новокаинамид

***
Волчаночный синдром может возникнуть при применении://


Аллапенина//
+Пропафенона//
Прокаинамида//
Верапамила//
Пропранолола

***
Укажите препарат, который может спровоцировать приступ предсердной тахикаритмии при синдроме WPW://


+ Дигоксин//
Амиодарон//
Пропранолол//
Этмозин//
Лидокаин

***
Укажите препарат, имеющий холинолитический побочный эффект://


Лидокаин//
Амиодарон//
Верапамил//
+ Новокаинамид//
Бисопролол

***
При каких видах нарушений ритма сердца применение верапамила является противопоказанным?//


Предсердная экстрасистолия//
Трепетание предсердий//
+ Пароксизм мерцательной аритмии при синдроме WPW//
Мерцательная аритмия//
Фибрилляция предсердий

***
Укажите состояние, когда назначение циннаризина нежелательно://


ОНМК//
Мигрень//
Лабиринтные расстройства//
+ Болезнь Паркинсона//
Нарушение переферического кровообращения

***
Укажите препарат, не относящийся к нейроаминокислотам://


+ Танакан//
Глутаминовая кислота//
Пантогам//
Фенибут//
Пикамилон

***
Отметьте механизм действия холина альфосцерата://


+ Обладает холиномиметическим действием//
Обладает холинолитическим действием//
Прямое влияние на гладкую мускулатуру сосудов//
Тормозит возбудимость центральных вазомоторных центров//
Блокада переферических -адренорецепторов

***
Укажите состав препарата фезам://


Танакан 40 мг + циннаризин 25мг//
Пирацетам 400 мг. + винпоцетин 5 мг//
+ Пирацетам 400 мг + циннаризин 25 мг//
Пентоксифиллин 400 мг + циннаризин 25 мг//
Винпоцетин 5 мг + флунаризин 5 мг

***
Укажите неправильное утверждение о мексидоле://


Оказывает церебропротекторное действие//
Обладает ноотропным действием//
Улучшает реологию крови//
Является транквилизатором дневного типа//
+ Оказывает атерогенное действие

***
Укажите состав инстенона://


Гексобендина гидрохлорид 5 мг + экстракт гинго билоба + этамиван 25 мг//
Этамиван 25 мг + этофиллин 50 мг + пиридитол 25 мг//
+ Гексобендина гидрохлорид 5 мг + этамиван 25 мг + этофиллин 50 мг//
Гексобендина гидрохлорид 5 мг + этофиллин 50 мг + пантогам 50 мг//
Этамиван 25 мг + этофиллин 50 мг + агапурин 400 мг

***
Назовите состояние, когда прием нейролептиков противопоказан://


Психомоторное возбуждение//
Алкогольный психоз//
Острое бредовое состояние//
+ Болезнь Паркинсона//
«Неукротимая» рвота

***
Отметьте основные недостатки короткодействующих инсулинов://


+ Замедленное начало – ч\з 30 мин., длительность действия – до 8 ч.//
Быстрое начало – ч\з 3 мин., длительность 2 ч.//
Начало действия – ч\з 10 мин., длительность – до 4 ч.//
Замедленное начало – ч\з 1 час., длительность действия – до 6ч.//
Медленное начало – ч\з 3 час., длительность – 1 ч.

***
Назовите короткодействующий аналог человеческого инсулина://


Хумулин//
+ Хумалог//
Лантус//
Протофан//
Ультралонг

***
Назовите аналог инсулина длительного действия://


Хумулин//
Хумалог//
Ново Рапид//
+ Лантус//
Монотард

***
Укажите, что не относится к осложнениям инсулинотерапии://


Феномен Самоджи//
Инсулинорезистентность//
Нарушение зрения//
Инсулиновые отеки//
+ Гиперурикемия
***
Отметьте механизм действия производных сульфанилмочевины://
Стимулируют высвобождение инсулина из b-клеток поджелудочной железы//
Снижение уровня глюкагона//
Увеличение количества инсулиновых рецепторов в клетках//
+ Все перечисленное//
- Ничего из перечисленного

***
Укажите препарат, не относящийся к производным сульфанилмочевины II генерации://


глибенкламид//
+ хлорпропамид//
гликвидон//
гликлазид//
глимепирид

***
Укажите побочный эффект, не относящийся к производным сульфанилмочевины://


Гепатотоксичность//
Диспепсические явления//
Гематологические нарушения//
+ Нарушения ритма сердца//
Увеличение массы тела

***
Укажите препарат сульфанилмочевины, действующий 24 часа://


+ глимепирид//
гликвидон//
гликлазид//
глибенкламид//
Ничего из перечисленного

***
Укажите дополнительный эффект диабетона://


снижение HCL//
+снижение адгезии и агрегации тромбоцитов//
снижение АД//
увеличение ХС ЛПВП//
нормализация моторики ЖКТ

***
Укажите состав препарата глибомет://


глибенкламид2,5 мг + акарбоза 100мг//
метформин 400 мг + акарбоза 100мг//
метформин 400 мг+ репаглинид 0,5 г//
глибенкламид 2,5 мг + пиоглитазон 30мг//
+ глибенкламид 2,5 мг+ метформин 400 мг

***
Укажите неправильное утверждение о метформине://


Снижает переферическую инсулинорезистентность//
Подавляет ГНГ//
+ Стимулируют высвобождение инсулина из b-клеток поджелудочной железы//
Усиливает анаэробный гликолиз//
Снижает агрегацию тромбоцитов

***
Что не относится к побочным эффектам бигуанидов://


Аллергические реакции//
Диарея//
актацидоз//
+Увеличение массы тела//
Металлический вкус во рту

***
Укажите препарат, относящийся к прандиальным регуляторам://


Пиоглитазон (актос)//
метформин (сиофор)//
глимепирид (амарил)//
+ Репаглинид (новонорм)//
Ничего из перечисленного

***
Местные анестетики в большей степени влияют://


+ на болевую чувствительность//
на тактильную чувствительность//
на температурную чувствительность//
на моторную функцию//
на сенсорную чувствительность

***
Местные анестетики влияют на моторную функцию://


в низких дозах//
+ в высоких дозах//
в среднетерапевтических дозах//
отсутствует влияние на моторную функцию//
влияние на моторную функцию не зависит от дозы

***
Отметьте механизм действия местных анестетиков://


+ блокада натриевых каналов//
блокада калиевых каналов//
увеличение цитоплазматического кальция//
блокада ЦОГ//
активация фосфолипазы

***
Выберите местный анестетик, вызывающий эйфорию://


новокаин//
+ кокаин//
лидокаин//
бупивакаин//
тетракаин

***
Выберите местный анестетик с антиаритмическим эффектом при в\в введении://


новокаин//
бупивакаин//
тетракаин//
+ лидокаин//
артикаин

***
Для пролонгации эффекта местные анестетики часто используют в комбинации://


+ с адреналином//
анестезином//
β-адреноблокаторами//
допмином//
папаверином

***
Какой местный анестетик имеет минимальный сосудорасширяющий эффект://


новокаин//
лидокаин//
бензокаин//
+ мепивакаин//
артикаин

***
Какой местный анестетик имеет in vitro антибактериальный эффект://


+ лидокаин//
бензокаин//
артикаин//
мепивакаин//
бупивакаин

***
Снижение эффективности местных анестетиков в очаге воспаления объясняется://


+ снижением pH при воспалении//
повышением pH при воспалении//
сгущением крови//
вазодилатирующим действием//
воспалением

***
При заболеваниях печени можно получить передозировку следующего анестетика://


новокаина//
тетракаина//
бензокаина//
кокаина//
+ лидокаина

***
Отметьте наиболее токсичный местный анестетик://


+ лидокаин//
новокаин//
артикаин//
мепивакаин//
все препараты одинаково токсичны

***
Выберите препарат с наиболее слабым анестезирующим эффектом://


+ новокаин//
лидокаин//
мепивакаин//
артикаин//
бупивакаин

***
Выберите препарат с наибольшим анестезирующим эффектом://


+ бупивакаин//
артикаин//
мепивакаин//
лидокаин//
новокаин

***
Сочетание гипотонии и брадикардии наиболее вероятно при передозировке://


+ клофелина//
нифедипина//
каптоприла//
празозина//
нитроглицерина

***
Какой из перечисленных препаратов не повышает атерогенность плазмы крови://


фуросемид//
+ нифедипин//
диакарб//
гипотиазид//
пропранолол

***
Какой из препаратов даст наименьшие колебания артериального давления в течение суток://


нифедипин//
+ амлодипин//
верапамил//
нифедипин-ретард SR//
фелодипин

***
Какой вид взаимодействия вызывает совместное назначение рамиприла и гипотиазида://


суммация эффектов//
+ потенцирование//
аддитивное действие//
сенситизация//
антагонизм

***
Назначение лекарственного средства «offlabal» - это://


применение лекарственного средства для повышения эффективности другого лекарственного средства//
+ применение лекарственного средства не по показаниям, регламентированными в инструкции или с другими нарушениями инструкции//
применение лекарственного средства в качестве терапии прикрытия//
применение лекарственного средства не по стандартам//
применение лекарственного средства по жизненным показаниям

***



Достарыңызбен бөлісу:
1   2   3   4   5




©dereksiz.org 2024
әкімшілігінің қараңыз

    Басты бет